Wikipedia:Auskunft/Archiv/2014/Woche 01

aus Wikipedia, der freien Enzyklopädie
Zur Navigation springen Zur Suche springen


Dies ist ein Archiv der Seite Wikipedia:Auskunft. Es enthält alle Abschnitte, die in der Kalender-Woche 01 im Jahr 2014 begonnen wurden.

Möchtest du in einer neuen Diskussion auf dieses Archiv verweisen? Nutze folgenden Link und ersetze ________ durch den Namen des Abschnittes. Die Nummer im Inhaltsverzeichnis gehört nicht dazu:

[[Wikipedia:Auskunft/Archiv/2014/Woche 01#________]]


← vorherige Woche Gesamtarchiv nächste Woche →

Feuerwerksfrage 2: Schwarzpulverrakete in den Weltraum

Wäre es möglich, mit einer Schwarzpulverrakete nach Art der Silvesterrakete Satelliten in den Weltraum oder Lower Earth Orbit zu bringen? Wie müsste eine derartige Rakete aufgebaut sein? --Häpinujir (Diskussion) 00:39, 1. Jan. 2014 (CET)

Das ist dann eine Feststoffrakete. Die werden meistens als Booster beim Raketenstart benutzt. --Mauerquadrant (Diskussion) 00:56, 1. Jan. 2014 (CET)
Fliegen zwar nicht in den Weltraum, aber praktisch alle Raketenwaffen/Flugabwehrraketen sind Feststoffraketen. --79.232.200.63 09:49, 1. Jan. 2014 (CET)
" mit einer Schwarzpulverrakete nach Art der Silvesterrakete Satelliten in den Weltraum ... zu bringen" Jules Verne war der Meinung, dass es mit poudre à canon (= Schwarzpulver) bis zum Mond klappt. GEEZER… nil nisi bene 10:59, 1. Jan. 2014 (CET)
kleine Korrekturen. Vernes tapfere Mondfahrer haben eine Kanone und keine Rakete und auch kein Schwarzpulver sondern fulmi-coton benutzt. Funktioniert aber nur im Buch. --Rubblesby (Diskussion) 11:16, 1. Jan. 2014 (CET)
Stimmt. Etwas gelernt! Fulmi-coton war moderner und das hat Jules gleich eingebaut. GEEZER… nil nisi bene 11:32, 1. Jan. 2014 (CET)
Eine interessante Frage der Mittelstufenphysik. Die Rakete müsste soviel Schub enwicklen, um das ganze Vehikel (Rakete mit dem zum Erreichen der gewünschten Höhe (Weltraum = 100 km) erforderlichen Treibstoff plus Satellit) abheben zu lassen. --Fqugdvin (Diskussion) 11:22, 1. Jan. 2014 (CET)
Stimmt, da war was in der Mittelstufe. Wenn es mir recht gedenkt, hat unser Lehrer uns erklärt, die Rakete müsste die erste kosmische Geschwindigkeit erreichen, und das ginge nach der Raketenformel nicht mit Schwarzpulver, weil die Energieausbeute im Verhältnis zum Gewicht zu gering sei. Stimmt das? Grüße Dumbox (Diskussion) 11:44, 1. Jan. 2014 (CET)
Das Problem ist wahrscheinlich eher die negative Sauerstoffbilanz? Oder man gibt ihr soviel Schub in niedrigen Höhen, dass es bis 100 km reicht? Ich glaube, die 1. kosm. Geschwindigkeit muss nicht erreicht werden, es reicht wenn die Rakete 100 km hoch kommt und dann herunterfällt. --Fqugdvin (Diskussion) 11:54, 1. Jan. 2014 (CET)
Wobei ich gerade merke, dass du einen Satelliten gefordert hast. Also doch 1. kosm. Geschwindigkeit. Mit der Jules-Verne-Kanone geht das wahrscheinlich. Wie schwer soll dein Satellit denn sein?
Ich bezweifle, dass eine Kanone funktionieren würde. Eine Mündungsgeschwindigkeit von 7900 m/s ist das viereinhalbfache der Mündungsgeschwindigkeit einer DM53, wie sie beim Leopard II eingesetzt wird. Dazu käme dann noch der Luftwiderstand. Ich meinte eigentlich irgendeinen Satelliten, beispielsweise einen Cubesat o.ä. --Häpinujir (Diskussion) 14:46, 1. Jan. 2014 (CET)
Dann musst du nur noch ausrechnen, mit wieviel Mündungsenergie du 1,33 kg auf 28476 km/h = 7910 m/s beschleunigen kannst (es sind 83,2 Megajoule), und wieviel Schwarzpulver du dafür benötigst. --Fqugdvin (Diskussion) 15:12, 1. Jan. 2014 (CET)

Wir haben das früher mal ausgerechnet und es ging nicht. Man kann auch zum Beispiel nicht mal die leere Hülse einer Schwarzpulverrakete in eine Umlaufbahn schießen, egal wie groß sie ist. 87.181.38.14 21:59, 1. Jan. 2014 (CET)

Ich meine mich zu erinnern, dass man für einen Satelliten bzw. eine stabile Umlaufbahn den Schub regeln können muss. Das geht mit Schwarzpulver nicht, man braucht Flüssigtreibstoff. --92.202.2.156 22:27, 1. Jan. 2014 (CET)

Danke für alle Antworten. --Häpinujir (Diskussion) 00:27, 2. Jan. 2014 (CET)

Archivierung dieses Abschnittes wurde gewünscht von: Dieser Abschnitt kann ins Archiv geschossen werden. --Häpinujir (Diskussion) 00:27, 2. Jan. 2014 (CET)

Elternzeit

Ich (werdender Vater) möchte nach der Geburt unseres Kindes zwei bis sechs Monate Elternzeit nehmen. Muss ich dabei auf die Belange des Arbeitgebers Rücksicht nehmen (analog zum Erholungsurlaub) oder kann ich festlegen, dass ich von April bis September zu Hause bleibe und die Vertretungssituation (gerade im Sommer immer etwas angespannt) ignorieren? Danke. --141.15.33.1 11:24, 2. Jan. 2014 (CET)

Muss ich dabei auf die Belange des Arbeitgebers Rücksicht nehmen => Nein, Du kannst Beginn und Dauer der Elternzeit selbst frei wählen, musst den Arbeitgeber allerdings rechtzeitig (mind. 7 Wochen vor Antritt) informieren. Siehe auch [1].--Zockmann (Diskussion) 11:39, 2. Jan. 2014 (CET)
Danke dir.
*quetsch* Ich finde schon, Du hättest bei der Terminierung der Zeugung etwas mehr auf die Belange des Arbeitgebers Rücksicht nehmen können. ;-) *duckundweg* --Anna (Diskussion) 02:33, 3. Jan. 2014 (CET)
Archivierung dieses Abschnittes wurde gewünscht von: 141.15.33.1 12:19, 2. Jan. 2014 (CET)

Mathematik - Binomische Formel

Bei binomischen Formel gibt es doch die Standardformel:

  • (a+b)²= a²+2ab+b²
  • (a-b)²= a²-2ab-b²
  • (a+b)(a-b)= a²-b²

Jetzt gab es zu Hause eine Diskussion, worum die Frage ging

Was muss jetzt die binomische Formel sein, damit jetzt in der binomischen Formel also ( )² stehen, damit hinten also nach dem =(a-b)*(a-b). Jetzt noch einmal in binomischer Form:

( )²=(a-b)(a-b).

Diese Formel gibt es in der Diskussion zu klären, was leider ohne Ergebnis blieb.

--Iralopp (Diskussion) 13:39, 2. Jan. 2014 (CET)

"a-b" --Eike (Diskussion) 13:40, 2. Jan. 2014 (CET)
Du hast offensichtlich meine Frage missverstanden! Die Aufgabe lautet:

( )²=(a-b)(a-b) und wenn ich (a-b)² mache, dann kommt die zweite Formel von den Standardformel (siehe oben) heraus. Bitte so ausfüllen diese Lücke, dass zwei mal Minus herauskommt. --Iralopp (Diskussion) 13:43, 2. Jan. 2014 (CET)

Immer noch (a-b)² = (a-b)(a-b)
Weil für jedes x gilt: (x)²=(x)(x)
Auch für x=(a-b).
Dass (a-b)² auch zu etwas anderem umgeformt werden kann, spielt dafür keine Rolle.
--Eike (Diskussion) 13:45, 2. Jan. 2014 (CET)
Im übrigen muss es heißen (a-b)² = a²-2ab + b², denn (-b)*(-b) = +b² -- Janka (Diskussion) 13:54, 2. Jan. 2014 (CET)
Archivierung dieses Abschnittes wurde gewünscht von: Momotaro 18:11, 2. Jan. 2014 (CET)

Feuerwerksfrage 1: Farben des Feuerwerks

Haben die Farben des Silvester-Feuerwerks (rot, grün. blauviolett, „Temperaturstrahler“) eigentlich eine bestimmte Bedeutung nach Art der Farbsymbolik oder des Hanky Code?--Häpinujir (Diskussion) 00:36, 1. Jan. 2014 (CET)

Völkermord an den Armeniern

Sehr geehrte Damen und herren, welche persönliche Schuld har der deutsche General von Bronsat zu Schellendorf beim Völkermord an den Armeniern


--62.246.114.222 10:32, 1. Jan. 2014 (CET)

Friedrich Bronsart von Schellendorf hieß er. Wenn dir die Informationen im Artikel nicht reichen, solltest du dir mit der dort genannten Literatur, die sicher weitere Literaturangaben enthält, eine fundierte eigene Meinung bilden können. 85.180.197.45 10:55, 1. Jan. 2014 (CET)

Wann mit normaler kieferorthopädischer Behandlung beginnen?

Kieferorthopädie ist neben dem medizinisch notwendigem auch ein gutes Geschäft. Demzufolge ist die Spange seit Jahren der Standard, anstatt wie vor 20 Jahren die Ausnahme. Informationen bei den Top-Googletreffern sind meist pro-Spange (aus bekannten Gründen) am besten schon in möglichst frühen Jahren. Demgegenüber sieht man (jedenfalls in Großstädten) viele Erwachsene mit einer Spange (aus Ästhetikgründen) rumlaufen. Wenn es nicht medizinische notwendig ist, sollte man die Entscheidung über kieferorthopädische Maßnahmen nicht den Heranwachsenen überlassen (ähnlich der Bechneidung)?--77.185.206.132 10:34, 1. Jan. 2014 (CET)

Nein, "Zahnspange" (Korrektur frühestens ab 9 Jahren) ist wie "Phimose"-Operation (kein gutes Beispiel, da auch Mädchen Zahnspangen bekommen...) oder das Anlegen von abstehenden Ohren oder Spreizhose; "Beschneidung" ist wie Entfernung der Schneidezähne oder Coupieren der Ohren oder Umwicklung der Füsse. GEEZER… nil nisi bene 10:47, 1. Jan. 2014 (CET)
Das mit dem guten Geschäft stimmt schon. Aus persönlicher (Laien-!)Erfahrung würde ich warten, bis die Milchzähne draußen sind, sonst gibt es alle paar Wochen eine neue Spange. 85.180.197.45 11:01, 1. Jan. 2014 (CET)

Mythos Simón Bolívar

Um was handelt es sich genau beim Mythos von Simón Bolívar? Ich habe ihn nämlich nicht wirklich verstanden, da es nicht offensichtlich genug angegeben wurde in den Quellen, die ich gefunden habe.--Puu Oo (Diskussion) 13:35, 1. Jan. 2014 (CET)

Siehe Politischer Mythos und Simón Bolívar. --Chricho ¹ ² ³ 13:40, 1. Jan. 2014 (CET)
Ich darf Wikipedia nicht als Quelle brauchen hat mir meine Geschichtslehrerin gesagt.--Puu Oo (Diskussion) 13:41, 1. Jan. 2014 (CET)
Dazu ist der Vortrag, den ich darüber halten werde auf französich.--Puu Oo (Diskussion) 13:42, 1. Jan. 2014 (CET)
Du hast nach Verständnis gefragt, nicht nach Quellen. --Eike (Diskussion) 13:53, 1. Jan. 2014 (CET)
Ja, aber dort verstehe ich es auch nicht.--Puu Oo (Diskussion) 13:54, 1. Jan. 2014 (CET)
Was ist bei den Artikeln der Mythos? Es gibt keinen Absatz dazu!--Puu Oo (Diskussion) 13:57, 1. Jan. 2014 (CET)
Du meinst, da steht nirgendwo "Mythos" drüber? --Eike (Diskussion) 14:23, 1. Jan. 2014 (CET)
Wenn du das Referat auf Französisch halten musst, wäre fr:Simón Bolivar#Hommage und fr:Bolivarisme schon mal ein Ausgangspunkt. --Jossi (Diskussion) 14:25, 1. Jan. 2014 (CET)
Genau!--Puu Oo (Diskussion) 14:29, 1. Jan. 2014 (CET)
(BK)Der Mythos ist, dass um ihn aufgrund seiner politischen Leistungen ein Personenkult gemacht wird. Die politischen Leistungen von S.B. sind im Artikel aufgeführt und bequellt. Diese Quellen kannst Du für Dein Referat heranziehen. Auf fr:Simón Bolívar sind auch französischsprachige Quellen aufgeführt, falls Dir das weiterhilft. --Rôtkæppchen68 14:28, 1. Jan. 2014 (CET)
Das genau war zu dem, was Eike gesagt hat. Aber auf französisch habe ich nichts gefunden, deshalb habe ich es auf deutsch versucht.--Puu Oo (Diskussion) 14:31, 1. Jan. 2014 (CET)

Wann werden die Tage wieder länger?

--109.193.194.65 13:44, 1. Jan. 2014 (CET) Wann werden die Tage ab Januar wieder länger?--109.193.194.65 13:44, 1. Jan. 2014 (CET)

--109.193.194.65 13:44, 1. Jan. 2014 (CET)

Ich möchte wissen wann die Tage länger werden ?
Jetzt schon.--Puu Oo (Diskussion) 13:45, 1. Jan. 2014 (CET)
Seit Dezember schon. Siehe Wintersonnenwende. —[ˈjøːˌmaˑ] 13:47, 1. Jan. 2014 (CET)
Das Datum ist bekannt, der 21. Dezember.--Puu Oo (Diskussion) 13:53, 1. Jan. 2014 (CET)
Manchmal isses auch der 22., daher schlug ich die Lektüre des Artikels vor. —[ˈjøːˌmaˑ] 15:35, 1. Jan. 2014 (CET)
Hier kannst du nach den Sonnenaufgangs- und Sonnenuntergangsdaten deines Ortes suchen. Den Artikel Tageslänge haben wir auch noch. --Mauerquadrant (Diskussion) 15:18, 1. Jan. 2014 (CET)

Buch

wo kann ich ein altes buch neu binden lassen in frankfurt/main? --85.180.136.48 06:18, 3. Jan. 2014 (CET)

Das erfährst Du in den Gelben Seiten. --Rôtkæppchen68 07:08, 3. Jan. 2014 (CET)
Archivierung dieses Abschnittes wurde gewünscht von: keine Wissensfrage Rôtkæppchen68 07:08, 3. Jan. 2014 (CET)
Wieso ist das keine Wissensfrage, der weiß es doch nicht und fragt deshalb. Nicht jeder ist so ein erfahrender Intelligenzbolzen, der auf alles sofort eine eigene Antwort hat.--87.162.251.166 08:36, 3. Jan. 2014 (CET)
Der Fragesteller hat die Einleitung nicht gelesen. Google:buchbinder+frankfurt hätte ihm vermutlich sofort geholfen. Leicht ergoogelbare Fragen sind nunmal keine enzyklopädischen Wissensfragen. --Rôtkæppchen68 09:22, 3. Jan. 2014 (CET)
Hu, hu, hu, ha, ha, ha Eigentor: Dein Link führt ausschließlich zur Autovermietung Buchbinder.--87.162.251.166 10:28, 3. Jan. 2014 (CET)
Ha! Ich hingegen biete eine Suchabfrage, die funktioniert buch binden frankfurt -- southpark 10:32, 3. Jan. 2014 (CET)
Ich weiß ja nicht, was dein Google ausspuckt, 87.162.251.166, aber bei mir steht an fünfter Stelle http://www.gelbeseiten.de/buchbinder/frankfurt und an der siebten http://www.dasoertliche.de/Themen/Buchbindereien/Frankfurt-am-Main.html . --Eike (Diskussion) 10:33, 3. Jan. 2014 (CET)
Das dürfte doch mittlerweile bekannt sein, dass Google seine Ergebnisse personalisiert. Mir als bekennendem Nichtführerscheininhaber bietet Google natürlich erst einmal ein paar Links auf den Autovermieter, aber noch auf der ersten Seite zielführende Suchergebnisse. --Rôtkæppchen68 11:40, 3. Jan. 2014 (CET)

Die gelben Seiten sind hierfür ein gutes Werkzeug, der dort verwendete Suchbegriff lautet „Buchbindereien“. Gelistet werden 15 Einträge für ganz Frankfurt, so dass die integrierte Umkreissuche helfen kann, den Suchradius einzugrenzen. Darüber hinaus wäre eine kleine Druckerei in der Nähe ebenfalls ein kompetenter Ansprechpartner: Selbst wenn dort keine Bücher gebunden werden sind entsprechende Kontakte vorhanden. --84.178.63.216 12:46, 3. Jan. 2014 (CET)

Literaturverzeichnis - Angabe einer Namensänderung

Liebes Team,

ich habe bereits vor einiger Zeit auf dieser Seite: https://de.wikipedia.org/wiki/Russische_Staatsbibliothek zu meinem Mädchennamen meinen jetzigen Namen ergänzt. Heute habe ich durch Zufall gesehen, dass er wieder weg war. Ich habe dies dann ohne mich zu registrieren wieder geändert. Nun habe ich mich aber doch noch registriert. Wie kann man es machen, dass mein Name, den ich bei der Hochzeit angenommen habe, nicht wieder irgendwann verschwindet? Als "Ursula Maria Müller" kennt mich keiner mehr...

Danke fürs Feedback!

--Magistra1992 (Diskussion) 14:34, 3. Jan. 2014 (CET)

Ich weiß nicht, ob wir dafür eine Reglung haben, aber ich würde einfach den Namen verwenden, der auf dem Buch steht. Oder hat sich das auch geändert, mit einer neuen Auflage oder so? Besser aufgehoben wäre die Frage auf Wikipedia:Fragen zur Wikipedia, da weiß bestimmt einer, ob wir uns für den Fall schon etwas ausgedacht haben. --Eike (Diskussion) 14:36, 3. Jan. 2014 (CET)

Danke! Es war ja ein Zeitschriftenartikel, d.h. es gab keine Neuauflage bzw. wird auch nie eine geben. Im Artikel steht logischerweise nur der Mädchenname, den ich dann zwei Jahre später abgelegt habe. Danke auch für den Hinweis auf "Fragen zur Wikipedia". (nicht signierter Beitrag von Magistra1992 (Diskussion | Beiträge) 14:56, 3. Jan. 2014 (CET))

Archivierung dieses Abschnittes wurde gewünscht von: Weiter geht's auf Fragen zur Wikipedia... --Eike (Diskussion) 14:59, 3. Jan. 2014 (CET)

Welcher Boot kann in allen Aktuellen Landkreisen, Kreisen, Stadtkreisen und kreisfreien Städten die eine Schlüsselnummer haben

, den Kfz-Parameter aus der Infobox Landkreis entfernen ? Altkreise sollen davon nicht berührt werden ! Dies gilt nur für Deutschland ! Die Antwort bitte auf Vorlage Diskussion:Metadaten Kfz-Kennzeichen DE --Thomas021071 (Diskussion) 00:57, 4. Jan. 2014 (CET)

Die Frage dürfte auf WP:FzW besser aufgehoben sein. --Stefan1973HB Disk. 01:00, 4. Jan. 2014 (CET)

verschoben --Thomas021071 (Diskussion) 01:04, 4. Jan. 2014 (CET)

Archivierung dieses Abschnittes wurde gewünscht von: Stefan1973HB Disk. 01:06, 4. Jan. 2014 (CET)

Stadtbahn Paderborn

Leider keinen Wiki Artikel zur geplanten stadtbahn Paderborn gefunden

--62.246.114.222 10:53, 1. Jan. 2014 (CET)

Das ist richtig, denn ein Artikel über eine eventuelle zukünftige Stadtbahn Paderborn wäre reine Glaskugelei. Das, was im Internet darüber zu finden ist, ist sehr dünne: Planung, bestehenden Regionalbahnstrecken auf Stadtbahnbetrieb umzustellen. --Rôtkæppchen68 11:20, 1. Jan. 2014 (CET)
In den Artikeln Paderborner Elektrizitätswerke und Straßenbahn AG und PaderSprinter findet sich jedoch etwas zur ehemaligen Straßenbahn Paderborn. --DaBroMfld (Diskussion) 21:54, 1. Jan. 2014 (CET)

Kalenderdruck

Ich möchte den Kalender 2014 drucken, zwecks Ferien-/Feiertagsplanung von Familie und Mitarbeitern. Nach Lektüre der Bulle Inter gravissimas stellen sich mir folgende Fragen: Auf welche Bankverbindung des Vatikans sind die 100 Golddukaten einzuzahlen? Wird auch Euro und/oder Paypal akzeptiert? Gilt das für mich überhaupt noch, weil ich nach dem Ausdruck des Kalenders ja automatisch (latae sententiae) nicht mehr Katholik sein werde? Oder hat einer diese Vorschrift mittlerweile aufgehoben? Wenn ja, wer und wann?

--Fqugdvin (Diskussion) 11:28, 1. Jan. 2014 (CET)

Hier steht nur "Haec prohibitio fuit postea revocata", leider ohne genauere Angaben. Aber es wird schon stimmen; imprime fortiter, et ama Deum! Grüße Dumbox (Diskussion) 11:37, 1. Jan. 2014 (CET)
Da fällt mir ein Stein vom Herzen. --Fqugdvin (Diskussion) 11:46, 1. Jan. 2014 (CET)
Da sich hier einige Antworter mit dem Thema auskennen, bitte ich um ein paar Belege für den Artikel Inter gravissimas. Auch die en:WP hat dieses Problem. GEEZER… nil nisi bene 12:24, 1. Jan. 2014 (CET) Nehmen das zurück. Verwechseln immer wieder den Unterschied zwischen Glauben und Wissen. Webseiten reichen da völlig aus. GEEZER… nil nisi bene 14:39, 1. Jan. 2014 (CET)
Klang meine Antwort wirklich so, als kennte ich mich in der Materie aus? In diesem Zeitungsartikel lese ich, dass das Druckmonopol noch 1582 gelockert wurde, was immer das heißt. Vor allem aber wird auf das Buch Egidio Mezzi, Francesco Vizza, Luigi Lilio: medico astronomo e matematico di Cirò : ideatore della riforma del calendario gregoriano verwiesen. Hat jemand logistischen und sprachlichen Zugang zu diesem Werk? Ansonsten könnte man auch mal bei Frau Sieglinde Borvitz anfragen. Grüße Dumbox (Diskussion) 14:50, 1. Jan. 2014 (CET)
Also Absatz 13 sanktioniert ja nur das einem römischen Drucker erteilte Druck-Privileg (ohne Namensnennung in Absatz 12). Die 100 Dukaten werden nur für Untertanen des Kirchenstaats fällig. Drucker in auswärtigen Territorien (Rest der Welt) müssen eine päpstliche Lizenz erwerben. Die Gesetzeslage dazu dürfte diejenige sein, die sich inzwischen geändert hat. Solche Vorformen des Copyrights wurden damals selten länger als auf 10 Jahre gewährt. Das hätte ja auch der Idee der maximalen Verbreitung der Anordnung widersprochen. --Laurentianus (Diskussion) 18:14, 1. Jan. 2014 (CET)
Hmm, interessant was Gregor unter "Jurisdiktion" verstand. Wenn man nicht im Kirchenstaat lebt, und die 100 Golddukaten somit nicht fällig werden (meine Ferienpläne sind gerettet), wie kann er dann die Exkommunikation ("und andere unserem Urteil unterstehenden Strafen") erklären? --Fqugdvin (Diskussion) 18:32, 1. Jan. 2014 (CET)
Das eine ist die Territorialgewalt, die er als Souverän des Kirchenstaates ausübt. Die Androhung der Exkommunikation, die übrigens nichts daran ändert, daß man Katholik ist (das kann nicht mal der Papst beschließen), fließt aus der geistlichen Leitungsgewalt, die der Papst hier als Druckmittel einsetzt, wie auch sonst öfters. Schönen Urlaub! --Laurentianus (Diskussion) 18:48, 1. Jan. 2014 (CET)

Soziale Herkunft der Bootsflüchtlinge

Bei etwas näherer Betrachtung der Berichterstattung zu dem Thema stellt amn fest dass das nicht wirklich besonders arme Leute sind die an Südeuropas Küsten angeschnwemmt werden. Ein vierstelliger Betrag für die Reise muss aufgebracht werden, die Leute sprechen ganz ordentlich die Kolonialsprache und wenn es Tote gibt können die Angehörigen schon mal per Flugzeug zur Identifizierung anreisen. Daraus lässt sich schließen dass es dabei nicht wirklich um "Lumpenproletariat", sondern eher um Angehörige der oberen Unter- bzw. unteren Mittelschicht der Städte handelt, denen mangelnde eigene Fähigkeiten und/oder die Situation in den Herkunftsländern (weiteren) sozialen Aufstieg versperrt (damit etwa die Schichten die vor einem halben Jahrhundert die Dekolonisation vorangetrieben haben). Wer hat nähere Informationen dazu?--Antemister (Diskussion) 13:10, 1. Jan. 2014 (CET)

Das ist doch aber bei ALLEN Flüchtlingsbewegungen so, egal ob heute, früher, oder noch früherer. Ist wie beim Spenden/Betteln: Nicht der Bedürftigste bekommt, sondern der "Lauteste".--Wikiseidank (Diskussion) 13:22, 1. Jan. 2014 (CET)

Es handelt sich in der Regel nicht um "Flüchtlinge" im Wortsinne, sondern um (illegale) Immigranten. Die FAZ von heute http://www.faz.net/aktuell/feuilleton/italien-die-lampedusa-realitaet-12729834.html --Geometretos (Diskussion) 13:56, 1. Jan. 2014 (CET)

Selten so einen dummen Artikel gelesen. Achso, FAZ, Feuilleton. Alles klar. --92.202.2.156 22:18, 1. Jan. 2014 (CET)
Wirklich fundierte Kritik, Alle Achtung!--80.129.182.176 06:11, 2. Jan. 2014 (CET)
„nicht wirklich besonders arme Leute ...“ – aus der Fernsehberichterstattung meine ich zu wissen, dass zumindest einige von denen vorher einen Kredit aufnehmen (und danach in entsprechende Abhängigkeiten geraten, falls ihr Plan nicht funktioniert). Im übrigen gibt es bei der von Antemister genannten Gruppe noch folgendes zu berücksichtigen:
Entweder die Leute sind tatsächlich schlecht ausgebildet und werden mit falschen Versprechungen von anderen Leuten, die damit ihr Geld verdienen, nach Europa gelockt. Die sollten dann zumindest theoretisch in ihren Heimatländern besser einen Job finden können, weil dort noch mehr einfache Tätigkeiten nachgefragt werden (wenig Industrialisierung, keine Dienstleistungsgesellschaft).
Oder die Leute sind gut ausgebildet, können aber nicht arbeiten, weil es die politische Situation in ihrem Land nicht zulässt (schwacher Staat im bürgerkriegsähnlichen Zustand, institutionalisierte Diskriminierung usw.). Diese könnten hier schon eher einen Job finden und werden ihr Land auch früher oder später verlassen, weil es ihnen keine Perspektive mehr bietet (so eine Entscheidung trifft man nicht von heute auf morgen, sondern über einen längeren Zeitraum). --Verwaltungsgliederung (Diskussion) 16:54, 1. Jan. 2014 (CET)

Legasthenie / Akustische Differenzierung

Heute ausnahmsweise mal als IP.

Folgendes:

Jemand, eine sehr nett anzusehende Sie (Mitte 20), falls das zur Motivation der Auskunftgebenden beiträgt, hat Probleme mit Doppelkonsonanten und mit Wörtern, die sie noch nicht geschrieben gesehen hat (bspw. Aubergine/Oberschiene). Sie nennt das "Hörschwäche".

Sie kann Wörter mit einfachen und doppelten Konsonanten klanglich nicht immer unterscheiden, bspw. wen und wenn, oder fals und falls. Auch meine ich, dass sie nicht immer sauber Buchstaben voneinander abgrenzen kann (bspw. abgelenkt/abgelengt). Letzteres ist aber nicht so ausgeprägt.

Wenn merkt sie sich mittlerweile über eine Brücke zu kann, weil sie letzteres mit Doppel-n schreibt. Sie spricht dann beides vorher laut aus. Auch, weil ich ihr gesagt habe, dass wen "langsam gesprochen" wird, Stichwort "lange und kurze Vokale".

Im Internet habe ich Übungsblätter zur akustischen Differenzierung und zu Doppelkonsonanten gefunden. Bei der Übung zur akust. Diff. soll man Striche bzw. Punkte unter die Vokale zeichnen, je nachdem, ob der Vokal kurz oder lang betont wird. Macht es Sinn, das Training so zu beginnen, d. h. mit dem Lesen und vermutlich lauten Aussprechen dieser Wörter und anschließendem Markieren der Vokale? Alternativ könnte man ihr die Wörter ja auch vorlesen oder diktieren.

Bei den Doppelkonsonanten sollen Lücken ausgefüllt werden, bspw. Doppel-r oder nicht.

Macht es darüber hinaus Sinn, ihr die einschlägigen Rechtschreibregeln zu vermitteln?

Ich wäre für weitere Tipps, Hinweise und für Links zu möglichst kostenlosen Übungsmaterialien sehr dankbar. Professionelle Hilfe kommt nicht in Betracht. Die Schwächen beeinträchtigen sie nicht in ihrem beruflichen Alltag.

Danke und Gruß, --87.172.217.205 14:09, 1. Jan. 2014 (CET)

Die Zeiten, in denen Deutsch nach Gehör geschrieben wurde, sind lange vorbei. Das funktioniert leider heute nicht mehr. Sie müsste eben lesen, vielleicht eine Zeitung abonnieren, nach Eingewöhnung vielleicht sogar ein Buch lesen. Wörter wie Aubergine/Oberschiene folgen keinem erlernbaren System ('Au' war im Lateinischen und Französischen die Schreibweise für das geschlossene 'O', während das 'O' offen gesprochen wurde) und müssen vom Schriftbild her erlernt werden. --91.50.34.44 14:32, 1. Jan. 2014 (CET)
Danke schon mal für diesen Hinweis. Wenn man die anderen Fragen und Antworten hier so liest, fragt man sich schon, weshalb das ein oder andere Wort so geschrieben oder ausgesprochen wird, wie es geschrieben oder ausgesprochen wird. Da kommt man nämlich mit langer oder kurzer Betonung auch nicht weiter. Akustik dürfte für jmd. mit dieser Schwäche bspw. verwirrend sein... Man muss dafür doch mind. eine weitere Regel kennen, um das Wort nicht mit Doppel-s zu schreiben. Oder wir sprechen es alle falsch aus... Wie du schon angemerkt hast, muss man die Wörter vllt. einfach kennen (Wortschatz und ggf. Sprachgefühl). Sie liest in der Tat sehr wenig. Da könnte man mal ansetzen. Andererseits chatten Menschen in dem Alter doch ununterbrochen. Das scheint in diesem Bereich aber keine Vorteile zu bringen. --87.172.217.205 14:37, 1. Jan. 2014 (CET)
Hast du die Chats schon mal gesehen? Die sind bestenfalls voll von Schreibfehlern und in vielen Fällen näher an lolcat als an Deutsch. --88.130.131.244 14:54, 1. Jan. 2014 (CET)
In "unserem" Chat wird nur in feinstem Hochdeutsch kommuniziert. ;-) Aber ja, ich kenne natürlich auch andere Chat-Verläufe. Vermutlich ist es also eher kontraproduktiv. -.- --87.172.217.205 15:00, 1. Jan. 2014 (CET)
Moin und Frohes Neues ... Zum Thema schreiben lernen siehe auch hier: Jürgen Reichen. Das deckt sich nicht mit der Ansicht von91.50.34.44, genau die Probleme, die 87.172.217.205 anspricht, entstehen bei Anwendung dieser Methode. Hilfreich ist, viel zu lesen (allerdings weniger Chats u. ä., die Wikipedia-Artikel sind meist zu empfehlen) und im angesprochenen Fall gemeinsam akustisch vernehmbar (ich vermeide den Begriff »laut«) zu lesen. LG --Gwexter (Diskussion) 15:32, 1. Jan. 2014 (CET)
Lernen muss vor allem Spaß machen. Oder der Leidensdruck muss groß genug sein (berufliche Sackgasse). Wenn es um's Lesen geht: Auch daran muss man Freude haben, Tageszeitungen sind für junge Menschen oft wenig interessant. Vielleicht besser eine (Frauen)Zeitschrift oder einen Roman vorschlagen. Wikipedia-Artikel sind oft leider in Fachchinesisch geschrieben. --79.204.198.99 15:36, 1. Jan. 2014 (CET)
Naja, Zeitungs- bzw. Zeitschriftendeutsch deutsch ist auch nicht mehr das, was es mal war, die Schreiber verlassen sich eher auf automatische Korrekturhilfen, Schriftsetzer als Rechtschreibfanatiker gibt es kaum noch, auch die Zeiten des Metteurs, der häufig zwecks letzter Korrektur las, sind vorbei; zu heutigen Massenwaren-Romanen habe ich keine Beziehung. Wenn ich an WP denke, dann eher an Artikel, in denen weniger spezifische Themen aus den verschiedenen Wissenschaften verständlich erklärt werden sollen (sic!). Nehmt euch einfach eine Städtebeschreibung o. ä. vor. --Gwexter (Diskussion) 15:50, 1. Jan. 2014 (CET)
Gut, mehr Lesen ist sicher sinnvoll. Ihr meint also, dass das Gehirn dann automatisch die Regeln generiert, also das, was wir Sprachgefühl nennen, oder lernt sie die Wörter dadurch "nur" auswendig, ohne logische Verbindungen zu anderen Wörtern herzustellen?
Ich frage, weil Sprachen doch einer gewissen Logik folgen müssen, sonst könnten (Klein-)Kinder keine Sprachen lernen. Uns Erwachsenen fallen bestimmte Muster ja mitunter gar nicht mehr auf: Wörter mit Str- bspw. beschreiben zumeist schmale, linienförmige Dinge, wie Straße, Strecke, Strich, Strand, Stringenz oder Stroh. Wörter mit Kn- häufig kleine, runde Dinge, wie Knoten, Knolle, Knopf usw. Aus meiner Sicht erkennt das Gehirn solche Muster in der Sprache. Sind aber nur Vermutungen, kein gesichertes Wissen. Sie zum Lesen zu animieren, schaffe ich sicher, aber ich dachte eben auch, dass man diese Dinge nebenbei gezielt üben kann. Möchte das mit ihr zusammen machen. Die emotionale Verknüpfung spielt ja auch eine Rolle beim Lernen, und da erreiche ich sie... --87.172.217.205 16:22, 1. Jan. 2014 (CET)
Nein, einer solchen Logik folgen Sprachen nicht; aus einer synchronischen Perspektive erscheinen die meisten Wörter und Schreibungen arbiträr, d.h. wenn man kein studierter Indogermanist ist, ist es nicht sinnvoll, über Eselsbrücken hinaus nach Regeln zu suchen. Es gibt zwar die Rechtschreibregeln, aber wenn man ganz ehrlich ist, scheinen die ein ziemlicher Betrug zu sein: man kann die Schreibung kaum eines Wortes allein auf die Rechtschreibregeln zurückführen, und es gibt aus der Sicht der Regeln extrem viele Ausnahmen. Das liegt daran, daß die Schreibung der Wörter nicht etwa den Rechtschreibregeln folgt, sondern die Schreibung war schon vorher da und unterlag bis zu einem bestimmten Zeitpunkt der Tradition. Die Regeln wurden sekundär "erfunden", um die Schreibung in eine lernbare Ordnung zu bringen; das konnte naturgemäß nur teilweise gelingen, denn Tradition neigt zur Entropie. Auch auf die Lautung kann man sich nicht verlassen, wie Dein Beispiel AubergineOberschiene schön illustriert; selbst in Sprachen mit einer extrem phonetischen Orthographie wie dem Tschechischen und dem Litauischen können sich die Schreiber nicht auf die Aussprache der Wörter verlassen, sondern müssen zuweilen der Tradition Rechnung tragen, zum Beispiel um homophone Wörter in der Schreibung zu unterscheiden; um so mehr trifft das natürlich aufs Deutsche mit seiner langen und abwechslungsreichen Sprachgeschichte und seiner dialektalen Vielfalt zu. Man kann also festhalten, daß die Schreibung weder den Rechtschreibregeln noch der Aussprache unbedingt folgt.
Das ist aber nicht weiter schlimm, weil ein gesunder, geübter Leser und Schreiber von Natur aus weder lautiert, also die Wörter nach ihrer Lautgestalt niederschreibt, noch bei jedem Wort, das er schreibt, die einschlägigen Rechtschreibregeln im Kopf hat. Vielmehr erkennt ein geübter Leser die Wörter und Wortgruppen mit einem Blick als sogenannte Wortbilder, so wie man die Gesichter von Menschen leicht auf einen Blick auseinanderhalten kann, ohne sich bewußt zu sein oder auch nur annähernd mit Worten beschreiben zu können, wie das jeweilige Antlitz aussieht. Das Schreiben geht einem also normalerweise in Fleisch und Blut über -- man kann es, wenn man es gelernt hat, automatisch, wie das Treppensteigen und Fahrradfahren. Zugleich ist es aber kein Anwenden von Regeln, sondern zum allergrößten Teil das Wiedergeben von Auswendiggelerntem: Ja, wir lernen tatsächlich die Wörter, die wir lesen, auswendig -- allerdings unbewußt. Deshalb war der Rat der Kollegen oben, viel zu lesen, genau der richtige: wer viel liest, schult sein Gedächtnis für die „Gesichter der Wörter“: je öfter er ein Wort gelesen hat, desto besser kann er es schreiben, und je mehr Übung und Vertrautheit er im Lesen erwirbt, desto leichter und einfacher merkt er sich die Schreibung eines Wortes, selbst wenn er es nur einmal gelesen hat. --Hermine Tuzzi (Diskussion) 17:16, 1. Jan. 2014 (CET)
Weiters folgt ein Zuhörer ja dem Kontext des Gehörten und hat laufend eine Vorstellung, was jetzt kommen kann. Das geht ja bekanntermaßen soweit, dass fehlende Worte im Hirn ergänzt werden und der Hörer trotzdem Stein und Bein schwört, die fehlenden Worte tatsächlich gehört zu haben (da gibt es Versuche). Beispiel: ob "das" oder "dass" kommt, hört man nicht, sondern weiss es vorher. --RobTorgel (Diskussion) 17:26, 1. Jan. 2014 (CET)

Latein: Cicero, De officiis; Nebensatz

Hallo! Nun komme ich schon wieder mit einer Frage zur Übersetzung von Ciceros De officiis! In Buch I, 71 steht folgender Satz:

…, qui … a re publica recesserunt, cum eius administrandae potestatem aliis laudemque concederent.

In Bezugnahme auf diese Seite lautet meine Übersetzung:

„… die sich … aus dem Staat zurückzogen, wobei sie die Gelegenheit und das Verdienst, diesen zu verwalten, anderen überließen.“

Es geht um den Nebensatz, der durch cum (in der Übersetzung: „wobei“) eingeleitet wird: Im Deutschen gilt „wobei“ in diesem Zusammenhang laut DUDEN als weiterführendes Adverb. Aber welche Funktion hat das cum im Original und um welchen Nebensatz handelt es sich in der Folge? Gibt es vielleicht auch eine bessere, grammatikalisch entsprechende Übersetzung? Schöne Neujahrsgrüße, XanonymusX (Diskussion) 14:44, 1. Jan. 2014 (CET)

"Cum" mit Konjunktiv drückt im weitesten Sinne aus, dass es neben dem zeitlichen auch einen inhaltlichen Zusammenhang zwischen zwei Handlungen gibt, das reicht von kausal bis konzessiv. Ich würde es einen Adversativsatz nennen. Mit "während" (bei Gleichzeitigkeit) machst du nichts falsch, da ja auch die deutsche Konjunktion mehrdeutig ist. Grüße Dumbox (Diskussion) 15:13, 1. Jan. 2014 (CET)
Danke, so kann ich es nachvollziehen. Allerdings wurde mir gegenüber behauptet, es handle sich um einen Konzessivsatz, was ich absolut nicht nachvollziehen kann – wäre das aber eine valide Möglichkeit?--XanonymusX (Diskussion) 13:00, 2. Jan. 2014 (CET)

Kryptographie

Ich habe auf Wikipedia das interessante Thema Zahlensender entdeckt, weiß aber nicht, ob ich die Sache richtig verstanden habe. Es wird da ja anscheinend die Möglichkeit der One-Time-Pads zur Verschlüsselung genutzt; allerdings liest man ab und zu, daß diese Möglichkeit zwar sicher, aber auch sehr aufwendig bis unpraktikabel sei, weil man als Schlüssel Kolonnen von echten Zufallsbuchstabenreihen braucht, die der Empfänger mit sich führen und aufbewahren muß, um überhaupt Nachrichten empfangen zu können.

Nun habe ich mir überlegt, daß es doch viel einfacher wäre, wenn man zur Verschlüsselung eine Reihe von scheinbar zufälligen Büchern verabredete, die der Empfänger zur Hand hat: dann nähme man die Buchstaben an den Rändern einer Spalte anstelle der Zufallsbuchstaben als Schlüssel; man müßte dem Empfänger nur verschlüsselt vermitteln, auf welcher Seite und Spalte, in welcher Zeile, an welchem Rand und in welcher Richtung er seinen Schlüssel abzulesen hätte. Er hätte dann keine verdächtigen Zettel zu verstecken und könnte, wenn es sicher ist, die verschlüsselten Nachrichten aufbewahren, um sie bei Bedarf wieder zu entschlüsseln. Die Menge der zur Verfügung stehenden Schlüssel wäre sicherlich auch größer, als wenn man ihm einen Zettel mit Buchstabenreihen überließe, so daß man länger ohne direkte Kontaktaufnahme auskäme; außerdem kann der Sender ad hoc den Schlüssel aus einer großen Menge an Kode auswählen; fällt dagegen dem ausspionierten Land ein Zettel mit One-Time-Pads in die Hände, kann man mit deren Hilfe gewiß recht schnell alle Nachrichten, die der Agent empfangen hat, entschlüsseln.

Selbst wenn der Empfänger aufflöge und man herausfände, daß er die Büchermethode nutzt, um seine Nachrichten zu entschlüsseln, könnte sich die Polizei des ausspionierten Staates immer noch dumm und dämlich suchen, bis sie die entsprechenden Stellen in den entsprechenden Büchern zu den entsprechenden Nachrichten gefunden hätte, wenn dem Empfänger nur eine genügend große Bibliothek zur Verfügung stünde, in der die Schlüsselbücher unauffindbar einsortiert wären. Ich würde sogar meinen, der Aufwand wäre so groß, daß es überhaupt nicht möglich wäre.

Oder renne ich gerade in eine ganz falsche Richtung, und das, was ich mir gedacht habe, ist überhaupt nicht sicher? Wo liegt mein Denkfehler? --77.186.50.15 16:52, 1. Jan. 2014 (CET)

Für mich hört sich das nicht unvernünftig an. Zur Situation, wenn sie ihn haben, gibt's allerdings den passenden xkcd... --Eike (Diskussion) 17:12, 1. Jan. 2014 (CET)
Die Antwort steht im Abschnitt One-Time-Pad#Kryptologisch unsichere Schlüssel. --Rôtkæppchen68 17:29, 1. Jan. 2014 (CET)
Den Abschnitt hatte ich natürlich auch gelesen -- aber trifft er hier zu? Die Ränder einer Spalte, senkrecht gelesen, ergeben ja keinen sinnvollen Text, sondern eine zufällige Reihe von Buchstaben; auch wenn die Häufigkeit, mit der Buchstaben am Anfang oder Ende von Silben auftreten, natürlich nicht gleichmäßig ist wie bei einer idealen Zufallsreihe, so ist die Reihenfolge, in der die Buchstaben senkrecht am Spaltenrand auftreten, doch völlig zufällig -- wie kann man einen so verschlüsselten Text dekodieren? --77.186.50.15 19:04, 1. Jan. 2014 (CET)
z. B. könnte der Bedarfsträger Zugang zu allen veröffentlichten Werken haben und schon kann man rumprobieren (das rumprobieren geht besonders einfach, wenn man den Klartext kennt (z. B. weil es sich um Nachrichten in einem bestimmten Format handelt: „Nachricht Nr. X Von Y An Z Datum A Ort B...”...)... --Heimschützenzentrum (?) 19:20, 1. Jan. 2014 (CET)
@77.186.50.15, Klartext eines Buches ist schon deswegen kryptographisch unsicher, weil die einzelnen Buchstaben ungleich verteilt sind. In der deutschen Schriftsprache sind beispielsweise E und N viel häufiger als Q und X. In einem ausgewürfelten echten Zufallstext kommt jeder Buchstabe statistisch gleich häufig vor, was einen damit verschlüsselten Text schwerer zu knacken macht. Anhand der Buchstabenhäufigkeit im verschlüsselten Text kann der Codeknacker Rückschlüsse auf die Art des Schlüssels ziehen. In einem mit einem deutschen Text (oder den Anfangsbuchstaben eines deutschen Textes) verschlüsselten deutschen Text werden beispielsweise die Buchstaben J (=E+E), B (=N+N) und S (=E+N) statistisch gesehen häufiger auftauchen als in einem mit Zufallstext verschlüsselten Text. --Rôtkæppchen68 19:39, 1. Jan. 2014 (CET)
gab's für sowas nich ne Möglichkeit doch noch eine Gleichverteilung zu erreichen? also wenn man weiß, dass bei einem Butterbrot „Butter” mit 0,8 kommt und „ohne Butter” mit 0,2, dann ist „Butter+Butter” schonmal näher an der Gleichverteilung (nämlich 0,64)... oda? --Heimschützenzentrum (?) 20:15, 1. Jan. 2014 (CET)
Hmm, verstehe! Wie gut sind denn die Geheimdienste informiert? Ich hätte hier zum Beispiel auch einige Bücher auf armenisch, ge'ez und altgriechisch stehen -- oder bekommen die auch das heraus? Wie auch immer, ich mache es halt einfach so, wie der VEVAK es mir sagt, und behalte meine Verbesserungsvorschläge für mich. ;-) --77.185.207.214 09:39, 2. Jan. 2014 (CET)
Es kommt natürlich immer auf die Art des Buches an, wie zufällig der Inhalt bzw. bestimmte Teile des Inhalts sind. Die Gruppe Sorge verwendete ein statistisches Jahrbuch, und ähnliche Bücher sollten auch heute noch sicher sein, wenn man die letzten Ziffern verwendet (nicht die ersten: Benfordsches Gesetz). Geeignet sind auch Telefonbücher (auch wieder nur die letzten Ziffern, die ersten kodieren meist einen Stadtteil, und wenn man beim Namen gerade einen vietnamesischen Namen erwischt hat, und die alle im Vietnamesen-Viertel wohnen, sind die Ziffern wieder nicht unabhängig). Anderseits muss man aber auch beachten: Wenn bei einem Spion ein obskures Telefonbuch gefunden wird, liegt der Verdacht nahe, dass es zur Verschlüsselung verwendet wird.
Bücher mit nicht-zufälligen Texten dagegen liefern immer Hinweise. Wenn etwa im verschlüsselten Text X der häufigste Buchstabe ist, dann liegt es nahe, dass ein Klartext-E verschlüsselt mit dem Buchstaben, der im Schlüsselbuch am häufigsten vorkommt, eben jenes X ergibt, völlig unabhängig davon, welch obskure Sprache dein Schlüsselbuch hat. Mit ein bisschen Rumprobieren müsste es möglich sein, die Häufigkeitsverteilung der Sprache des Schlüsselbuches herauszubekommen (ich sehe vor meinem geistigen Auge irgendein großes lineares Gleichungssystem) und daraus die Sprache. Das wäre dann der Zeitpunkt, wo die Kryptologie verlassen wird und der Geheimdienst mal bei dir vorbeischaut, was für Bücher in du in deinem Bücherregal stehen hast. --132.230.221.144 10:40, 2. Jan. 2014 (CET)

Farbwidergabeindex handelsübliche LED Lampen als Glühbirnenersatz

Die Energiesparlampen waren in Sachen Farbwidergabe ja nicht so der hit, heisst, der Glühbirne weit unterlegen. Ist das jetzt mit den LED Beleuchtungen besser geworden? --93.132.21.68 00:12, 2. Jan. 2014 (CET)

LED-Lampen haben je nach Type einen Farbwiedergabeindex Ra von 70 bis 90. Billige Energiesparlampen haben einen Ra von 60. Normale Energiespar- und Leuchtstofflampen haben einen Ra von 80. Teure Sonderleuchtstofflampen haben einen Ra von 90. Bei den meisten Herstellern wurden Leuchtstofflampen mit Ra=70 aus dem Sortiment genommen. --Rôtkæppchen68 00:18, 2. Jan. 2014 (CET)
Ich hatte bisher Osram und Philips. Also kann ich mich kaum verbessern durch Umstellung auf LED. Schade. Btw, was ist eine teure "Sonderleuchtstofflampe"? Mir ist so etwas noch nie begegnet. --93.132.21.68 00:23, 2. Jan. 2014 (CET)
Bei Osram sind das z.B. die hier, erkennbar an der 9 an erster Stelle der Phosphorbezeichnung, siehe auch Leuchtstofflampe#Lichtfarbe. --Rôtkæppchen68 00:36, 2. Jan. 2014 (CET)

libysche Nachkriegsregierung

Hallo! Kriegt jemand raus, wer 2012 der "post-war Libyan government's top religious official" (meine Übersetzung: oberster Religionsbeamter der libyschen Nachkriegsregierung) war/ist? Vielen Dank, -- Doc Taxon @ DiscI ♥ BIBR14:45, 2. Jan. 2014 (CET)

Hamza Abu Faris (gem. fr:Gouvernement Abdel Rahim al-Kib). --= (Diskussion) 15:04, 2. Jan. 2014 (CET)
wow! Das ging ja fix, danke sehr, -- Doc Taxon @ DiscI ♥ BIBR15:07, 2. Jan. 2014 (CET)

Preisangabe Sondernummer 032

Moin, besteht für 032-Rufnummern eine Preisangabepflicht nach § 66a TKD? Ich möchte eine solche Nummer auf einer Webseite veröffentlichen. (Mir ist klar, dass Wikipedia keine Rechtsauskunft ist, aber vielleicht kann doch jmd. helfen.) Gruß --Stefan1973HB Disk. 00:13, 4. Jan. 2014 (CET)

Siehe Vorwahl 032 (Deutschland)#Nachteile. --Rôtkæppchen68 01:09, 4. Jan. 2014 (CET)
Soweit klar. Müsste aber vielleicht dennoch einen Hinweis folgender Art anbringen: "Zu den Preisen für 032-Rufnummern schauen Sie in den Tariflisten Ihres Telefonanbieters." --Stefan1973HB Disk. 01:19, 4. Jan. 2014 (CET)
Nein, bei normalen Festnetznummern musst Du das ja auch nicht. 032-Nummern sind weder Mehrwert- noch Sondernummern, sondern Festnetznummern ohne geographische Zuordnung. --Rôtkæppchen68 07:06, 4. Jan. 2014 (CET)
Spontan hätte ich Rotkäppchen eigentlich zugestimmt, aber auch wenn man außen vor lässt, dass diese Nummern nicht geographisch zugeordnet sind, dann sind es dennoch keine normalen Festnetzrufnummern. Dass man den Ort des Angerufenen nicht aus der Vorwahl entnehmen kann, ist denke ich gar nicht das Problem (wen interessiert der schon?); der Punkt ist ein anderer:
Während vom Festnetz die Kosten für Anrufe auf diese Nummern idR genau dieselben sind wie für andere Festnetznummern auch, gilt das für Mobilfunkanrufe nicht. Für Anrufe aus dem Mobilnetz sind die Kosten nämlich andere, oft deutlich höhere. Und das ist auf die Schnelle auch nicht ersichtlich; die Nummer sieht ja aus, als wäre sie eine ganz normale Festnetznummer mit ganz normalen Festnetzkosten. Ist sie aber nicht. Das ist genau die Art von Fällen, in denen das TKD eine Preisangabe fordert, um eine Täuschung des Anrufers zu unterbinden. Insofern würde ich, wenn ich es wäre, einen Hinweis in der Art "Die Kosten ergeben sich aus den Vereinbarungen mit Ihrem Telekommunikationsanbieter." dazuschreiben. --88.130.114.151 15:01, 4. Jan. 2014 (CET)
Auch wenn ich es wahrscheinlich nicht muss, habe mich entschlossen eine Fußnote anzubringen. Nach den Ausführungen der IP 88.130.114.151 ist dies nur fair gegenüber den Lesern der Webseite. --Stefan1973HB Disk. 22:21, 4. Jan. 2014 (CET)
Wie gesagt, ich halte die Interessenlage in den in § 66a TKG genannten Fällen schon für mit deinem Fall vergleichbar; insofern kann es sein, dass du vielleicht sogar doch auf die Kosten hinweisen musst. Nur: Sicher weiß ich das nicht. Und genau wie du schon sagst: Da die Kosten doch höher sein können als erwartet, ist es nur fair, das dazu zu schreiben. Und - abgesehen davon, dass es fair ist - bist du damit dann auch definitiv auf der sicheren Seite. --88.130.114.151 23:04, 4. Jan. 2014 (CET)
Archivierung dieses Abschnittes wurde gewünscht von: Stefan1973HB Disk. 22:21, 4. Jan. 2014 (CET)

Doppelter Buchtitel: Doppeltitel, mit Untertitel bzw. Nebentitel oder ganz anders?

Gibt es einen Fachbegriff für Roman- bzw. Buchtitel, die aus zwei Titeln und dem Wörtchen „oder“ zusammengesetzt sind? Zwei Beispiele:

--84.178.13.228 13:50, 4. Jan. 2014 (CET)

Buchtitel => "Ein Buchtitel bezeichnet den Werktitel eines Buches, unter dem es erscheint (publiziert wird) und in Titellisten geführt wird. Oft werden dem Haupttitel auch ergänzende Angaben in einem Untertitel bzw. Nebentitel hinzugefügt." GEEZER… nil nisi bene 13:53, 4. Jan. 2014 (CET)
Es ist nicht Alternativtitel. Das ist, wenn es auch unter einem separat geführten anderen Titel bekannt ist. GEEZER… nil nisi bene 14:02, 4. Jan. 2014 (CET)
Unter Buchtitel hatte ich hier in WP schon nachgeschaut und die von Dir zitierte Erklärung auch gelesen. Wirklich eindeutig finde ich sie aber nicht, deswegen meine Frage hier. Dass man Alternativtitel ausschließen kann hilft mir ein bisschen weiter. --84.178.13.228 14:19, 4. Jan. 2014 (CET)
Wir machen hier auch lange Wege für interessante Fragen. Ein WP-relevanter Literaturwissenschaftler wurde angeschrieben und antwortete darauf:
"Der literaturwissenschaftliche Fachausdruck dafür heißt...... ? DOPPELTITEL. Kein Sch*** ! Steht so in meinen Fachwörterbüchern, bes. beliebt im Barock. Häufig gibt der 1. Titel den Stoff, der 2. den Ideengehalt wieder."
Mal mit => Barock Doppeltitel <= suchen, da kommt einiges. Ob das nun auch noch heute auf deine Frage zutrifft, wirst du nach dem Lesen entscheiden. :-) GEEZER… nil nisi bene 18:04, 4. Jan. 2014 (CET)
Archivierung dieses Abschnittes wurde gewünscht von: 84.178.24.65 22:13, 4. Jan. 2014 (CET) Vielen Dank an GEEZER… nil nisi bene und den unbekannten LiWi für die Unterstützung!

Warum muss man bei downladprotalen mehrere Seiten durchklicken...

... bis endlich mal der download startet? Kann man da nicht auf der ersten Seite einen Downloadlink einbauen, der gleich das Paket downloaded? Selbst bei seriösen Portalen wie Chip ist das so-

--87.157.35.69 12:12, 1. Jan. 2014 (CET)

"Kostenlos" heißt eben, dass man anders bezahlt: Werbung, Angebote von kostenpflichtigen "Premium-Downloads" etc. There's no such thing as a free lunch. Grüße Dumbox (Diskussion) 12:21, 1. Jan. 2014 (CET)
Manchmal ist es auch harmlos: Da liegt z. B. ein und dasselbe Paket auf den verschiedensten Servern, und der Benutzer kann sich dann durch zusätzlichen Klick denjenigen aussuchen, dem er am meisten vertraut, der seiner Erfahrung nach der schnellste (der nächstgelegene) ist usw. --PeterFrankfurt (Diskussion) 03:59, 2. Jan. 2014 (CET)
Man kann. " direct download " --217.84.97.199 17:24, 2. Jan. 2014 (CET)

Deutsche Bahn...

Liebe Leute, ich weiß nicht, wie es euch damit geht, aber mich stören die horrenden Preise der Deutschen Bahn; dreistellige Beträge für eine "einfache" Fahrt durch das Bundesgebiet, das sogar noch im Sparpreis! Was ich suche, sind legale Spartricks; ich meine bspw. von jemandem gehört zu haben, dass man den Fahrpreis gut reduzieren kann, wenn man die Fahrt im Ausland beginnt. Ich (und sicherlich auch einige andere unter uns) wären sehr dankbar für ein entsprechendes Informationssharing. Frohes Neues und viele Grüße, --85.178.55.18 14:11, 1. Jan. 2014 (CET)

Das kannst Du ganz einfach selbst herausfinden. Du kannst Dir einfach auf den Websites der diversen Eisenbahngesellschaften dieselbe Strecke anbieten lassen. Die Reise Stuttgart–Berlin kostet bei der DBAG beispielsweise 142 Euro. Wenn man bei der SBB aber Basel SBB–Berlin über Stuttgart bucht, kostet das nur 135 CHF, das sind ca. 110 Euro. --Rôtkæppchen68 14:22, 1. Jan. 2014 (CET)
Seit einiger Zeit kannst Du ach den Bus nehmen. --Rubblesby (Diskussion) 14:25, 1. Jan. 2014 (CET)
…oder das Flugzeug. --Rôtkæppchen68 14:30, 1. Jan. 2014 (CET)
Quer-Durchs-Land-Ticket und Schönes-Wochenende-Tickett sollten noch unter 50€ sein, man kann sogar günstig Mitfahrer mitnehmen. --88.130.131.244 14:31, 1. Jan. 2014 (CET)
Mich stören die Bahnpreise nicht. Bahncard 25, Fernverkehr früh buchen, im Regionalverkehr stets den jeweiligen Verkehrsverbund für lau obendrauf - find' ich gut & günstig. Gr., redNoise (Diskussion) 14:32, 1. Jan. 2014 (CET)
Es gibt auch noch die indische Sparmethode. --Fqugdvin (Diskussion) 15:24, 1. Jan. 2014 (CET)
Wenn man mit dem Quer-Durchs-Land-Ticket aber wirklich quer durch's Land will, ist man einen ganzen Tag unterwegs (Stuttgart -> Berlin 11h mit RB/RE statt 6h mit EC/ICE). Und das ist noch nicht die längste innerdeutsche Fahrstrecke. --mfb (Diskussion) 15:27, 1. Jan. 2014 (CET)
Die Bahn selbst mit IC ist günstiger als das Auto, zumindest wenn man alleine fährt und die Bahn ist schneller wenn es von City zu City geht. Das Preis-/Leistungsverhältnis ist immer noch konkurrenzfähig. Das mit den Verspätungen wird sich nie vermeiden lassen, haben wir bei anderen Verkehrsmitteln ebenso. Amerikanische Geschäftsfreunde sind immer wieder begeistert, zumal der Kurzstrecken-Luftverkehr ordinär geworden ist.--79.232.200.63 16:11, 1. Jan. 2014 (CET)
Teuer??? Naja, so ein Lok fährt auch nicht mit Luft und Liebe... da geht ne Menge an inzwischen sauteurer Energie drauf - je schneller man fährt desto mehr. Dazu kommt noch die inzwischen aufwändige und dadurch sauteure Technik... da verdienen ne Menge an großen Konzernen, Programmierern, usw. mit. Außerdem das Risiko, wenn wieder einmal etwas schief geht und die Leute entschädigt, umsorgt,... etc. werden müssen. So ein gesetzlich auferlegtes Schadensersatz-Sorglos-Packet hat eben auch seinen Preis. Und zu guter Letzt, der ganze Verwaltungswasserkopf mit den ganzen Managern will auch finanziert werden... gut den mag es so ähnlich bei der DB auch gegeben haben und auch wenn die Defizite vom Staat aufgefangen wurden hatte der Laden damals für die Gesellschaft den Vorteil, dass so mancher der heute als unbrauchbarer Hartz-4-Empfänger rumhängt einen Job hatte, ob als Kofferträger, Fahrkartenverkäufer oder als Hilfskraft in einem der AWs. --Btr 18:59, 1. Jan. 2014 (CET) PS:Die Bahn selbst mit IC ist günstiger als das Auto,... das will ich bezweifeln und kommt mit Sicherheit aufs Auto an. Ich hatte erst letztens den Fall, 950 km an einem Tag mitm PKW... das waren gerade einmal knapp 60 Liter Diesel und ich hab kein Taxi, Bus etc. (=Zusatzkosten) gebraucht um von irgendwelchen Bahnhöfen zum eigentlichen Ziel zu kommen. Außerdem kein Schleppen von irgendwelchen Gepäckstücken und vollkommene Unabhängigkeit...
Soso ... siehe Milchmädchenrechnung --84.178.21.55 19:09, 1. Jan. 2014 (CET)
Wenn Du 950 Kilometer mit dem Auto durch Deutschland fährst, kostet das nicht nur 60 Liter Diesel. Du darfst da gerne anteilig noch die Kosten für Anschaffung und Wartung des Autos dazuzählen, dazu kommen Versicherungen und Steuer. Dann musst Du Deine aufgewendete Arbeitszeit (Stundenlohn) rechnen. Im Auto hast Du als Fahrer keine Chance auf irgendeine produktive Tätigkeit oder Entspannung. Im Zug kannst Du schlafen, arbeiten, lesen, lecker essen, entspannen und erholen. Als Autofahrer musst Du das vor oder nach Deiner Fahrt tun, hast also zusätzlichen Zeitaufwand. --217.85.72.53 19:23, 1. Jan. 2014 (CET)
Die tatsächlichen Kosten pro Kilometer beim Auto liegen je nach Fahrzeug und Anschaffungspreis bei Mittelklassewagen zwischen 55 und 65 Cent. Quelle ADAC und Auto Bild.--79.232.200.63 21:01, 1. Jan. 2014 (CET)
Zum Anschaffungspreis: hängt davon ab, es gibt eigentlich keinen vernünfitgen Grund, warum man unbedingt einen Neuwagen anschaffen muss, ein älterer Gebrauchter, dessen Anschaffungspreis pro gefahrenen Kilometer im Vergleich zu den übrigen Kosten kaum ins Gewicht fällt, tuts genauso. Der Verbrauch ist bei weitem nicht so stark gesunken, wie uns die Autoindustrie weismachen will. Wg. der Vesicherung: das trifft natürlich zu, wenn man sonst kein Auto hätte, wenn man aber ohnehin ein Auto braucht oder haben will ist die Höhe der Versichrung ja unabhängig davon, wie viel man fahrt (alerdings gibt es shcon noch Kosten außer dem Sprit, die von der gefahrenen Distanz abhängen: Wartungs- und Reparaturkosten). Und zum Thema verlorene Zeit: das ist Ansichtssache: man kann im Zug mehr machen als im Auto, aber bei weitem nicht so viel wie z.B. zuhause, außerdem gibt es auch Leute, die beim Zugfahren weniger Freude haben als beim Autofahren, weil sie z.B. nicht gerne mit relativ vielen Leuten lange auf relativ kleinem Raum sind. Und zum Stundenohn: so kann man eig. nur rechnen, wenn man reich ist, die meisten Leute haben ein begrenztes Budget, daher wenn sie fürs Zugfahren mehr ausgeben, als ihnen die Autofahrt kostet, dann müssen sie sich irgendwo anders einschräken oder mpssen die Reise verkürzen oder können sie ev. garnicht antreten. außerdem ist autofahren für die meisten nicht wie Arbeit. Btw, ich bin kein Autofahrer, nur mag ich eben keine wenig sinnvollen Argumente. Es gibt durchaus auch sinnvolle Argumente fürs Zugfahren, die noch nicht gannnt wurden, z.B. der viel kleinere ökologische Fußabdruck und dass die durchschnittlich pro Stunde Fahrt durch Zugverspätungen verlorene Zeit geringer ist als die durchschnittlich pro Ttunde durch Staus verlorene Zeit. --MrBurns (Diskussion) 21:07, 1. Jan. 2014 (CET)
(BK) zu ADAC: die rechnen soviel ich weiß nur mit Neupreisen (möglicherweise ist das auch politisch motiviert, um zu betonen, wie viel die "armen" Autofahrer schon zahlen müssen), mittlerweile werden in Westeuropa aber deutlich mehr Gebrauchtwagen als Neuwagen gekauft.--MrBurns (Diskussion) 21:07, 1. Jan. 2014 (CET)
Auch hier gilt Milchmädchenrechnung.--79.232.200.63 21:14, 1. Jan. 2014 (CET)
Kannst du das mit "Auch hier gilt Milchmädchenrechnung" auch begründen? Falls du dich auf den Anschaffungspreis beziehst: wenn ich einen Gebrauchtwagen kaufen würde, wäre es für mich völlig irrelevant, wie hoch der Neupreis war... --MrBurns (Diskussion) 21:19, 1. Jan. 2014 (CET)
(BK) Anschaffungskosten und Unterhalt interessieren mich nicht, da ich das Auto sowieso brauche, wenn ich damit fahre ists sogar günstiger, weil ich diese Kosten auf mehr Kilometer verteilen kann. Verschleiß ist in meinem Fall ebenfalls nicht erwähnenswert, da die Wartungsarbeiten selbst in eigener Werkstatt erledigt werden und dadurch nur Materialkosten anfallen. Und um dem gleich zu entgegnen... Mehrkilometer interessieren mich auch nicht, da ich meine Autos bezahlt sind und i.d.Regel erst dann abgestoßen werden, wenn die gefahrenen Kilometer nicht mehr wirklich interessieren - ob da am Ende 250, 300, oder 400.000km steht ist egal.
Wenn ich unterwegs bin, dann geht das spontan, da habe ich keine Zeit irgendwelche Bahnfahrpläne zu studieren bzw. mich durch das Abrechnungs-/ Rabattsystem der Bahn zu kämpfen. Beim Auto steige ich bei A ein und wenn ich will gehts Non-Stop (bis zu 1000km) bis nach B durch und das zu jeder Tag- und Nachtzeit. Außerdem habe ich keine Lust mich in irgendwelche verranzten, dreckigen oberflächlich durchgewischten Zugabteile zu setzen, wo ein paar Stunden vorher vielleicht Hinz und Kunz was weiß ich getrieben haben. Dazu kommt noch der ganze Umsteigezirkus inkl. Gepäckschleppen... nö, wirklich nicht... solls von mir aus sogar einiges mehr kosten... bevor ich mich anhand von Fahrplänen durch irgendwelche Linienbusse, Taxis, Bahn, etc. hangele, da steig ich lieber ins Auto. --Btr 21:21, 1. Jan. 2014 (CET) PS: Das lecker Essen soll bestimmt ein Scherz sein bei dem Billigmist, den die Bahn da so sauteuer in ihren Zügen vertickt... aber freut mich trotzdem, wenn es Leute gibt, die das so toll finden und nutzen. Nur frage mich nur warum sich unsere Herrn Politiker und auch die Oberen dieser tollen Bahn so selten -meist nur zu Presse Anlässen- in die Züge verirren und verirrt haben (So fährt Bahnchef Grube zur Arbeit (Bericht aus 2010)).
Wobei das Essen in der deustchen Bahn nach meinen Erfahrungen sowohl von der Qualität bessser als auch vom Preis her billiger ist als in den meisten deutschen Autobahnraststätten. Und im Zug hat die Küche wohl 24/7 geöffnet, an Raststätten nicht immer. Fahrpläne studieren muss man heute übrigens nicht mehr wirklich, man muss nurmehr im Internet Start- un Zielbahnhof eingeben und bekommt dann die Verbindungen angezeigt und mit einem weiteren Klick die Preise. Dauert insgesamt <5 Min., wenn man das Ticket gleich kauft ist der Zeitaufwand, der zurb Reise dazukommt <10 Min. (jeweils inkl. der Ausdrucke). --MrBurns (Diskussion) 21:30, 1. Jan. 2014 (CET)
Um Raststätten u.ä. mache ich normalerweise einen großen Bogen... entweder Selbstversorger oder was hindert einen daran -man is ja unabhängig- mal ein paar Kilometer Landstraße zu fahren um eine vernünftige (kleine) Gaststätte aufzusuchen. So läßt man auch Land und Leute mitkommen und donnert nicht nur mit Höchstgeschwindigkeit von Zentreum zu Zentrum... ich hab da irgendwie eine Abneigung nur an irgendwelche ausgetrettenen Touristen-/Reisendenpfade entlang geschoben zu werden. Zudem mag ich mich einfach nicht auf irgendwelche Sachen verlassen, die ich nicht selbst in der Hand habe.
Außerdem stören mich -das sind mit die gravierendsten Gründe- bei der Bahn AG ein paar Sachen die sagen wir politischer Natur sind... z.B. Hauptstrecken werden vermeintlich überoptimal bedient, während das flache Land abgehangen wird. Oder auch das ganze Hochgeschwindigkeits-Zeugs... bei den anderen -z.B. dem Auto- regt man sich auf und führt an, dass der Energieverbrauch quadratisch zur Geschwindigkeit steigt um dann selbst die Landschaft mit Hochgeschwindikeitstrassen zu zerschnippeln und mit jeder Zuggeneration einen neuen Geschwindigkeitsrekord aufzustellen. Von einer Bahn, wie sie mir sympatisch wäre, ist das weit entfernt... bei einer Deutschen Bundesbahn, wie es sie einmal gegeben hat würde ich vielleicht einsteigen, aber nicht bei diesem von geldgierigen Managern geführten Konzern. --Btr 22:07, 1. Jan. 2014 (CET)
Es war nett, Deine Vorurteile zu lesen. Wo sind bitte die Fakten? --Rôtkæppchen68 22:13, 1. Jan. 2014 (CET)
Vorurteile, aha und von diesem Irrsinn mit der ICE-Trasse drüben bei Nürnberg hab ich wohl immer nur geträumt. Während man bei uns hier seit Jahren schon nichts mehr von dieser tollen Bahn AG gesehen hat. Ohne die Vogtlandbahn wären die Lichter schon komplett aus... aber halt nö, stopp demnächst solls ja losgehen, die BIs sind ja schon fleissig am Wetzen der Messer, weil die tolle Bahn den ganzen Güterverkehrsdreck auf dieser für sie unrentablen Strecke durchschicken will. --Btr 22:26, 1. Jan. 2014 (CET)
Lass Dir Deine Posts hier mal von jemand anderes laut vorlesen. Vielleicht begreifst Du ja dann, was Du hier geschrieben hast. --Rôtkæppchen68 23:05, 1. Jan. 2014 (CET)
Obligatorischer XKCD: http://xkcd.com/481/ -- 88.67.153.104 18:44, 2. Jan. 2014 (CET)

Freundin am Flughafen treffen, die umsteigt (eilt)

Guten Abend,

kann ich am Frankfurter Flughafen eine Freundin treffen, die dort von einem Flug aus Israel in ein Flugzeug in die USA umsteigt???Im Internet finde ich dazu keine eindeutige Antwort. Danke für die Auskunft! --84.59.27.90 18:49, 1. Jan. 2014 (CET)

Nur wenn die Person den Transitbereich verlässt. Sie muss einreisen mit Pass etc. was die gesetzlichen Bestimmungen dazu sagen. Sie muss dazu auch wieder ausreisen, einchecken mit Kontrolle etc. und richtig heftig kann es mit dem Gepäck werden. ÜBERHAUPT NICHT GERNE gibt es herrenlose Gepäckstücke, oder Koffer, die ohne Besitzer in ein Flugzeug geladen werden. Das kann richtig Theater geben und dazu führen dass man nicht mitfliegen kann.--Giftzwerg 88 (Diskussion) 19:06, 1. Jan. 2014 (CET)
So lange sie wieder rechtzeitig in den Transitbereich hineingeht und puenktlich wieder ins Flugzeug einsteigt, wuerde ich denken, dass es irrelevant ist und dass sie sich in Frankfurt weder um ihr Gepaeck kuemmern noch neu einchecken muss, wenn sie in Israel schon die Bordkarte fuer den Flug in die USA erhalten hat und das Gepaeck durchgecheckt wurde. Sofern ich nicht ganz falsch liege, muss sie lediglich die Bedingungen fuer die Einreise nach Deutschland erfuellen. -- 160.62.10.13 02:40, 2. Jan. 2014 (CET)
Alternative, du buchst dir ein vollflexibles Flugticket in einen non-Schengen-Staat, bei einer Airline, die vom gleichen Terminal abfliegt. Damit hast du Zutritt zum Transitbereich und ihr könnt euch dort treffen. --MB-one (Diskussion) 05:00, 3. Jan. 2014 (CET)

was bedeutet „deterministisch” z. B. in One-Time-Pad#Kryptologisch_unsichere_Schl.C3.BCssel?

Hi! Ich stolper immer wieder über das Wort „deterministisch” in der Wikipedia (gerade eben hier: One-Time-Pad#Kryptologisch_unsichere_Schl.C3.BCssel) und bei den Juristen auch irgendwie (der Schuldvorwurf impliziert ja irgendwie, dass es auch anders hätte laufen können...)... Es ist doch eigentlich umstritten, ob es überhaupt nicht-deterministische Folgen geben kann... Soll das nur toll klingen? Oder gibt es eine Bedeutung des Wortes, die ich nich kenne? Soll es dort im Artikel vielleicht bedeuten, dass es bis zur letzten Zahl spannend bleibt, wenn man den zugrunde liegenden Prozess nicht kennt (z. B. könnte es ja doch ein Muster bei den Ergebnissen einer Würfel-werf-Maschine geben... so wie bei den Pseudozufallszahlengeneratoren...)? Thx. Bye. --Heimschützenzentrum (?) 19:06, 1. Jan. 2014 (CET)

Wenn, wie du schreibst, umstritten ist, ob es überhaupt nicht-deterministische Folgen geben kann, dann ist wohl nicht ausgeschlossen, dass es nicht-deterministische Folgen geben kann. Damit fällt IMHO deine Frage in sich zusammen. --Eike (Diskussion) 19:28, 1. Jan. 2014 (CET)
ähm? wir hatten uns damals schon nich verstanden, glaub ich... Ich versuch's nochmal: 1. Annahme: Die Welt ist deterministisch; Schlussfolgerung: Es gibt keine wie in One-Time-Pad#Kryptologisch_unsichere_Schl.C3.BCssel gefordert seienden Zufallszahlen... 2. Wie kann jetzt also der Artikel etwas fordern, was es vielleicht gar nicht gibt... wenn die Annahme unter 1. zuträfe, was keiner hier ausschließen kann, dann wäre ja der Artikel über etwas, das es nicht geben kann... aber: es gibt doch OTP in der Wirklichkeit... 3. es läuft also praktisch auf security by obscurity (also für heute lebende Menschen ist es unmöglich, diese Zahlen auf nicht trivialem Wege zu wissen...) hinaus, so dass diese philosophische Frage im Artikel belanglos ist, weswegen das Wort „deterministisch” gar nicht auftauchen sollte, weil wir ja nicht ins Blaue irgendwelche Thesen vermuten sollen (vgl. WP:TF)... --Heimschützenzentrum (?) 20:09, 1. Jan. 2014 (CET)
Ich versuch’s mal aus meiner Sicht zu erklären: Die Aussage, dass OTP perfekt sicher ist, ist eine mathematische Aussage die Folgen von Zufallsvariablen mit bestimmten mathematischen Einschaften (stochastisch unabhängig und gleichverteilt) voraussetzt. Ob bzw. bis zu welchen Grad solche Folgen in der Realität existieren, und wenn ja, wie man sie erzeugen kann, ist dann (für die mathematische Aussage!) unwichtig. -- HilberTraum (Diskussion) 21:28, 1. Jan. 2014 (CET)
ja eben: in meinen Mathematik-Vorlesungen kam das Wort „deterministisch” nie vor... ich denk immer es hackt, wenn ich das in Informatik-Artikeln lese (außer das mit dem „angelischen Nicht-Determinismus” in Theoretischer Informatik)... :) --Heimschützenzentrum (?) 22:24, 1. Jan. 2014 (CET)
(BK)Mir ist auch nicht ganz klar, was genau du meinst. Es wird allgemein unterschieden zwischen einem Determinismus in der Natur und einem Determinismus in der Informationstechnologie. Gibt es irgendwelche Inkonsistenzen und Erklärungsprobleme in der Wissenschaftsphilosophie, dann wird meist der Determinismus zuerst geopfert, um noch eine schlüssige Argumentation zu haben. Am besten du klickst dich zunächst durch die entsprechenden Artikel (die allerdings auch nicht sehr gut sind) und stellst deine Frage nochmal. mfg --84.172.151.246 21:34, 1. Jan. 2014 (CET)
Hier scheint esoterischer Schwurbel eine mathematisch / logische Aussage zu vernebeln. Wir haben auch Artikel über Gott, dessen / deren Existenz / Existenzen umstritten ist / sind. Wenn wir nur Artikel über "beweisbare" Realitäten hätten, dann würde meine Wikipedia aus einem einzigen Lemma nach Descartes bestehen. Und auch dann könnte ich mich irren, Hi hi hi :o) --Cookatoo.ergo.ZooM (Diskussion) 22:07, 1. Jan. 2014 (CET)
warum ist der Artikel „OTP” dann nich auch wie der Artikel „Gott” formuliert... stattdessen wird da sogar gleich ganz oben n real wirken sollendes OTP abgebildet... *lol* als würd man in den anderen Artikel ganz oben n Bild vom Weihnachtsmann bappen und „Das isser!” drunter schreiben... --Heimschützenzentrum (?) 22:33, 1. Jan. 2014 (CET)
1. ist es also ein Homonym (Teekesselchen)? und wenn ja: wozu? seit wann? und wer hat das eingeführt? fühlt sich für mich nich sehr wie aus ner Vorlesung an einer Universität an... mehr wie von ner HeizdeckenverkaufsveranstaltungSchulung mit Sonder-Certifikat von Mikroschrott am Ende... 2. was soll Determinismus in der IT genau sein? gibt's da auch n Link auf den entsprechenden Artikel? ich find nämlich nur Determinismus (Algorithmus), der etwas komisch bis gar nicht belegt ist... --Heimschützenzentrum (?) 22:24, 1. Jan. 2014 (CET)
Ich denke der (naturwissenschaftlich/mathematische) Determinismusbegriff ist schon immer in etwa der Gleiche. Es geht darum, ob das mathematische Modell, das einen Vorgang beschreibt, stochastisch ist oder nicht, im zweiten Fall nennt man es deterministisch. Zum Beispiel kann die Position eines Teilchens (deterministisch) durch eine Differentialgleichung modelliert werden oder (nicht deterministisch) als stochastischer Prozess (z.B. als Brownsche Bewegung). -- HilberTraum (Diskussion) 08:46, 2. Jan. 2014 (CET)
1. das liest sich in Determinismus#Physikalischer_Determinismus aber anders... :) erstmal geht's da gar nicht um Mathematik, sondern um Physik... zum anderen geht es da ganz eindeutig mehr um die philosophische Frage: „Ist es ein deterministisches System?”, während das OTP ganz ohne diese Frage auskommt (wenn ja: es gibt eh keinen, der es vorhersagen kann (und wenn doch, dann kann er sowieso gleich bitteschön den Klartext vorhersagen); wenn nein: es gibt eh keinen, der es vorhersagen kann (ohne wenn doch))... 2. an welcher Universität werden denn „nicht deterministisch” und „stochastisch” synonym verwendet? kommt mir echt total merkwürdig vor... --Heimschützenzentrum (?) 13:45, 2. Jan. 2014 (CET)
In 1) nennst du es noch Annahme, in 2) ist es noch ein vielleicht, aber in 3) tust du plötzlich so, als wäre es gesichert.
Du weißt, dass deine Annahme umstritten ist. Du weißt, dass man aus einer potentiell falschen Annahme wahre wie falsche Aussagen folgern kann (boolsche Logik). Damit wird IMHO eine Diskussion über die Folgerung hinfällig. Man müsste zuerst die Annahme sichern. Das wird uns hier wohl nicht gelingen.
Also... wenn deine Annahme richtig wäre, würde das zu Problemen führen, da hast du recht. Da wir es aber nicht wissen, führt sie zu gar nichts. Denk dir bei den Artikeln hinter "Determinismus" einfach immer ein "(falls Nichtdeterministismus existiert)".
--Eike (Diskussion) 09:21, 2. Jan. 2014 (CET)
1. das Wort „vielleicht” räumt eben die Möglichkeit ein... 2. in „3)” steht eben, dass es egal ist, ob 's deterministisch ist oder nich... s. o. (Antwort auf user:HilberTraum)... --Heimschützenzentrum (?) 13:45, 2. Jan. 2014 (CET)
Wenn es Nicht-Determinismus gibt - was wir beide nicht abschließend beurteilen können -, woher kommt dann der Schluss auf "Security by Obscurity"? Dieser Schluss ist nicht möglich, wenn Nicht-Determinismus existiert. Du machst hier deine ungewisse Annahme implizit zur Gewissheit, schließt daraus, es müsse sich um Security by Obscurity handeln, und ziehst daraus weitere Schlüsse. Aber wie gesagt: aus Ungewissem folgt Beliebiges, das kann dann wahr oder falsch sein.
In einem Satz, um Verhaspelungen möglichst schwierig zu machen:
Daraus, dass wir nicht wissen, ob Nicht-Determinismus existiert, folgt nicht, dass OTP Security by Obscurity wäre.
--Eike (Diskussion) 13:57, 2. Jan. 2014 (CET)
ich hab es wohl etwas überspitzt formuliert, aber es ist in der Tat in beiden Fällen (also: Det und nichDet) die Funktionsweise des OTP geheim, wenn man den Prozess der Zufallszahlerzeugung als Teil des OTP sieht... *kicher* --Heimschützenzentrum (?) 17:37, 2. Jan. 2014 (CET)
Schau dir den Artikel Perfekte Sicherheit an, das ist ein ganz formaler, mathematischer Begriff. Verschlüsselungsverfahren und Kryptoprotokolle werden auch als solche aufgefasst, was sie auch der formalen Verifizierung zugänglich macht, genauso wie Algorithmen. Philosophisch hinterfragen ließe sich wohl, was genau die Realisierung eines Algorithmus, eines Protokolls bedeutet etc.
Zum Thema Physik: Die ontologischen Begriffe von Determinismus spielen für die Physik meines Erachtens keine besondere Rolle. Die grundlegende Methodik sieht jedoch vor, dass Vorhersagen mit Wahrscheinlichkeiten versehen werden (ob nun tief innen in der Theorie Stochastik eine Rolle spielt, oder aber einfach die Fehler der Parameter berücksichtigt werden, was solls…), Messwerte mit Fehlern, und das Funktionieren der Theorien in den jeweiligen Situationen mit Wahrscheinlichkeiten bewertet wird. Aber wen interessiert dabei, was für ontologische Aussagen gemacht werden? --Chricho ¹ ² ³ 17:56, 2. Jan. 2014 (CET)
1. zum Artikel Perfekte Sicherheit Anschauen: hab ich schon... 2. das sag ich doch: was soll der Begriff „nicht-deterministisch” im OTP-Artikel? 3. warum versteht mich denn keiner? :) --Heimschützenzentrum (?) 18:26, 2. Jan. 2014 (CET)

Kopplung von Computer zu Handy

Es geht um folgendes Problem: Ich habe seit gut drei Monaten ein neues Handy, heute war mir ein bisschen langweilig und habe die Bilder von alten Handy auf das Neue übertragen. Dies sollte jedoch nur ein Zwischenschritt sein, denn der Hauptschritt sollte später danach sein, dass die Bilder auf dem Computer kommen.

Das Problem ist, dass die Kopplung von Computer auf das Handy aufgrund unterschiedlicher Kopplungscodes nicht möglich sind. Wären beide Kopplungscodes gleich, wäre es einfach ja, ich müsste auf den Hinweis des Computer nur auf das Ja klicken. Das ist jetzt dadurch nicht mehr möglich und bin jetzt sehr hilflos und brauche die Hilfe von anderen Handybegeisterten. Habt ihr hier eine Lösung? --Slmnjkl (Diskussion) 19:18, 1. Jan. 2014 (CET)

Wenn ich Kopplungscode lese, denke ich an Bluetooth. Verrate uns bitte, welches Mobiltelefon und welches PC-Betriebssystem Du nutzt, damit Dir geholfen werden kann. Wenn Du zwei Kopplungscodes hast, ist eines der beiden Geräte im falschen Kopplungsmodus. Eines der beiden Geräte musst Du auf sichtbar schalten, dann mit dem anderen Gerät entdecken und dann den Kopplungscode eingeben. Alternative: Nimm USB statt Bluetooth oder stöpsel die Speicherkarte des Mobiltelefons in den PC. --Rôtkæppchen68 19:29, 1. Jan. 2014 (CET)

Es geht um Bluetooth, dass ist richtig. Folgendes habe ich jetzt gesucht und gefunden:

  • PC: AMD A6-3410MX APU (Marke Samsung)
  • Handy: Samsung Galaxy S3

Hoffe es hilft was --Slmnjkl (Diskussion) 19:35, 1. Jan. 2014 (CET)

Der Betriebssystemtyp fehlt noch. Du kannst aber auch Samsung Kies auf Deinem Rechner installieren. Dieses Progi nimmt Dir die ganze Arbeit ab. Dazu gibt es eine Android-App fürs Samsung Galaxy, mit der Du den Knochen wahlweise über USB, WLAN oder Bluetooth mit dem PC verbindest. --Rôtkæppchen68 19:50, 1. Jan. 2014 (CET)
Ist das gratis oder kostenpflichtig? --Slmnjkl (Diskussion) 19:55, 1. Jan. 2014 (CET)
probiers einfach aus... *grins* sieht kostenfrei aus... --Heimschützenzentrum (?) 19:58, 1. Jan. 2014 (CET)
normalerweise ist so eine Synchronisierung Usus. Aber ich hätte da noch eine kostenfreie App: >> Tonido <<. Damit kann man seine eigene Cloud erstellen. --80.108.60.158 16:36, 3. Jan. 2014 (CET)

Unabhängigkeit Schaumburg -Lippe

Suche einen Wikipedia Artikel der Bestrebungen Schaumburg-Lippes ein eigenes Bundesland zu werden

--80.147.118.183 09:58, 2. Jan. 2014 (CET)

Schaumburg-Lippe#Seit dem Zweiten Weltkrieg, einen eigenen Artikel gibt es wohl nicht. --Mauerquadrant (Diskussion) 11:55, 2. Jan. 2014 (CET)
„Ein Volksentscheid vom 19. Januar 1975 zur Wiederherstellung des ehemaligen Landes (nach Art. 29 GG) wurde trotz positiven Ausgangs vom Bundesgesetzgeber zurückgewiesen.“ Dazu finde ich nichts in der angegebenen Quelle. Was soll das überhaupt heißen? Der Volksentscheid wurde durchgeführt und es gab eine Mehrheit für die Separatisten, wurde dann aber nicht umgesetzt? Bislang wusste ich nur von einem fehlgeschlagenen Volksentscheid oldenburgischer Separatisten. PοωερZDiskussion 17:53, 2. Jan. 2014 (CET)
Spiegel-Artikel von 1975. --Wrongfilter ... 18:41, 2. Jan. 2014 (CET)

umgestaltung hannover

Wie sollte Hannover nach den Plänen von Elkhard umsgestltet werden in der NS Zeit

und wie waren die Pläne zur Nordstart Verwaltung in hannover

--80.147.118.183 10:00, 2. Jan. 2014 (CET)

Du meinst, wie Hannover mit Möchtegern- und Großkotz-Bauten verschandelt werden sollte!--79.232.195.47 11:00, 2. Jan. 2014 (CET)
Hier im Artikel sind weitere Stichwörter für eine Suche. Und da steht ein Modell, nach dem man auch mal suchen könnte. GEEZER… nil nisi bene 13:35, 2. Jan. 2014 (CET)
Na bitte! Da steht ganz unten, wo man sich den Krempel noch heute ansehen kann. GEEZER… nil nisi bene 13:39, 2. Jan. 2014 (CET)
Das wusste der sicher schon, nur wollte er Interesse wecken.--79.232.195.47 16:30, 2. Jan. 2014 (CET)
Regel I : Gehe von guten Absichten aus.
Regel II: Was man gemeinhin mit Hinterlist oder Bösartigkeit verbindet, ist in 95 % aller Fälle ein Mangel an Erfahrung und/oder Wissen (und solche Leute brauchen am meisten Hilfe....)
Regel III: Seit Andy W. hat jeder hat ein Recht auf 15. Minuten Ruhm. ;-) GEEZER… nil nisi bene 18:52, 2. Jan. 2014 (CET)

Eulensage von leipzig

Hallo guten tag kann mir bitte jemand sagen, in welcher hausnummer in der petersstraße sich die kleine Eule in einer niche im hof befindet. es grüßt g. h.--178.24.125.74 10:44, 2. Jan. 2014 (CET)

Eventuell Nr. 19 (auf des Bild klicken) ... GEEZER… nil nisi bene 11:11, 2. Jan. 2014 (CET)
Könnte aber auch Nr. 17 gewesen sein... GEEZER… nil nisi bene 11:14, 2. Jan. 2014 (CET)

uelensage

hallo danke für die bilder von der leipziger eule.leipzig information konnte auch nicht weiterhelfen, auch sonst wußte niemand davon, außer unser enkel(10 jahre) es grüßt g.h--178.24.125.74 16:32, 2. Jan. 2014 (CET)

Na ja, nach der Sage hier war der Herr der Fliegen diese Eule. Und nach getaner Arbeit (= Beendigung des Vertrages) hat er sich dann zurückgezogen und sitzt jetzt bei seiner Grossmutter vorm Kamin und trinkt Rotwein. Gruss an den Enkel - und er soll sich weiter für Eulen und so interessieren ... ;-) GEEZER… nil nisi bene 17:55, 2. Jan. 2014 (CET)

Notebook und externer Bildschirm

Ich habe ein Notebook Lenovo T430, das ich an meinen neuen Monitor NEC MultiSync EA244WMi anschließen möchte. Mit den mitgelieferten Kabeln scheint es nur mit dem normalen (analogen) Monitorkabel möglich zu sein. Die Bildqualität auf dem Monitor sieht damit nicht wirklich optimal aus. Da das Notebook weder über einen HDMI- noch über einen DVI-Ausgang verfügt, bin ich bezüglich Alternativen eingeschränkt. Ein Mini-DisplayPort ist immerhin vorhanden. Wäre vielleicht da ein Adapter zu DVI oder HDMI eine gute Lösung? Oder was sonst? Danke für eure Hilfe! --178.83.122.77 11:46, 2. Jan. 2014 (CET)

Würd ich genauso machen, ja. VGA zu DVI wär ja quatsch. --Eike (Diskussion) 11:57, 2. Jan. 2014 (CET)
Schau mal bei ebay nach der Artikelnummer 161076785364, dann hast Du die Anschlüße. War jetzt der erste Treffer, gibt es vielleicht noch günstiger. --91.0.188.89 12:02, 2. Jan. 2014 (CET)
Eine Docking Station würde ich eigentlich lieber vermeiden. Ginge vielleicht dieser Adapter auch? 178.83.122.77 15:15, 2. Jan. 2014 (CET)
Sorry, aber Dein Bildschirm hat laut der von Dir verlinkten Spez einen Display Port. Da brauchst Du nichts wandeln, sondern einfach nur ein Kabel von Mini-Displayport auf Normalgroß-Displayport. --Rôtkæppchen68 15:32, 2. Jan. 2014 (CET)
Gibt's sowas nicht billiger? [2] Auf jeden Fall Tipp für den Fragesteller: Es sollte nicht nötig sein, Kabel/Adapter für standardisierte Ports beim Hersteller des Geräts zu kaufen. --Eike (Diskussion) 15:37, 2. Jan. 2014 (CET)
Ich würde keine Lösung mit Wandlung auf DVI-D oder HDMI wählen, wenn der Bildschirm nativen Displayport unterstützt. So ein Kabel gibt’s auch billiger.[3] Das obige Kabel hab ich nur als Beispiel verlinkt. Nur ein Adapter von Mini-Displayport auf Displayport tut es nicht, da fehlt noch das eigentliche Monitorkabel. Und dann kann der Fragesteller gleich das Kabel mit den richtigen Steckern nehmen. --Rôtkæppchen68 15:53, 2. Jan. 2014 (CET)
Das sieht doch schon bess... billiger aus. :o) Muss natürlich jeder wissen, wieviel er zahlen will, aber mir persönlich sind so hohe Beträge für ein Kabel zu viel (wenn es keine tolle Wandlung machen muss). --Eike (Diskussion) 15:56, 2. Jan. 2014 (CET)
DisplayPort ist ursprünglich Apple-spezifisch. Für Apfelrechner und Zubehör sind die Leute bekanntlich bereit, Fantasiepreise zu zahlen. Das macht auch was aus. Und der Computerhändler um die Ecke (oben verlinkt) rechnet für unverzichtbares Zubehör sowieso eine höhere Marge. Blödmarkt und Co. machen das genauso. --Rôtkæppchen68 02:22, 3. Jan. 2014 (CET)

Jammern um die Kosten kommt jetzt zu spät. Beim nächsten Mal vorher identifizieren, was an Zubehör nötig ist und das dann ins Verkaufsgespräch so einbauen, dass es Aufpreisfrei beim Kauf (z. B.: des Monitors) dabei ist. Mit einer solchen Vorgehensweise wird man selbst bei Blödmarkt & Co. nicht als Gimpel behandelt. --84.178.63.216 11:47, 3. Jan. 2014 (CET)

Was heißt ULED

Im Artikel "Leuchtdiode" wird im Abschnitt 5.1 in der Formel die Größe ULED benutzt. Was ist damit gemeint? --DieterO (Diskussion) 14:03, 2. Jan. 2014 (CET)

U = Spannung, ULED = Spannung LED. Grüße Dumbox (Diskussion) 14:07, 2. Jan. 2014 (CET)

Gehört dann aber tiefgestellt. Und wenn es das nicht schon ist, dann sollte es so bald wie möglich tiefgestellt werden. Sonst verwechseln noch mehr Leute ULED beispielsweise mit OLED... --Eu-151 (Diskussion) 23:47, 2. Jan. 2014 (CET)

Das LED ist in der Formel schon tiefgestellt. -- Jonathan 00:39, 3. Jan. 2014 (CET)

Wird Pofalla Vorstand der deutschen Bahn oder nicht?

Das Satiremagazin Der Postillon berichtet darüber, aber auch seriöse Nachrichtenportale wie Spiegel, ARD, ... Ich bin verwirrt. --87.157.46.64 21:39, 2. Jan. 2014 (CET)

Der Postillon hat den Artikel auch erst heute geschrieben und auf gestern zurückdatiert, um eben diese Verwirrung zu stiften. Fiese Möpp. :) —[ˈjøːˌmaˑ] 22:55, 2. Jan. 2014 (CET)
.Gute Lösung, er wird die Verspätungen für beendet erklären.--79.232.195.47 23:00, 2. Jan. 2014 (CET)
Chapeau, Postillon! Der Umgang mit Realsatire ist immer eine Herausforderung für ein Satiremagazin. Elegant gelöst! Dumbox (Diskussion) 23:09, 2. Jan. 2014 (CET)
Oder Verspätungen werden als angegeben wenn der Zug schon abgefahren ist...--Antemister (Diskussion) 23:29, 2. Jan. 2014 (CET)
Der Bundesministerium für Verkehr und digitale Infrastruktur (aktuell Alexander Dobrindt) ist, als zuständiger Vertreter der Bundesregierung, sozusagen kommissarisch, Alleinaktionär der DB, für die Bestellung der Aufsichtsräte (unter anderen der ehemalige FDP Generalsekretär Patrick Döring) zuständig. Die DB wird also nicht, wenn Pofalla dort ein Büro bekommt, plötzlich ein Selbstbedienungsladen für goldene Fallschirme sondern sie ist wohl längst ein solcher. --Kharon 02:45, 3. Jan. 2014 (CET)
Das ist der eigentliche Skandal, dass auch die Bahn eine Versorgungseinrichtung für gescheiterte Politiker ist. Gut möglich, dass der das als Kanzleramtsminister noch selbst eingefädelt hat, als er erfahren hat, welch lukrative Posten die Bahn hat oder schaffen kann. Mit einem Salär zehnfach eines Ministers und praktisch ohne Erfolgszwang.--87.162.251.166 08:52, 3. Jan. 2014 (CET)
Ein guter Bericht über die Aktion des Postillon: http://www.rhein-zeitung.de/nachrichten/netzwelt/news_artikel,-Pofalla-absurd-Wie-der-Postillon-das-Netz-doppelt-trollte-_arid,1088341.html -- Felix König 11:16, 3. Jan. 2014 (CET)

Unterschied zwischen zwei TV-Modellen

Ich habe nun bestimmt eine Stunde gesucht, aber ich finde den Unterschied zwischen zwei Modellen von Samsung nicht heraus:

Auch diese Antworten haben mir nicht geholfen. Das obere Modell scheint besser verfügbar zu sein, das untere ist Testsieger in seiner Klasse. --178.83.122.77 04:52, 3. Jan. 2014 (CET)

Philips vergibt z.B. bei der 6000-Serie für den Deutschen Markt verschiedene Nummern für die einzelnen Einkaufsgemeinschaften/Händlerketten. Mediamarkt hat andere Bezeichnungen als z.B. ElectronicPartner. Es sind da tatsächlich auch kleine Unterschiede im Zubehör und Design. Die Hauptplatine ist identisch. Möglicherweise ist das bei Samsung auch so.--87.162.251.166 08:32, 3. Jan. 2014 (CET)
Ja ist bei Samsung auch so. Geräte für Mediamarkt/Saturn haben meist eigene Nummern, ganz absichtlich um die Kunden und automatisierte Preisvergleichssysteme zu verwirren. --El bes (Diskussion) 09:25, 3. Jan. 2014 (CET)
Hier scheint es sich eher um ein Modell für den österreichischen und eines für den deutschen Markt zu handeln, jedenfalls kennt die jeweilige Samsung-Landeswebsite jeweils nur eines der Modelle (wobei der UE55F7090 auf der deutschen Website nur angekündigt wird, zwar für März 2013, aber auf der Produkteseite wird in der Grösse und in der 7er-Serie ausschliesslich das alte ES-Modell präsentiert). Ich tippe also darauf, dass Samsung in Deutschland noch versucht, ES-Restbestände abzuverkaufen, während in Österreich das (vermutlich technisch zum F7090er-Modell identische) F7080-Modell längst angeboten wird, weshalb die Händler dann womöglich auch in Deutschland ggf. gerne das österreichische Modell anbieten. --YMS (Diskussion) 10:00, 3. Jan. 2014 (CET)

Da jeweils von Samsung geht beides nicht. Und die Sache mit dem Berliner Bildröhrenwerk ist auch noch keine zehn Jahre her und sollte bei einer Kaufentscheidung berücksichtigt werden. --84.178.63.216 11:55, 3. Jan. 2014 (CET)

Bearbeiten von Artikeln

Wenn ich beim Bearbeiten eines Wikipedia-Artikels einen Abschnitt markieren will und Strg+Shift+ drücke, wird nicht nur der betreffende Abschnitt markiert, sondern der gesamte restliche Artikel. Gleich ist es, wenn ich Strg+ drücke, dort wechsle ich, anstatt zum Ende des Abschnitts, zum Ende der Seite. Kann mir hier jemand helfen? --SteEis. (Diskussion | Bewertung | Beiträge) 18:56, 1. Jan. 2014 (CET)

Hab’s gerade ausprobiert: Bei mir passiert bei diesen Tastenkombinationen gar nichts. -- HilberTraum (Diskussion) 19:36, 1. Jan. 2014 (CET)
Welchen Browser und welches Betriebssystem nutzt du? Vielleicht kommt die Tastenkombination daher. Wieso erwartest du, dass genau der betreffende Abschnitt markiert wird (bis zu welcher Überschriftenebene) und wie soll das ggf. funktionieren? --mfb (Diskussion) 14:18, 2. Jan. 2014 (CET)
Ich benutze Windows 7 und den Internet Explorer 10. Ich erwarte nicht, dass bis zu einer Überschriftenebene markiert werden soll, sondern bis zur nächsten Zeilenschaltung. --SteEis. (Diskussion | Bewertung | Beiträge) 10:54, 4. Jan. 2014 (CET)

Schlüsseldienst Beauftragung durch Minderjährige

Meine 16-jährige Tochter hat über Silvester mit zwei Freundinnen in unserem Haus gefeiert. Wir Eltern haben woanders gefeiert. Um Mitternacht sind die drei raus und haben den Schlüssel vergessen. Da sie Ihre Handys nicht dabei hatten, konnten sie uns nicht erreichen, haben dann den Schlüsseldienst gerufen. Der hat geöffnet und den dreien 270€ in Rechnung gestellt. 200€ haben sie sofort gezahlt (soviel hatten sie dabei), den Rest sollen wir jetzt noch zahlen. Zwei Fragen: 1.) Durfte der Schlüsseldienst den Auftrag annehmen (d.h. sind 16-jährige für solche Summen geschäftsfähig)? 2.) Sind 270€ noch im Rahmen? Mir kommt das selbst unter Berücksichtigung der Tatsache, dass Silvester war, enorm teuer vor. (nicht signierter Beitrag von 93.132.147.240 (Diskussion) 22:25, 1. Jan. 2014 (CET))

Ad 1.) Ja er durfte, muss sich aber im klaren sein, dass das Geschäft aufgrund der beschränkten Geschäftsfähigkeit der minderjährigen Kundinnen schwebend unwirksam ist, solange es nicht von den Eltern der minderjährigen Kundinnen genehmigt wird. Die Eltern können also auf der Nichtigkeit des Geschäfts bestehen. Siehe auch Minderjährigkeit#Geschäftsfähigkeit und Geschäftsfähigkeit (Deutschland)#Beschränkte Geschäftsfähigkeit --Rôtkæppchen68 22:33, 1. Jan. 2014 (CET)
Wenn die Jugendlichen den Schlüsseldienst anrufen konnten hätten sie auch die Eltern anrufen können. --Mauerquadrant (Diskussion) 22:37, 1. Jan. 2014 (CET)
Hätte hätte Fahrradkette. Hamse aber nich. --92.229.95.189 22:41, 1. Jan. 2014 (CET)
In der Frage wird behauptet sie konnten die Eltern nicht anrufen. --Mauerquadrant (Diskussion) 04:22, 2. Jan. 2014 (CET)
Danke für die Antworten. Eine der Freundinnen hatte ihr Handy dabei, aber meine Tochter wusste unsere Handynummer nicht auswendig. Intelligenter wäre es gewesen, wenn Sie ein Taxi gerufen hätten und sich zu der Freundin bringen lassen hätten, die ca. 20km entfernt wohnt. Aber die Mädels haben die Kosten des Schlüsseldienstes total unterschätzt, sie hatten mit ca. 100€ gerechnet, die sie dann untereinander aufteilen wollten. Ich denke, ich werde die 70€ mit Hinweis auf Nichtigkeit des Vertrags nicht bezahlen. Die 200€ habe ich abgeschrieben.--95.112.188.193 09:35, 2. Jan. 2014 (CET)
Falls du ne Hausratversicherung hast, manchmal zahlen die sowas. Schau mal in die Versicherungsbedingungen, vielleicht hast du Glück. --Ambur (Diskussion) 10:13, 2. Jan. 2014 (CET)
Mal nebenbei gefragt: Was kostet denn heutzutage eine neue Glasscheibe? Bei alten (nicht Isolierglas) Fenstern hat man da einfach eine Scheibe eingeschlagen, war billiger als so was.--Antemister (Diskussion) 12:40, 2. Jan. 2014 (CET)
  • zu 1.) wie Rotkaeppchen, Ihr habt die Genehmigung nicht erteilt, daher ist das Geschäft schwebend unwirksam. Zwar haben die Jugendlichen (augenscheinlich) mit eigenem Geld bezahlt, aber nicht den vollständigen Betrag, so dass der Taschengeldparagraph nicht greift.
  • zu 2.) Schlüsseldienste sind sehr teuer. Sittenwidrig ist mW, was mehr als 100% über dem ortsüblichen Tarif liegt, und da dürften die 270 Euro, wenn auch an Sylvester, etwa an der Grenze sein. Es kommt auch darauf an, was er konkret gemacht hat (Standardmasche: gleich Schließzylinder wechseln, obwohl vollkommen unnötig), wie lange er da war und welchen Anfahrtsweg er hatte.
Wenn Du Dich auf Nicht-Geschäftsfähkeit berufst, solltest Du Dich besser gleich an einen Anwalt wenden, denn freiwillig wird der Dienst sicher nicht auf das Geld verzichten. Sonst ist die Gefahr groß, dass man Dir aus einer ungenauen Formulierung eine Genehmigung des Geschäfts strickt, insbesondere wenn Du nur die Restzahlung verweigerst, und bist am Ende gelackmeiert.--Zockmann (Diskussion) 13:25, 2. Jan. 2014 (CET)

Auch wenn der Vertrag mit dem Schlüsseldienst endgültig unwirksam sein sollte, heisst das ja nicht, dass der Schlüsseldienst gar keinen Zahlungsanspruch hat. Das hiesse nur, dass er keinen *vertraglichen* Zahlungsanspruch hat. Man könnte zum Beispiel über GoA nachdenken.89.204.138.153 16:12, 2. Jan. 2014 (CET)

Service: Geschäftsführung ohne Auftrag (Deutschland) --Vsop (Diskussion) 17:19, 2. Jan. 2014 (CET)

Da werdet Ihr kaum eine Chance haben. Die Jugendlichen haben eine Dienstleistung bestellt, diese wurde ausgeführt, Sie haben einen Anzahlung geleistet und es wurde ganz sicher quittiert, dass hier noch ein Betrag aussteht. Das Sie vorher nicht nach den Kosten gefragt haben ist nicht das Problem der Firma. Könnte man beliebig Kosten mindern oder einfach aus einem Verfrag schlüpfen, nur weil einer der beiden Vertragspartner unter 18 ist, währe das ein Geschäftsmodell das Betrüger das ununterbrochen missbraucht würde. Die Firma wird ganz einfach Mahnungen ausstellen und wenn Ihr nichts tut wird das am Ende einfach nur noch teurer. Jeglicher Rechtsbeistand wird um ein vielfaches teurer als die 70 Euro. Und selbst der wird nur dazu raten einfach zu zahlen. Der Schlüsseldienst hat sich ja gar nichts zu Schulden kommen lassen, sondern eine Dienstleistung angeboten und die ordnungsgemäß ausgeführt. 270 Euro inklusive Anfahrt und das in der Sylvesternacht? ... Das ist teuer, aber wirklich kein Wucher. --92.228.42.137 20:37, 2. Jan. 2014 (CET)
Womit du einfach alles ignorierst, was oben schon gesagt und verlinkt wurde (beschränkte Geschäftsfähigkeit von Jugendlichen, nicht erteilte Genehmigung der Eltern, Taschengeldparagraph etc). Wer als Unternehmer teure Geschäfte mit Kindern/Jugendlichen machen will, der muss eben auch mit dem Risiko leben, dass die Eltern ihre Zustimmung nachträglich nicht erteilen. Das wissen die Unternehmer in der Regel auch, weswegen das auch nicht als Betrugsmasche gegen sie taugt. Bevor ein vernünftiger Unternehmer eine Leistung von mehreren hundert Euro an Minderjährige erbringt und in Vorleistung tritt, wird er nämlich erst einmal auf Kontakt mit den Eltern bestehen. Vermutlich haben die IP 89. und Vsop Recht mit dem Hinweis auf GoA, da die Kontaktaufnahme hier nicht so einfach möglich war und gewissermaßen eine Notlage vorhanden war, allerdings ist das eine andere Argumentation als deine. --Wermichangreiftwirdvernichtet (Diskussion) 22:30, 2. Jan. 2014 (CET)
Meine Argumentation ist, dass zwischen dem was theoretisch aufgrund der Minderjährigkeit anfechtbar währe und der Handhabung in der Realität ein himmelweiter Unterschied besteht. Die komplette Minderjährige Bevölkerung kauft permanent im Internet vom Lolli bis zum Rechner alles mögliche ein ohne je nach dem Alter gefragt zu werden. (Verpflichtende Altersangaben sind die absolute Ausnahme) Und in diesem Fall der 70 Euro ist das sowieso ausschließlich Theorie. Es gibt keine Möglichkeit irgendeine Maßnahme zur Preisminderung einzuleiten die weniger als 70 Euro kostet. --92.228.42.137 01:03, 3. Jan. 2014 (CET)
Die gibt es sicher. Die Eltern teilen dem Aufschließdienst mit, dass für das fragliche Geschäft keine Einwilligung i.S.v. §106ff BGB erteilt wird. Das kann sogar formlos erfolgen, sollte aber in jedem Falle beweisbar sein. Sollte der Aufschließdienst auf seiner Forderung beharren, geht das in jedem Fall auf seine Kosten (Inkassodienst, Mahnbescheid etc). --Rôtkæppchen68 01:21, 3. Jan. 2014 (CET)
Rotkäppchen, das ist eine mutige Position. Wie oben gezeigt, hat der Schlüsseldienst duchaus die Chance, an sein Geld zu kommen, auch wenn die Eltern die Einwilligung verweigern. Und wenn die Forderung iE berechtigt ist, dann gehen Mahn-etc Kosten nicht zu Lasten des Schlüsseldienstes.

89.204.137.94 07:49, 3. Jan. 2014 (CET)

Das wird nach dem abgeschlossenen Mahnverfahren bei dem Amtsgericht landen. Was die Eltern hier versuchen ist keine Rückabwicklung von irgend einem Handyvertrag der die Zustimmung der Erziehungsberechtigten erfordert hätte oder so was, sondern eine Preisminderung durchzusetzen für eine bereits abgeschlossene Dienstleistung die bereits angezahlt wurde. Dem Richter zu verklickern das man jetzt irgendwie 70 Euro weniger zahlen will, weil der Schlüsseldienstmann der mitten in der Nacht den eigenen Kindern die Tür aufgeschlossen hat, versäumt hat nach dem Alter zu fragen, dürfte doch wirklich ausgesprochen schwierig werden. Theoretisch kann mans natürlich mit Berufung auf §106ff versuchen, in der Praxis wird das nicht laufen. --92.228.42.137 10:41, 3. Jan. 2014 (CET)
Quatsch! Die wollen keine Minderung eines bestehenden Vertrags, sondern sie verweigern die Genehmigung zu einem Vertrag, dem sie nie zugestimmt haben. Und das ist ihr gutes Recht! So gesehen stellt sich vielmehr die Frage, inwiefern die Eltern den bereits gezahlten Betrag zurückerhalten können. Was können die Eltern dafür, dass der Schlüsseldienst so doof war, das Geschäft durchzuführen, obwohl er nicht wusste, ob die Kinder vll. dieses Geschäft gar nicht wirksam tätigen können? Eigene Dummheit. Würden die ihr Geld dann doch bekommen, bräuchte man ja gleich gar keinen Minderjährigenschutz; der Vertragspartner würde sein Geld ja doch bekommen. Das ist nicht Sinn des Minderjährigenschutzes! § 107 BGB gilt übrigens analog auch für die oben angesprochene GoA. --88.130.90.34 16:46, 3. Jan. 2014 (CET)
Die angezahlten 200 Euro würde ich nicht versuchen zurückzubekommen, da sie mit eigenen Mitteln bewirkt wurden. Da sieht es schlecht aus, siehe Taschengeldparagraph. --Rôtkæppchen68 07:21, 4. Jan. 2014 (CET)
Nein Rotkäppchen, da liegst du mMn falsch. "Bewirkt" meint die vollständige Zahlung des Preises und das haben die Kinder mit ihrer Teilzahlung nicht getan. --88.130.114.151 13:33, 4. Jan. 2014 (CET)
Für 2011 nennt Die Welt vom 13. Juni2011) Preise fürs Türöffnen von 50 bis 100 Euro (faire Schlüsseldienste") über 200 Euro ("an den falschen Monteur" geraten und Nachtzuschlag) und "in der Spitze weit über 300 Euro". Und weiter: "Eine Türöffnung solle tagsüber nicht mehr als 70 bis 80 Euro kosten, nachts etwa 100 Euro." Das Problem ist, daß die Welt ebenso wie der Fragesteller bei den Angaben über den konkreten Umfang der erbrachten Leistung (inkl. Anfahrtkosten) diffus bleibt. Der geforderte Betrag von 270 Euro ist also möglicherweise sachlich kaum anzugreifen. Auch der Verweis auf den Taschengeldparagraph ist ein wichtiger Aspekt. Vor 15 Jahren lag bei 16- bis 17jährigen der Schnitt angeblich noch zwischen 35 und 45 Euro [4]. Das dürfte heute deutlich mehr sein. Und wenn man sich auf Klagen und Verklagtwerden einläßt sollte man sich auch darüber im Klaren sein, daß man bei einem durchaus denkbaren Vergleich möglicherweise auf den eigenen Kosten sitzenbleibt und die Gesamtkalkulation dadurch deutlich nachteilig wird. Ich würde stattdessen über den Begriff "Lehrgeld" nachdenken und nicht gutes Geld dem schlechten Geld noch hinterherwerfen. --77.12.35.119 11:56, 4. Jan. 2014 (CET)
Die Abzocker behaupten auch gerne ihr Schlossknacker sei 100 km vom Kunden entfernt und berechnen dazu die Anfahrt. Tatsächlich wohnt der um die Ecke, nur der angebliche Firmensitz ist was weiß ich wo. Es gibt dazu ein Geflecht von Firmen und untereinander verschalteten Telefonnummern, so dass gewährleistet ist, dass der Abzocker Personal in der Nähe hat, aber einen passenden Firmensitz weit weg hat und entsprechende Fahrtkosten abgreifen kann. Grundsätzlich gilt: Erst nach Tarif fragen, dann erst beauftragen. Ich habe mich auch mal ausgeschlossen, war Sonntag oder Feiertag, der Schlüsseldienst kostete mich damals um die hundert Euro für Aufmachen, Anfahrt und Feiertagzuschlag. Er versicherte mir, dass er nicht aufbohren muss und er war auch ungefähr in einer halben bis dreiviertel Stunde da. Es war ein renomierter Schlüsseldienst, solider Handwerker aus der Gegend und die Tarife erschienen mir angemessen. Aber auch der wollte Bares.--Giftzwerg 88 (Diskussion) 13:39, 4. Jan. 2014 (CET)

Drogentest

Wie nennt sich und woher bekommt man Drogen-/(Pillen)teststreifen? Es geht hier nicht um Drogenschnelltests, also den Nachweis im Urin, sondern um den Test der Substanz. Ich erinnere mich, einst ein Testkit im Internet gesehen zu haben, das auf ca. 20 potentiell in Ecstasypillen enthaltene Wirkstoffe testet, der Preis war für 100 Streifen um 20 Dollar, aber ich weiß nicht mehr, wo ich es fand und auch nicht, wie ich danach suchen soll. Besten Dank im Voraus!

--88.75.118.15 19:55, 2. Jan. 2014 (CET)

Don't do drugs, egal was drin ist. --87.156.54.246 20:02, 2. Jan. 2014 (CET)
Danke für den Tip, aber das geht an der Fragestellung vorbei. Moralisieren und Zeigefingerheben kann man hier! (nicht signierter Beitrag von 88.75.118.15 (Diskussion) 20:28, 2. Jan. 2014 (CET))
Ich weiß nicht, wie man die nennt oder wo man die herbekommt, aber "früher" gab es Vereine, die Drogentests angeboten haben. Gibt's sowas nicht mehr? --Eike (Diskussion) 20:32, 2. Jan. 2014 (CET)
Gibt es denke ich schon noch, evtl. sogar vermehrt, aber da weiß ich auch nicht, wohin ich mich wenden soll, der deutschen Drogenpolitik sei Dank...bekommt man sowas evtl in der Apotheke? (nicht signierter Beitrag von 88.75.118.15 (Diskussion) 20:36, 2. Jan. 2014 (CET))
Geh zu Deinem Hausarzt, der hat Schweigepflicht. Oder falls Du Deinen Hausarzt von dieser entbunden hast, weil es z.B. um eine Einstellungsuntersuchung mit Drogentest geht, geh zu einem anderen Allgemeinmediziner, der nicht Dein Hausarzt ist. Falls Du meinst, Du wirst von anderen unter Drogen gesetzt, oder jemanden in Deinem Umfeld des Konsums verdächtigst und von ihm berührte Gegenstände beproben willst, denk daran, dass die Dinger recht empfindlich anschlagen und z.B. fast jeder von uns Kokainanhaftungen in einer für den Test-Ausschlag ausreichenden Menge an den Fingern hat, wenn er gerade ein paar Geldscheine gezählt hat - ein Nachweis, dass derjenige selbst konsumiert (ob nun freiwillig oder weil man ihm heimlich irgendwas irgendwo eingefüllt hat), ist auf diese Art nicht möglich. -- 88.67.153.104 21:34, 2. Jan. 2014 (CET)
Weder noch, SWIM will eine Pille testen. (nicht signierter Beitrag von 88.75.118.15 (Diskussion) 21:39, 2. Jan. 2014 (CET))
eztest.com. --Rôtkæppchen68 22:44, 2. Jan. 2014 (CET) Ich sehe gerade, wir haben einen Artikel dazu: Marquis-Reagenz. Das kannst Du also zur Not selber mixen. en:Marquis reagent listet die Farben auf. --Rôtkæppchen68 22:53, 2. Jan. 2014 (CET)
Vielleicht kann auch das Portal:Radsport weiterhelfen. --91.50.126.144 00:25, 3. Jan. 2014 (CET)
Warum schränkst du das auf eine einzelne Sportart ein? --Benutzer:Duckundwech 13:24, 3. Jan. 2014 (CET)
ich hab nur eine österreichische Adresse: >>checkyourdrugs.at<< die haben sicher noch eine Adresse für dich. --80.108.60.158 16:15, 3. Jan. 2014 (CET)
Schau mal bei Eve & Rave Schweiz, vielleicht ist deine Pille dabei. Oder direkt eine Probe hinschicken. --84.187.83.128 17:23, 3. Jan. 2014 (CET)
Genau, so hießen die (schon "damals")! --Eike (Diskussion) 21:11, 3. Jan. 2014 (CET)

Bayerische Lokalsender

Warum gibt es in Bayern mehrere Lokalsender, die Charivari oder Gong heißen? Einen gemeinsamen Gesellschafter haben die Sender nicht. --RadiohörerDiskBewertung 20:51, 2. Jan. 2014 (CET)

Alle diese Sender gehören zumindest zum Teil der Die Neue Welle Rundfunk-Verwaltungsgesellschaft mbH & Co. KG. --Rôtkæppchen68 22:09, 2. Jan. 2014 (CET)
Radio Charivari Rosenheim aber beispielsweise nicht. --RadiohörerDiskBewertung 23:47, 2. Jan. 2014 (CET)
In diesen Falle läuft das indirekt über Radio Galaxy. --Rôtkæppchen68 23:55, 2. Jan. 2014 (CET)
(BK) Studio Gong ist da ein weiterer Hotspot. Das ganze Kooperations- und Beteiligungsgeflecht scheint am Ende auf die üblichen Verdächtigen hinauszulaufen: Müller Medien, SV, Familie Burda. Mir san mir, oder so. Grüße Dumbox (Diskussion)
@Rotkæppchen68: Wie denn? Die Radio Charivari Rosenheim Programmanbieter GmbH & Co. ist Veranstalter der Sender Charivari Rosenheim und Radio Galaxy Rosenheim, an dieser GmbH sind eine Gesellschaft bürgerlichen Rechts sowie eine WWZ GmbH, die verschiedenen Zeitungsverlagen gehört beteiligt. Beide haben keine Beziehung zur neuen Welle und sind an keinem anderen Charivari-Sender beteiligt. Oder liege ich da falsch? --RadiohörerDiskBewertung 00:24, 3. Jan. 2014 (CET)
Auch da gerät man wieder, wenn man sich durchklickt, in ein Gestrüpp von Beteiligungsgesellschaften. Wo man's als Laie noch nachvollziehen kann, landet man immer bei derselben Handvoll Medienmogule. Dirk Ippen scheint gut dabei zu sein. Wie die kleinen Anteilseigner miteinander verbandelt sind, kann man vermutlich nicht wissen. Ist in der Sache aber auch egal. Offensichtlich darf sich jedes Radio Gong Radio Gong nennen; ich wüsste jedenfalls von keinem Markenstreit. Also kann man davon ausgehen, dass man sich einig ist, vielleicht in der Art eines Franchise-Modells. Grüße Dumbox (Diskussion) 09:08, 3. Jan. 2014 (CET)
Viele kleine Firmen mit ähnlichem Namen, keiner klagt wegen Markenrecht, da kommt doch gleich der Gedanke: Man steckt irgendwie unter einer Decke und Gewerkschaft soll draußen bleiben.--84.187.83.128 17:43, 3. Jan. 2014 (CET)

Breaking Bad in Hannover

War das eine Ente? http://www.bild.de/unterhaltung/tv/breaking-bad/kultserie-dreht-bald-in-hannover-25517984.bild.html --77.3.177.60 00:01, 3. Jan. 2014 (CET)

Vermutlich -- Jonathan 00:47, 3. Jan. 2014 (CET)
Eben. IMDb weiß auch nichts davon. Außerdem war es sowieso Hamburg und nicht Hannover. --Rôtkæppchen68 01:31, 3. Jan. 2014 (CET)
Welche Szenen sollen in Hamburg gedreht worden sein? Ich hab die letzte Staffel neulich gesehen und da spielte nichts in Deutschland; weder in Hannover noch in Hamburg... --88.130.90.34 16:26, 3. Jan. 2014 (CET)
Guck dir noch mal Folge zwei an. Und Hannover ist richtig, nicht Hamburg. --Komischn (Diskussion) 17:34, 3. Jan. 2014 (CET)
Richtigstellungen wie diese scheinen meinen Eindruck zu bestätigen. --88.130.90.34 23:54, 3. Jan. 2014 (CET)

Jesu Geburt die Weihnachtsgeschichte

--88.75.123.239 10:45, 3. Jan. 2014 (CET)

Siehe Weihnachtsgeschichte. --Eike (Diskussion) 10:46, 3. Jan. 2014 (CET)
?????????? --84.135.132.143 18:24, 3. Jan. 2014 (CET)

Handwerklich Frage

Da ist mal die Fachgruppe Handwerk gefragt. Die bessere Hälfte des Hauses wurde vor Jahren im Hauswitschaftsraum mit einer Klemmstange beglückt, die zwischen den beiden Raumwänden klemmt, um Wäsche, die nicht in den Trockner darf, aufzuhängen. Nun gibt diese Stange mit der Zeit etwas nach und verrutscht, weswegen ich sie fixieren will. Trotz Internetrecherche fiel mir bisher nix vernünftiges ins Auge. Es müßte im Prinzip eine Art anschraubbarer Teller sein, der an beiden Enden jeweils die STange aufnimmt. Hat jemand ne Idee?--scif (Diskussion) 12:50, 3. Jan. 2014 (CET)

Mach Foto von jetziger Befestigung. Oder willst du komplett austauschen? GEEZER… nil nisi bene 12:55, 3. Jan. 2014 (CET)
Ich verwende dafür so etwas (aber aus Holz). In der Mitte (unsichtbar) ist eine Bohrung, durch die der Halter an die Wand gedübelt werden kann und die Stange hält. GEEZER… nil nisi bene 12:59, 3. Jan. 2014 (CET)
Nix kompletttausch. Dir ist eine Klemmstange, die durch Federkraft hält nicht geläufig? Z.B. für Duschvorhänge? Ich hab sie halt zweckentfremdet. Ein Beispiel--scif (Diskussion) 13:01, 3. Jan. 2014 (CET)
Doch, ja. Deswegen habe ich sie ja durch die Auflagehilfe ersetzt. Da kann man sie auch leicht herausnehmen. :-) GEEZER… nil nisi bene 13:54, 3. Jan. 2014 (CET)
Für die Gardinchen an den Fenstern hab' ich was ähnliches: Die zwei Hauptteile der teleskopartigen Stange werden mit einer innenliegenden Feder auseinander gedrückt. Den „Arbeitspunkt“ der Feder kann man aber durch verdrehen der inneren zur äusseren Stange in Grenzen einstellen. Vielleicht geht das bei Dir auch und die Kraft läßt sich noch ein bisschen erhöhen? --84.178.63.216 13:15, 3. Jan. 2014 (CET)
Du könntest im Baumarkt in der Möbelbeschlagabteilung mal nach Kleiderstangenbeschlägen suchen. Die gibt es evtl auch bei Amazon. Duschvorhangstangen haben ähnliche Befestigungen, aber meistens eine Innenmuffe. Die Trockenstange müsste also auch an den Enden innen hohl sein. --Rôtkæppchen68 15:12, 3. Jan. 2014 (CET)
Kauf dir ein Glas Senf und pack die Bohrmaschine aus: ultimative Anti-Rutsch-Armatur ein lächelnder SmileyVorlage:Smiley/Wartung/:)  --тнояsтеn 15:11, 3. Jan. 2014 (CET)
Besser zwei Gläser Senf, sonst muss Scialfa den Deckel durchsägen. --Rôtkæppchen68 15:16, 3. Jan. 2014 (CET)
bei der Feder-Technik würde ich Vorsicht einmahnen, weil es besonders bei "alten" Ziegelwänden eventuell zu Beeinträchtigungen des Mauerwerks kommen könnte. Betonwände sind harmlos. Wie schon erwähnt: Anschrauben wäre die "statisch" vernünftigere Lösung. mit hübschen Schrauben könnte es eventuell sogar nett aussehen. Sollte aber auf nette Schrauben kein Wert gelegt werden, so könnte man eventuell auch noch die Stange einstemmen und das Loch offen klaffen lassen --80.108.60.158 16:10, 3. Jan. 2014 (CET)
Alternative: Stange entfernen, Trcokner bei ebay einstellen, mehrere Haken in die Wand dübeln und eine Wäscheleine in parallelen Reihen spannen. Es freuen sich bessere Hälfte, Umwelt und Geldbeutel. --91.0.188.84 16:18, 3. Jan. 2014 (CET)
Alternative II: Montagekleber unter die Federteller, 24 Stunden warten und die Sache hält bombenfest.--87.162.251.166 16:23, 3. Jan. 2014 (CET)
ich weiß zwar nicht wie hoch die Räume sind, aber ich kenne solche Dinger welche man per FlaschenzugSystem an den Plafond zurrt. --80.108.60.158 16:27, 3. Jan. 2014 (CET)

Ich danke für die tw. kreativen Vorschläge, mit dem Senfglas würde meine Frau dich wahrscheinlich erschlagen @ Thorsten. An so Kleiderstangebeschläge hatte ich auch schon gedacht, die handelsüblichen sind dann aber zu klein. Die Lösung von Geezer wirds wohl werden, elegant und sauber. Problem war, wie so oft, den richtigen Begriff zu finden. Wandlager muß einem erstmal einfallen. Also, Danke vielmals und es zeigt sich wieder, das WP doch besser als so manches Forum sein kann.--scif (Diskussion) 01:48, 4. Jan. 2014 (CET)

Nachtrag: Die passenden Wandlager für meine Stange... habe ich heute im Baumarkt gekauft, Stück 2,...€, hingen unter den Klemmstangen.... Aber schön, das wir mal drüber gesprochen haben.--scif (Diskussion) 13:07, 4. Jan. 2014 (CET)

Tagesschau

Gibt es eigentlich ein Grund, warum die Tagesschau um 20:00 Uhr auf so vielen Sendern gleichzeitig übertragen wird? Dass sie auf den Sendern Das Erste, 3sat (DACH-Übertragung), Phoenix (mit Gebärdensprache) und tagesschau24 (nomen est omen) übertragen wird, verstehe ich ja, aber wozu müssen die dritten Programme das auch noch zeigen? —Morten Haan · Wikipedia ist für Leser da 14:22, 3. Jan. 2014 (CET)

Spekulation: Die Dritten wollen weder dem Ersten Konkurrenz machen, noch, dass viele Menschen umschalten und dann ggf. beim Ersten hängenbleiben. --Eike (Diskussion) 14:28, 3. Jan. 2014 (CET)
Bei den ARD-Sendern ist das Programmschema traditionell darauf abgestellt. Es erlauben sich nur wenige ARD-Sender (BR inkl. BRalpha, MDR inkl. Kika und Einsplus vom SWR) statt Tagesschau etwas anderes zu senden. Sogar Nicht-ARD-Sender haben gegen die Macht der Gewohnheit kapituliert und senden zwischen 20 Uhr und 20:15 irgendeinen unwichtigen Schrott, da ja sowieso jeder Tagesschau schaut. Versuche von ZDF und Privatsendern, das Hauptabendprogramm nicht um 20:15 Uhr beginnen zu lassen, sind alle gescheitert. Arte wurde in zwei Programme aufgespalten, weil den deutschen Fernsehzuschauern ein an den französischen Sehgewohnheiten orientiertes Zeitschema nicht zu vermitteln war. --Rôtkæppchen68 14:32, 3. Jan. 2014 (CET)
TS ist als Aushängeschild der ÖR sakrosankt, Kollateraleffekt natürlich jene Umschalteffektsynergie, wie von Eike geschildert. VG--Magister 14:33, 3. Jan. 2014 (CET)
Die Dritten sind Vollprogramme und müssen daher auch Nachrichten bringen. Da greifen sie natürlich zur Tagesschau aus der eigenen Senderfamilie, ergänzt von Regionalnachrichten zu anderen Zeiten. Rainer Z ... 14:33, 3. Jan. 2014 (CET)
Heute morgen haben wir überlegt, was ein Programm so alles haben muss, danke daher für den Hinweis auf Vollprogramm! Ein Kollateralnutzen... --Eike (Diskussion) 14:38, 3. Jan. 2014 (CET)
Die Unterscheidung zwischen Voll- und Spartenprogramm gibt es beim ÖRR in Deutschland offiziell nicht, außerdem sind Regionalnachrichten ja auch Nachrichten. —Morten Haan · Wikipedia ist für Leser da 14:44, 3. Jan. 2014 (CET)
Die würden dann eigene Nachrichten machen müssen, was extra Geld kostet und letztendlich die Gebühren erhöhen würde. Aber ich gebe Dir insofern recht und frage mich auch, warum die Tageschau auf allen Kanälen gezeigt werden muss. Offensichtlich will die ARD sich nicht gegenseitig Konkurrenz machen. Andererseits bringen die Bayern die Tagesschau auf BR nicht um 20 Uhr. Es gehört wohl eine Portion Eigenbrötlerei dazu aus den Rahmen zu springen.--87.162.251.166 14:36, 3. Jan. 2014 (CET)
An den Kosten dürfte es wohl nicht liegen, da ARD, ZDF und das DRadio durch ihre Steuer mehr einnehmen als jeder Privatsender und die Gebühren auch wieder durch Sportrechte oder Starmoderation verpulvern... typisch unser ÖRR. Die Dritten wissen wohl eher nicht, was sie um 20 Uhr zeigen sollen, deshalb die Übernahme der Tagesschau. --RadiohörerDiskBewertung 17:04, 3. Jan. 2014 (CET)
Das kommt vielen Zuschauern (insbesondere der Generation "mit den Dritten sieht man besser") aber entgegen: Wenn man um 19.00 Uhr damit beginnt, das regionale Vorabendprogramm zu schauen und um 20.15 Uhr das regionale Abendprogramm sehen will, braucht man um 20.00 Uhr nicht umschalten, wenn man die TS sehen will. Nebenbei: Gibt es Statistiken, wie die Zuschauerverteilung der Tagesschau in den den einzelnen Sendern aussieht? 2A00:1398:9:FB00:1C77:B9EB:EEB3:C611 17:23, 3. Jan. 2014 (CET)
In den ersten 3 Quartale von 2012 sah es so aus: http://www.infosat.de/Meldungen/?msgID=69710
4,92 Mio beim Ersten, 3,78 Mio bei den anderen Kanälen --Baladid Diskussion 11:23, 4. Jan. 2014 (CET)

Boot aus Flugzeugteil

Mir fielen bei der Bildersuche für den Artikel Ferienlager diese zwei Fotos auf. Ich vermute das es sich bei dem Boot um einen Schwimmer oder Rumpf eines Wasserflugzeuges handelt. Oder kann das jemand von euch noch genauer einordnen. --sk (Diskussion) 21:13, 3. Jan. 2014 (CET)

Habe mal Bilder der Arado Ar196 zugefügt. Entscheide selbst, ob das passt. Von der Arado waren viele im WW II im Einsatz, u.a. auch bei den Russen noch bis 1950. --79.232.209.99 09:01, 4. Jan. 2014 (CET)
Das passt perfekt, die zwei Leisten auf der Oberseite, die rundliche Luke im vorderen Teil. Ich füge das mal mit in die Kategorie ein. Danke. --sk (Diskussion) 09:18, 4. Jan. 2014 (CET)
Der Schwimmer hatte 7 Kammern (Schotten) in denen Gummitanks für Sprit eingelegt waren. Daher die Luken für das Einlegen und den Tankstutzen. Ich meine auch, dass es sich bei dem Boot um die halben Reste eines solchen Schwimmers der Ar handelt. Weil das Hinterteil fehlt liegt der Schwimmerrest nach vorne geneigt in Wasser. Das zweite Indiz ist die (fehlende) Bugkappe, der Schiffsbeschlag scheint mir später angebracht zu sein. --79.232.209.99 10:49, 4. Jan. 2014 (CET)

Energieeffizienzklasse Haushaltsgeräte

inwiefern unterscheidet sich ein A+++ Gerät zu einem A++ und zu einem A+ Gerät KONKRET. Im Internet finde ich viel belangloses Geschwurbel darüber aber keine Tabelle, die konkrete Daten liefert. --93.132.58.36 23:28, 3. Jan. 2014 (CET)

Siehe Artikel Energieverbrauchskennzeichnung und dort referenzierte Quellen. --Rôtkæppchen68 23:34, 3. Jan. 2014 (CET)
Auf der Website des Umweltbundesamtes sind die EU-Verordnungen mit Mess- und Berechnungsvorschriften für die verschiedenen Gerätearten verlinkt, für Kühlschränke z.B. DELEGIERTE VERORDNUNG (EU) Nr. 1060/2010 DER KOMMISSION vom 28. September 2010 zur Ergänzung der Richtlinie 2010/30/EU des Europäischen Parlaments und des Rates im Hinblick auf die Kennzeichnung von Haushaltskühlgeräten in Bezug auf den Energieverbrauch. --Rôtkæppchen68 23:39, 3. Jan. 2014 (CET)

Delayed gratification - Belohnungsaufschub

Die englischen Kollegen haben einen langen Artikel, wir einen kurzen quelllosen. Psychologen in meinem Bekanntenkreis kennen den englischen Begriff nicht und den deutschen als Fachterminus eher auch nicht. Ist das Konzept tatsächlich in der englischen Psychologie viel verbreiteter? --92.202.57.120 23:54, 3. Jan. 2014 (CET)

Das ist eine wichtige Sache, die in der engl. Literatur den Sprung in die pop science gefunden hat. Hier was ein Lexikon dazu sagt. Auch hier oder hier. Sind es vielleicht Freudianer ? Oder Jung-Jünger? :-)) GEEZER… nil nisi bene 00:05, 4. Jan. 2014 (CET)
Visuelle Umsetzung der Begriffsverwendung: delayed gratification und Belohnungsaufschub. Das mag erklären, warum der deutsche Begriff deinen (jungen ?) Freunden weniger geläufig ist. GEEZER… nil nisi bene 11:30, 4. Jan. 2014 (CET)
Wird von Daniel Goleman in seiner Auslegung der emotionalen Intelligenz als ein Indikator aufgeführt. Nach dieser Interpretation ist die Beherrschung der eigenen Triebe ein wesentliches Merkmal emotionaler Intelligenz. Yotwen (Diskussion) 15:57, 4. Jan. 2014 (CET)

Hetracil und Homofin

Das Medikament Hetracil gegen Homosexualität und effenimiertes Verhalten gibt es ja jetzt schon ein Weilchen. Neu auf dem Markt ist seit kurzem das türkische Medikament Homofin. Leider werde ich aus den jeweiligen Websites nicht schlau. Bei Hetracil sind 20 Milligramm drin, aber wovon? Bei Homofin steht auch nichts von den Wirkszoffen. Was ist in diesen Medikamenten eigentlich drin? --84.147.113.213 02:44, 3. Jan. 2014 (CET)

Hetracil ist einfach nur ein Scherz und Homofin anscheinend Geldschneiderei auf Pflanzenbasis. PοωερZDiskussion 02:54, 3. Jan. 2014 (CET)
Sehe es eher als Geldschneiderei auf Menschenbasis. Die Homofin-Hersteller wurden ausserdem von der Psychiatric Association of Turkey (PAT) verklagt. Warum? Das ist wohl ableitbar ... :-) GEEZER… nil nisi bene 10:35, 3. Jan. 2014 (CET)
Von Putin werden wahrscheinlich auch gigantische Schadenersatzforderungen kommen. Immerhin hat er den ganzen Vorrat aufgekauft, um es den Sportlern in Sotschi unters Essen im Olympischen Dorf zu mischen. Ohne Hetracil und Homofin würden die Spiele ja sonst zu einem Homopropagandafestival und zum Fiasko für die russische Homophobiestaatsideologie. --Rôtkæppchen68 11:13, 3. Jan. 2014 (CET)
Das Zeug findet in arabischen Ländern vermutlich reißenden Absatz. Saudi Arabien und Kuwait versuchen gerade einen medizinischen Schwulenschnelltest zu entwickeln, den man an Flughäfen durchführen kann um Homosexuelle an der Einreise zu hindern. (kein Witz) --92.228.42.137 12:47, 3. Jan. 2014 (CET)
Ja, das hab ich irgendwo gelesen. Schlimm sowas. Und Lukaschenko darf jetzt über Frau Hendricks herziehen, nachdem Westerwelle weg ist. Peter Altmaier traut sich nach wie vor nicht aus dem Kleiderschrank. Er hat wohl Angst, von Dobrindt als schrill bezeichnet zu werden. --Rôtkæppchen68 13:01, 3. Jan. 2014 (CET)
Die Frage ist wohl ernst gemeint. Daher: Dass es ein Medikament geben könnte, das 'gegen Homosexualität wirken' könnte, ist wissenschaftlich gesehen völliger Humbug. Zudem ist Homosexualität auch keineswegs eine 'Krankheit', und daher weder therapiebedürftig noch -fähig. Sie wird in der Wissenschaft vielmehr als gleichberechtigte/gleichwertige (!) Variante des sexuellen Verhaltens angesehen, siehe etwa Homosexualität#Übersicht und Ex-Gay-Bewegung#Kritik. Und wer meint, sich selber oder andere so therapieren zu können/müssen, könnte zB mal das hier über verbreitete Vorurteile lesen. Besagte Frau Neu-Umweltministerin Hendricks hat laut ihrem Artikel übrigens mal eine brennende Zigarette auf dem Handrücken eines CDU-Politikers ausgedrückt. Da werden bei mir Assoziationen zu aktuellsten Geschehnissen geweckt... Frau Hendricks, wir brauchen sie heute mehr denn je ;-) Solaris3 (Diskussion) 19:08, 3. Jan. 2014 (CET)
Errm, ob Homosexualität grundsätzlich nie therapierbar sein wird ist wohl noch nicht abschließend geklärt. Es gibt ja schließlich medizinische Gründe für zumindest Homosexualität unter Männern. Währe das nicht der Fall bliebe als Ursache höchstens noch der Wille Gottes als Grund übrig und das ist doch recht unwahrscheinlich. :) Aktuell geht man von einem Hormonungleichgewicht während der Schwangerschaft aus. So dass es möglicherweise auf absehbare Zeit nicht direkt therapierbar sein wird, aber in nicht all zu ferner Zukunft eine Art Impfung der Mutter durch künstlich zugeführte Hormone während der Schwangerschaft geben könnte. Der medizinische Aspekt von Homosexualität ist nur deshalb so wenig erforscht, weil kein Arzt und Wissenschaftler der Wert auf seine Karriere legt das Thema auch nur anspricht. Zu behaupten, dass es völliger Humbug wäre, dass es je eine Therapie gegen Homosexualität geben könnte ... ist völliger Humbug. --85.181.211.239 00:48, 4. Jan. 2014 (CET)
Irgendwie fällt mir bei deinen Argumentation jemand ein - er hieß Wouter Basson, ein Südafrikaner. Der hätte seine helle Freude an dir, er wollte unter anderem per Pille Schwarze zu Weißen machen (oder wars umgekehrt?). Er lebt noch, vielleicht kann er dir ja helfen bei deinem Projekt. Viel Glück. Solaris3 (Diskussion) 01:01, 4. Jan. 2014 (CET)
Mir sträuben sich bei der Agumentation der IP alle Nackenhaare. Wenn ich höre "... Therapie gegen Homosexualität ...", könnte ich k...! Mir ist keine Lesbe oder Schwuler bekannt, die bzw. der so etwas will. Wo leben wir? Mehr Diskussion lohnt imho mit der IP nicht. --Stefan1973HB Disk. 01:31, 4. Jan. 2014 (CET)
Lieber Stefan1973, du verstehst ihn nicht. Er kennt nach obiger Aussage Gottes Plan bzw. Willen (Zitat: Währe das nicht der Fall bliebe als Ursache [Anm. von mir: für Homosexualität] höchstens noch der Wille Gottes als Grund übrig und das ist doch recht unwahrscheinlich. :). Das unterscheidet die IP von uns gewöhnlichen Sterblichen und stellt ihn auf eine Stufe mit anderen erleuchteten Menschen in Afghanistan, Pakistan, Saudi-Arabien, und nicht zuletzt im US-Bible Belt. Manche sagen, dass es schwere Gotteslästerung wäre, anderen Menschen seine eigenen Vorstellungen unter der Behauptung "Gott will es so" aufzudrängen/-zwingen. Und diese Leute haben ganz Recht. Solaris3 (Diskussion) 01:42, 4. Jan. 2014 (CET)
Niemand sprach von wollen. Es ging nur darum ob es medizinisch möglich wäre oder nicht. Wenn es das vermutete Hormonungleichgewicht während der Schwangerschaft ist, wäre es ein Wunder wenn man das nicht irgendwann frühzeitig erkennen und mit einer Hormontherapie gegensteuern könnte. Da Homosexuelle eine mehrfach höhere Suizidversuchsrate als Heterosexuelle haben, könnte dann auch durchaus darauf hindeuten das nicht alle Homosexuellen automatisch mit Ihrer Homosexualität glücklich sind. Männer können sich heutzutage entscheiden ob Sie nicht lieber eine Frau sein wollen und die Gesellschaft akzeptiert das. Warum sollte man es nicht akzeptieren, wenn sich Homosexuelle entscheiden doch lieber Heterosexuell sein zu wollen, wenn es denn ein Mittel gäbe um das zu bewerkstelligen? @Solaris3: Angehängte Smilies deuten für gewöhnlich auf einen vorangegangenen Scherz hin. Das weißt du aber auch. --85.181.211.239 01:46, 4. Jan. 2014 (CET)
@Solaris3: Als Atheist kann ich ihn auch nicht verstehen.
@IP Zitat: "Wenn es das vermutete Hormonungleichgewicht während der Schwangerschaft ist, wäre es ein Wunder wenn man das nicht irgendwann frühzeitig erkennen und mit einer Hormontherapie gegensteuern könnte." Nun sollen also andere (Eltern?) entscheiden ob ein Kind sich zu einem Homosexuellen entwickelt? HILFE.... --Stefan1973HB Disk. 02:01, 4. Jan. 2014 (CET)
Na dann biete mal die Früherkennung und Therapie an und du wirst feststellen wie viele Eltern deine Meinung teilen. --85.181.211.239 02:09, 4. Jan. 2014 (CET)
@IP: Ich möchte eigentlich nicht mit dir diskutieren, ich versuche nur mal dir ein paar Anstöße zu geben: 1. Wenn wissenschaftlich eindeutig erwiesen ist, dass homosexuelles Verhalten nicht nur beim Menschen, sondern im gesamten Tierreich sehr weit verbreitet ist (siehe oben verlinkte WP-Artikel), hast du dann vielleicht ein falsches Verständnis von Gottes Plan? Das nun zu erwartende Argument wie etwa "Die Abneigung gegen Homosexuelle/Gefühl der Unnatürlichkeit etc ist dem Menschen angeboren" ist nachweislich schlicht falsch. Viele Kulturen hatten/haben nicht das geringste Problem damit, zB die alten Griechen oder diverse Indianervölker. Deine (und die deiner gerade zitierten 'besorgten Eltern') offensichtliche Vorstellung davon als etwas "Minderwertiges" gehen, vereinfacht gesagt, im Kern auf das alte Testament zurück (siehe Artikel zur HS). Das ist ein von Menschen geschriebener Text und hat denkbar wenig mit "Gottes Wort" zu tun. Vielleicht ist angesichts von Punkt 1. oben also einfach Unsinn, was da über HS drinsteht, abgesehen davon dass du es mit Sicherheit nicht im Original gelesen oder die Richtigkeit der Übersetzung geprüft hast. Und dass Homosexuelle eine höhere Selbstmordrate hätten (wo? und wann gemessen? Quellen?), könnte -falls korrekt- daran liegen, dass sie ihr Leben lang latent deprimiert davon sind, von Leuten wie dir als "besser im Mutterleib durch Impfung im Keim verhindert" angesehen werden (und 'angesehen' kann man da leider ganz wörtlich nehmen), denn deine Einstellung ist auch noch keineswegs ausgestorben und war vor nicht langer Zeit noch recht verbreitet. Vielleicht lässt du dir das mal durch den Kopf gehen. Du bist in der Hinsicht also möglicherweise nur ein Sammelsurium von angesammelten und als 'gottgegeben' angenommenen haltlosen, in einem Text der Bronzezeit von unbekannten Autoren aus unbekannten Beweggründen festgelegten Vorurteilen, die Gottes Plan (siehe 1.) entgegenstehen. Nur so als Denkanstoß. Solaris3 (Diskussion) 02:29, 4. Jan. 2014 (CET)
Haltet doch mal bitte die zwei Fragen auseinander: 1.) Ist eine ‚Therapie‘ im Sinne einer dauerhaften Heterosexualisierung möglich? und 2.) Ist sie sinnvoll und erwünscht? Soweit ich das sehe, hat sich die IP nur zu 1.) geäußert, aber Ihr tut so, als habe er 2.) beantwortet. Wenn Ihr die beiden Fragen unterscheidet, müßt Ihr Euch auch nicht so künstlich aufregen.
Bezüglich der Griechen muß ich Dich übrigens leider enttäuschen, Solaris: mit Homosexualität in unserem Sinne hätten die meistens wahrscheinlich sehr wohl ein Problem gehabt. Das Problem an der Rückprojektion dieser Frage ist, daß es den Lebensentwurf ‚Homosexualität‘, also als als ausschließlich und unabänderlich angesehene Disposition und mit Beziehungsformen, die nach dem Vorbild der eheähnlichen heterosexuellen Beziehung zwingend Liebe und Sex (oder auch Sex ohne Liebe) beinhalten, erst seit gut hundert Jahren gibt. Das kannten die Griechen so nicht (und die Indianer wahrscheinlich auch nicht, aber da kenne ich mich nicht aus), und darum ist es recht sinnlos zu fragen, ob sie ein Problem damit gehabt hätten. Wahrscheinlich aber schon, denn zumindest von einem Freien wurde in den meisten Poleis erwartet, daß er heiratete und Nachwuchs zeugte, um einem Hausstand vorstehen und ihn unterhalten zu können. Was daneben passieren konnte, das beantworteten die Griechen anders als wir -- allerdings wurden die griechischen Frauen auch nicht nach ihrer Meinung dazu gefragt. ;-) Homosexuelle Penetration betrachteten die Griechen als entehrend, siehe auch Rettichstrafe. --Hermine Tuzzi (Diskussion) 08:59, 4. Jan. 2014 (CET)

Auch wenn ich die Aussagen der IP inhaltlich ablehne, bin ich doch ein wenig erschrocken, wieso hier in epischer Breite so diskutiert wird, als hätte er religiös/mit Gottes Wille argumentiert, wenn das gar nicht der Fall ist. *verwirrt* -- HilberTraum (Diskussion) 09:42, 4. Jan. 2014 (CET)

@Hermine, zu den alten Griechen sollte vielleicht noch dazugesagt werden, dass Liebe von Mann zu Mann gang und gäbe war, Analverkehr aber verpönt war. Den H-Begriff hab ich jetzt extra nicht verwendet, da er doch erst 1869 erfunden wurde. Dieses Jahr feiert die H. erst ihren 145. Geburtstag. Verhalten, das heute als h. bezeichnet wird, gibt es jedoch schon laaange, siehe altes Testament. Alle alttestamentlich motivierten Homophobiker seien daran erinnert, dass im 1. und 2. Buch Samuel eine schwule Liebesgeschichte steht. --Rôtkæppchen68 10:27, 4. Jan. 2014 (CET)
*quetsch* Meine Rede -- Du bringst es besser auf den Punkt als ich. Bis auf die Bücher Samuel, da ist es m.E. Auslegungssache, aber nicht ganz von der Hand zu weisen. Es muß aber festgehalten werden, daß in der Geschichte gleichgeschlechtliche Liebe und homosexueller Geschlechtsverkehr in der Bewertung zumeist auseinandergehalten wurden -- auch wenn es die Kombination in der Lebenspraxis schon immer gegeben haben dürfte. --Hermine Tuzzi (Diskussion) 11:27, 4. Jan. 2014 (CET)
Das hier ist übrigens der Grund warum sich aktuell kaum jemand des Themas aus medizinischer Sicht annimmt. Wer das auch nur anspricht ist automatisch ein Homophobiker, alle distanzieren sich von dieser "Meinung" und dass er ein religiös Verwirrter ist, ist allen Beteiligten schon mal klar. Auch wenn absolut gar nichts homophobes gesagt wurde. :) Es ist aktuell eine seltsame Stimmung in Deutschland in der einige Themen einfach nicht mehr angesprochen werden dürft und sollte das doch jemand tun wird sofort eine Tonne unreflektierter Argumentationsmüll gepaart mit persönlichen Anfeindungen darüber gekippt. Meist von Leuten die sich für ganz besonders liberal und aufgeklärt halten. Man ist nicht aufgeklärt nur weil man ganz ganz viel Texte gelesen hat, welche die eigene Meinung stützen und jegliche andere Argumentation sofort mit KRANK! oder HILFE! :) abtut. Menschen die nicht nur keine andere Meinung außer eigenen akzeptieren, sondern sich schlicht angewidert weigern diese auch nur anzuhören oder darüber nachzudenken nennt man normalerweise rückwärts gewandte Reaktionäre. Die Art der vertretene Meinung ist dabei vollkommen unerheblich. Die andere mögliche Interpretationsmöglichkeit für Menschen die glauben das Sie die allseelig machende Weisheit besitzen und dass jeder der Ihren Glauben anzweifelt ein Ketzer sei den man nicht zuhören muss und der für seine Meinung bestraft werden muss ... nun die nennt man religiöse Fundamentalisten. Also ein Rat an die obigen "Diskutanten". Das nächste mal wenn euch das tiefe Gefühl der Abscheu übermannt weil jemand eine ganz banale Sache anspricht die aber vage eure Meinung ankratz, lest mal im Anschluß eure Antworten und denkt darüber nach ob Ihr nur ganz normale Reaktionäre seid oder Eure Meinung bereits in einen religiösen Glauben übergegangen ist der jeglichen Kritiker automatisch zum Ketzer macht. --85.181.211.239 11:03, 4. Jan. 2014 (CET)
@Rotkaeppchen viel weiter oben, re Altmaier: Klar, der "eingefleischte Junggeselle" war lange ein Euphemismus für Homosexualität. Aber wenn wir jetzt, gottlob, Hetero-, Homo- und Bisexualität akzeptieren können, was hindert uns daran, ebenfalls zu akzeptieren, wenn sich jemand zur Asexualität bekennt? Grüße Dumbox (Diskussion) 11:59, 4. Jan. 2014 (CET)
Ich finde es recht amüsant, dass ich hier als mich 'künstlich Aufregender' dargestellt werde. Ja, ich habe mich aufgeregt, zugegeben. Aber oben mal locker zu sagen, der Fragesteller "hätte doch nur nach der Möglichkeit einer Therapie gefragt" und keine Wertungen abgegeben, ist schon lustig, wenn man sich den Diskuverlauf ansieht. Der Frager hat nach und nach ein ganzes Bündel an wertenden Aussagen zum Fragegegenstand nachgeschoben, die die Ursprungsfrage auch keineswegs 'mal eben neutral gefragt' erscheinen lassen. Ich ziehe mal einen bewusst schiefen Vergleich, aber gerade deswegen ist er besonders hübsch. Wie würdet ihr das sehen, wenn er gefragt hätte: "Gibt es eine Möglichkeit, Schwarze zu Weißen umzutherapieren?" - gefolgt von Statements wie (wörtlich, homosexuell durch schwarz/weiß ersetzt) "Währe das nicht der Fall bliebe als Ursache [Anm. von mir: für Schwarz-Sein] höchstens noch der Wille Gottes als Grund übrig und das ist doch recht unwahrscheinlich. :) " und "Warum sollte man es nicht akzeptieren, wenn sich Schwarze entscheiden doch lieber weiß sein zu wollen, wenn es denn ein Mittel gäbe um das zu bewerkstelligen?" und "Wenn es das vermutete Hormonungleichgewicht während der Schwangerschaft ist, wäre es ein Wunder wenn man das nicht irgendwann frühzeitig erkennen und mit einer Hormontherapie gegensteuern könnte." und dann schließlich abschließend "Das hier ist übrigens der Grund warum sich aktuell kaum jemand des Themas aus medizinischer Sicht annimmt. Wer das auch nur anspricht ist automatisch ein Schwarzenfeind, alle distanzieren sich von dieser "Meinung" und dass er ein religiös Verwirrter ist, ist allen Beteiligten schon mal klar." Klingt das alles massiv absurd? Ja. Noch schöner wird es, wenn man "Schwarzer" durch andere Minderheiten ersetzt, zum Beispiel "Jude". Letztlich geht es um den Umgang mit Devianz und das Thema "Leben und leben lassen", und da hat die IP angesichts ihrer diversen Äußerungen, eben gerade zur deutlich ausgesprochenen Wertung einer aus seiner Sicht scheints höchst wünschenswerten Umpolung Homosexueller bzw deren 'eugenischer Vermeidung' (sic), ein paar größere Defizite. Mal abgesehen von dem oben angesprochenen Aspekt, dass diese strikte Trennung "hetero/homo" vor der Realität der Buntheit menschlicher sexueller Empfindungen einfach im Ansatz falsch ist. @Hermine ein hübsches Zitat dazu: Tatsächlich wirft die Bekanntschaft mit der Literatur der Antike ein äußerst verblüffendes Problem für den Geisteswissenschaftler auf, das den meisten Personen, die unvertraut mit den Klassikern sind, nicht in den Sinn käme: ob die Dichotomie, die durch die Termini »homosexuell« und »heterosexuell« unterstellt wird, überhaupt mit irgendeiner Realität korrespondiert. […] Das Bewusstsein über Gründe der Unterscheidung folgt auf das Verlangen zu unterscheiden. Die Frage, wer »schwarz«, »farbig« oder »Mulatte« ist, beunruhigt nur Gesellschaften, die von rassistischen Vorurteilen beeinträchtigt sind […]. In der antiken Welt kümmerten sich so wenige Menschen darum, ihre Zeitgenossen auf der Basis des Geschlechts zu kategorisieren, zu dem sie sich erotisch hingezogen fühlten, dass keine Dichotomie gebräuchlich war, um diese Unterscheidung auszudrücken.(John Boswell): Christianity, Social Tolerance, and Homosexuality. Chicago/ London 1980, S. 58 f. So, mehr muss ich dazu nicht mehr sagen. Solaris3 (Diskussion) 13:34, 4. Jan. 2014 (CET)
Das war mit "unreflektierter Argumentationsmüll" gemeint. Praktisch nichts von dem was du ansprichst hat überhaupt irgendjemand gefragt oder in Frage gestellt. PS: Ich bin gar nicht der ursprüngliche Fragesteller. --85.181.211.239 13:52, 4. Jan. 2014 (CET)
SCNR: Jetzt wo du „homosexuell“ durch „schwarz“ ersetzt hast, klingen die Statements wie Auszüge aus Micheal Jacksons Tagebuch :) -- HilberTraum (Diskussion) 13:55, 4. Jan. 2014 (CET)
Solaris, verwechselst Du die Fragesteller-IP 84.--- mit der IP 85.---? --Hermine Tuzzi (Diskussion) 14:13, 4. Jan. 2014 (CET)
(BK) Du scheinst ein grundlegend anderes Verständnis von der biologischen Determiniertheit von Hautfarbe, Religion (Judentum) und Sexualität zu haben als die meisten anderen Leute. Aus meiner Sicht vergleichst Du hier Unvergleichbares; das darfst Du auch gern, aber bitte stell das doch nicht als die Wahrheit dar, nach der alle andern sich zu richten hätten, und greife bitte nicht diejenigen, die tatsächlich oder vermeintlich anderer Meinung sind als Du selbst, in ihrer Würde an. Du (ja, Du!) läßt hier eine Kaskade von Vorurteilen raus, daß es nicht mehr schön ist. Laß das bitte sein!
Und Boswell stellt die Situation in der Antike aus meiner Sicht falsch dar. Mehr muß ich dazu nicht sagen. --Hermine Tuzzi (Diskussion) 14:24, 4. Jan. 2014 (CET)
Ach doch, eins noch, habe ich doch gerade im Artikel John Boswell diesen schönen Spiegel hiesiger Diskussionskultur gefunden: 1986 publizierte der damals ebenfalls an der Yale University tätige Richard B. Hays den Artikel ‚Relations Natural and Unnatural: A Response to John Boswell’s Exegesis of Romans I‘, in dem er John Boswells Auslegung von Römer 1 als Musterbeispiel von Eisegese bezeichnet und Boswells historische Rekonstruktion in Frage stellt. Boswell hat auf den Artikel nichts erwidert und soll sich geweigert haben, Hays danach auch nur zu grüßen. --Hermine Tuzzi (Diskussion) 14:31, 4. Jan. 2014 (CET)

An alle: ich, der Fragesteller, jetzt mit anderer IP, habe mit 85.... nichts zu tun. --93.205.22.16 14:22, 4. Jan. 2014 (CET)

Ja, ich habe die beiden IPs verwechselt. Mea culpa. Ich entschuldige mich bei dir in aller Form, Fragesteller-IP. Adressiert war also eigentlich IP 85.xxx, den Wechsel ganz am Anfang oben habe ich gestern Nacht fröhlich übersehen. Der Adressat war der falsche, macht jedoch die Argumente gegen IP 85.xxx nicht falsch. @Hermine: Ich schrieb bewusst "schiefer Vergleich", das macht ihn gerade besonders deutlich. Es ging mir wie beschrieben um den Oberbegriff Devianz und den Umgang damit. Dass dich der Vergleich anscheinend irgendwie unangenehm berührt hat, sehe ich als gutes Zeichen ;-) Aber das musst du nicht nachvollziehen können. Solaris3 (Diskussion) 14:34, 4. Jan. 2014 (CET)
Aus meiner Sicht läßt ein schiefer Vergleich eine Argumentation nicht besonders deutlich erscheinen, sondern – nun ja: schief. So etwas lasse ich ungern unwidersprochen stehen, mehr ist da nicht hinter. Irgendwelche Vermutungen über mich anzustellen verbitte ich mir; auch Vermutungen über andere Diskussionsteilnehmer berühren mich unangenehm, was Du bitte als weniger gutes Zeichen bezüglich der Aufnahme Deines Verhaltens hier sehen magst. Wir sind hier alle anwesend und antworten gern; man muß weder über mich noch über die IP 85.--- noch über sonstwen irgendwelche Vermutungen anstellen. --Hermine Tuzzi (Diskussion) 14:44, 4. Jan. 2014 (CET)
Ich verweise auf meinen obigen Absatz mit den Reaktionären und den Fundamentalisten die aufgrund Ihrer Betriebsblindheit nicht mal mehr mitbekommen (wollen) was eigentlich diskutiert wird, sowie den Absatz von Hermine in dem Sie dir erklärt das außer dir niemand über das Thema "wäre es wünschenswert" spricht. Das Thema war: "wäre es möglich". Ich sage dazu wertfrei: eventuell. Du sagst dazu gar nichts. Das hätten wir dann glaube ich ausführlich geklärt. Es sei denn du möchtest noch irgendjemand etwas frei ausgedachtes unterstellen und dann anschließend ein weitere einseitige Brandrede dagegen verfassen. :) --85.181.211.239 14:51, 4. Jan. 2014 (CET)
Ich schrieb bereits gaaanz weit oben, dass die erfragte Therapie wissenschaftlich (nach derzeitigem Wissensstand) Humbug ist, hinterlegt mit diversen Links, um die Antwort nicht elend lang zu machen. Der Grund ist vor allem, dass es nicht mal ansatzweise einen Konsens in der Wissenschaft gibt, was HS auslöst/begründet. Die Variante mit dem "Hormonspiegel während der Schwangerschaft", sprich endokrinologisch, ist nur eines von vielen Erklärungsmustern, wie man etwa am Ende des Abschnitts "Übersicht" in Homosexualität nachlesen kann. Und für etwas, von dem man (bzw die Wissenschaft) letztlich keine Ahnung hat woher es genau kommt, eine "Therapie" anzubieten, ist eben Humbug. Vielleicht ändert sich das mal, sieht aber derzeit nicht so aus. Und die Frage, ob es wertvoll und wünschenswert wäre, daran überhaupt zu forschen, dazu habe ich mich oben genug ausgelassen. Frag doch mal 100 Schwule, ob sie das sinnvoll fänden. Die Antwort wird ebenso ausfallen wie die Einschätzung zur Ex-Gay-Bewegung, nämlich ablehnend. Und alle anderen, ergo Nicht-'Betroffenen', haben kein Recht, ihre Vorurteile über die betroffene Menschengruppe in eine wie auch immer geartete 'Therapieform' zu gießen. Denn das führt direkt in eine ganz falsche Richtung, siehe Eugenik. Solaris3 (Diskussion) 15:03, 4. Jan. 2014 (CET)
Interwikilinks zu solchen Themen auf die deutsche Wikipedia sind komplett wertlos und POV. Halbwegs neutrale Information findest du auf der englischen Wikipedia. --85.180.191.117 15:33, 4. Jan. 2014 (CET)
Homosexualität hat sicher einen evolutionären Vorteil, sonst hätte sie sich nicht entwickelt. Worin der Vorteil jetzt genau liegt, darüber kann man IMHO nur spekulieren. Vielleicht ist es gut für den Nachwuchs, wenn sich außer den Eltern noch der schwule Onkel darum kümmert. Wer weiß…? --Rôtkæppchen68 15:38, 4. Jan. 2014 (CET)
Vielleicht kommt das sogar hin. Es ist allerdings auch möglich das die genetische Vielfalt erhalten werden soll indem Frauen nicht zu viele Kinder bekommen sollen die sich wiederum weiter vermehren. Mit jedem zusätzlichen Kind steigt die Wahrscheinlichkeit, dass es homosexuell wird. Es gibt vergleichsweise deutlich mehr Drittgeborene die homosexuell wurden als Erstgeborene. --85.180.191.117 15:52, 4. Jan. 2014 (CET)
@IP 85.x: Mit dem obigen liegst du mE schon richtiger. Da du den von mir angeführten deutschen WP-Artikel angezweifelt hast - der englische Artikel sagt das gleiche, nämlich dass keinerlei Einigkeit über die Ursachen besteht: There is no consensus among scientists about why a person develops a particular sexual orientation. Genannt werden als derzeit bevorzugte, also nicht eindeutig als korrekt identifizierte und fundamental unterschiedliche Erklärungsmuster: genetic factors, the early uterine environment, or both in combination. Also die Hormontheorie (bzw richtiger: die Summe der "Umweltbedingungen" im Mutterleib), oder auch genetische Faktoren, oder eine Kombination von beiden. Zu deutsch: man weiß es nicht. Geht man von der genetischen Erklärung aus, würde eine pränatale Therapie wohl in das Feld des Rumbastelns am Embryo führen, von der Therapie am Erwachsenen ganz zu schweigen - und das Ganze auf vagen Verdacht. Das meinte ich eingangs umschrieben mit "Humbug". Solaris3 (Diskussion) 18:17, 4. Jan. 2014 (CET) PS: Nähme man willkürlich die "genetischen Faktoren" als Begründung, erscheint mein oben als gar furchtbar unpassend und irgendwie seltsam dargestellter Vergleich mit der Verwandlung/"Therapie" von Menschen dunkler Hautfarbe in Weiße gar nicht mehr so abwegig, sondern genau passend. Aber das nur am Rande ;-) Solaris3 (Diskussion) 18:58, 4. Jan. 2014 (CET)
Im Nachhinein sehe ich diesen, von meiner Seite etwas ausgeuferten Thread als interessantes psychologisches Experiment. Ich hatte die Frage zunächst völlig neutral beantwortet, aufgeregt habe ich mich bei den dann folgenden mehr als uninformierten Beiträgen von IP 85.x, wobei ich diesen leider irrtümlich lange mit dem Fragesteller IP 84.x gleichgesetzt habe (ich hielt nicht für möglich, dass derartige Ansichten in so schneller Folge kommen...). Daher, fürs Archiv und die wenigen verbleibenden Leser, noch ein paar Hintergrundinfos zu dieser Frage: Man kann die Frage der Möglichkeit des "Umpolens" von Homosexuellen zwar (imho gerade noch) wertfrei diskutieren, aber sobald dann chauvinistische, d.h. abwertende Äußerungen wie oben dazukommen, bekommt das Ganze einen sehr üblen Beigeschmack. Thesen wie "Homosexualität kann wohl nicht auf Gottes Wille zurückgehen" (Smiley danach hin oder her) zusammen mit "Eltern würden sich bestimmt entscheiden, Kinder im Mutterleib umzupolen" plus "Schwule haben ja eine höhere Selbstmordrate, und deswegen wäre es legitim, ihre Zahl zu ihrem eigenen Wohl zu redzieren (alles von oben, teils leicht (!) verdeutlicht) etc. deuten einfach auf eine anti-egalitäre, diskriminierende Denkweise hin. Sie basieren auf halbgaren Vorurteilen (wenn auch weit verbreitet) und sind im Grunde menschenverachtend. Jeder kann glauben was er will, aber nicht sachlich falsche Vorurteile in so einem Wissensform hier als Wahrheit verbreiten, da hörts bei mir auf - vor allem wenn auf Argumente dann weitere Vorurteile kommen oder blanke Behauptungen, dass die deutschen WP-Artikel allesamt "schwulenfreundlicher POV" wären (siehe oben). Auch wenn in Deutschland momentan "offiziell" eine HS-freundliche/tolerante Stimmung vorherrscht, ist dies keineswegs durchweg gesellschaftliche Realität, sondern alle Schwulen/andersartig sexuell 'Abweichenden' haben auch hier nach wie vor mit häufigen Diskriminierungen zu tun - warum hat sich denn etwa noch kein einziger (der vorhandenen schwulen) Spitzenfußballer geoutet? In anderen Ländern gibt es sogar ein Rollback, siehe Russland, siehe arabische Länder. Schwule sind dort konkret jeden Tag mit dem Tod bedroht. In Uganda steht neuerdings die Todesstrafe auf Homosexualität, was vor allem durch das langjährige Wirken US-amerikanischer, evangelikaler 'Christen' der Ex-Gay-Bewegung zurückzuführen ist. Selbige haben in den USA großen Einfluss, wodurch dort etwa hunderttausende homosexuelle Teenager schwersten Erpressungen/Psychoterror durch ihr Umfeld ausgesetzt sind - und zwar in die Richtung, dass sie doch zu einer Therapie gehen sollten, um "normal" zu werden. Dies hat nun eine thematische Ähnlichkeit mit der Ausgangsfrage. Allein die ernsthafte Frage nach der Möglichkeit beinhaltet implizit die Möglichkeit, dies wäre eine valide Option, was dann im Verlauf der Diskussion von einem anderen Teilnehmer wiederholt durch entsprechende Äußerungen unterstrichen wurde. Und die dahinterliegende Motivation ist die gleiche wie bei allen homophoben Äußerungen/Handlungen, auch den eben geschilderten in USA, siehe etwa unter Coming-Out unter "Überwindung von Vorurteilen und Akzeptanz... Aus diesem Grund bin ich oben so ausgerastet - lustig fand ich, dass die eine in einem fort reaktionärste Äußerungen verbreitende IP mit dem Argument der Meinungsfreiheit mir dann vorwarf, "reaktionär" aufzutreten. Letzteres kann man definitionsgemäß nur, wenn man rückwärtsgewandte Positionen vertritt. Wer gesellschaftlichen Gruppen ihren Wert abspricht, indem er dafür eintritt, sie sollten am besten bereits im Mutterleib "verändert" werden, darf sich angesichts dieses menschenverachtenden Gedankenguts (sic) nicht darüber beschweren, wenn er mal verbal auf die Mütze bekommt. Bei allen anderen, die ich in meinem Furor teils möglicherweise unberechtigt angegangen bin, möchte ich mich jedoch nachträglich entschuldigen, insbesondere beim Fragesteller IP 84.x. Nochmal zusammenfassend: Nein, eine medikamentöse "Therapie" von Homosexualiät ist nach derzeitigem Stand der Wissenschaft nicht möglich und auch in weiter Ferne. Zudem ist sich die Wissenschaft einig (bis auf sehr wenige Exoten), dass HS auch nicht therapiert werden sollte, weil sie aus zahlreichen Gründen mittlerweile als gleichwertig zu Heterosexualität angesehen wird. Details siehe oben in den verlinkten Artikeln, die englischen Artikel sind übrigens im Gegensatz zu wilden Behauptungen weiter oben im wesentlichen inhaltsgleich zu den angeblich so "schwulenfreundlich-povigen" deutschen Artikeln. Solaris3 (Diskussion) 18:40, 5. Jan. 2014 (CET)
Legst Du Deinen Freunden, Kollegen und Anverwandten auch Haßreden in den Mund, um daraufhin Dein Mütchen an ihnen zu kühlen, oder machst Du das nur gegenüber Fremden, anonym und im Internet? --Hermine Tuzzi (Diskussion) 19:14, 5. Jan. 2014 (CET)
Ich habe keine Hassreden "in den Mund gelegt", sondern zitiert und die Aussagen kommentiert. Wenn jemand wie IP 85 oben hübsche Vorurteile wiederholt verbreitet, jegliche Gegenargumente ignoriert und überhaupt auf jede faktische Untermauerung seiner Behauptungen verzichtet, muss er sich das schon gefallenlassen. Habe ich den Sinn verfälscht? Ich meine nein. Wenn du mir das Gegenteil nachweist, ändere ich es gerne. Im Übrigen kannst du dich glaub ich nicht beschweren, bei allen anderen außer IP 85 hab ich mich explizit entschuldigt, das schloss dich ein, und du wurdest afaik auch nicht direkt angegangen. Und von wegen "anonym" - würde IP 85 wohl auch unter Account hier so vom Leder ziehen? Solaris3 (Diskussion) 19:47, 5. Jan. 2014 (CET)
Die Hassreden kommen wohl eher von 85.180.x. Können wir den Thread jetzt bitte im Archiv verschwinden lassen, bevor er noch peinlicher wird? Frage ist IMHO längst beantwortet: Homosexualität ist weder eine Krankheit, noch ist sie einer Therapie zugänglich, geschweige denn therapierbar, auch wenn manche Leute das Gegenteil behaupten oder sich wünschen. Sogar die Ex-Gay-Bewegung (Exodus International) hat eingesehen, dass es sinnlos ist, Homosexualität therapieren zu wollen. --Rôtkæppchen68 22:00, 5. Jan. 2014 (CET)
Archivierung dieses Abschnittes wurde gewünscht von: Solaris3 (Diskussion) 18:18, 6. Jan. 2014 (CET)

Franken ein Stamm?

Ein IP hat eine Bearbeitung von mir im Artikel „Franken (Volksgruppe)“ geändert. Laut Bayerischem Landesportal sind die Franken (nicht die Germanen mit Karl dem Großen, sondern die Bewohner Frankens in Nordbayern) ein Stamm, einer von Bayerns "vier Stämmen". Der IP hat die Änderung begründet mit: "Noch größerer Unsinn. Die Franken sind kein Stamm. Und erst recht kein bayerischer. [...] Schwachsinnige CSU-Propaganda." Das hat mich stutzig gemacht. Daher meine Auskunft-Frage: Sind die Franken ein Stamm? Wären dann nicht auch die Brandenburger oder die Ostwestfalen ein Stamm? --CG (Diskussion) 12:30, 3. Jan. 2014 (CET)

Ach ja, da in der Löschdiskussion und bei der Qualitätssicherung dies unklar erschien: Mit Franken ist nicht das germanische Volk gemeint, aus denen sich vor tausend Jahren u.a. Erzgebirgler und Luxemburger entwickelten und die das Fränkische Reich mit Karl dem Großen gegründet haben, sondern die Bewohner Frankens.--CG (Diskussion) 12:31, 3. Jan. 2014 (CET)

"Weder in Bezug auf eine evolutionäre Entwicklung vom einen zum anderen, noch als ein gleichzeitig parallel zum Staat existierendes globales Phänomen wird der Stammesbegriff heute noch ernsthaft diskutiert. Als allgemeine politisch-gesellschaftliche Kategorie wird die Bezeichnung Stamm als vorurteilsbehaftet, wertend und analytisch unscharf abgelehnt. Dagegen besitzt in der Ethnologie eine Definition von Stamm, die sich an der Selbstidentifikation sowie der kulturellen, religiösen und ethnischen Identität der jeweiligen sozialen Gruppe orientiert, weiterhin Bedeutung." (Wikipedia: Volksstamm). Eher folkloristisch, eher nicht enzyklopädisch, die Sache mit dem Stamm. Grüße Dumbox (Diskussion) 12:46, 3. Jan. 2014 (CET)
Volk oder Bevölkerungsgruppen kann man aber jetzt auch nicht sagen, denn laut Wikipedia: "Versuche, Menschen von außen auf ihre Zugehörigkeit zu einem „Volk“ im ethnischen Sinn amtlich festzulegen, werden im Zuge der Anerkennung nationaler Minderheiten heute zumeist abgewiesen." Ich würde es so versuchen. Franken ist die Selbstbezeichnung von Menschinnen und Menschen die temporär, vage mittig der aktuell als Europa bekannten Landmasse rumlungerten. Dann weiß man zwar nicht mehr um wen es geht aber wenigstens fühlt sich keiner diskriminiert. (Witzig das man inzwischen bei den einfachsten Begriffen auf die englische Wikipedia ausweichen muss, weil in der deutschen nur noch um die Gebräuchlichkeit des Begriffes unter Berücksichtigung der Diskriminierung (von wem auch immer) rumgeeiert wird, anstatt einfach den Begriff zu erklären) --92.228.42.137 13:51, 3. Jan. 2014 (CET)
Die Junktur Deutsche Stämme ist in der Geschichtswissenschaft zwar nicht mehr so zentral, aber noch in Gebrauch; die Ausdehnung der Stammesgebiete, wenn es um Folklore (auch ein Teilbereich des Lebens!) geht, deckt sich in etwa mit den alten Reichskreisen. Besonders in Bayern ist die Rede von den „vier bayerischen Stämmen“ sehr verbreitet und sollte im Artikel nicht fehlen. Man kann das ja auch referieren, ohne es sich gleich zueigen zu machen. Der Stammesbegriff, von dem Du sprichst, Dumbox, bezieht sich wahrscheinlich auf ‚exotische‘ (darf man nicht mehr sagen) ‚Stämme‘ (darf man auch nicht mehr sagen), denen diese Bezeichnung von uns Europäern (darf man das noch sagen?) beigelegt wurde. Die Ethnologie (Völkerkunde darf man bei manchen auch nicht mehr sagen) ist generell ein vermintes Gebiet, was Sprache betrifft -- ich erinnere nur an Eingeborene (darf man nicht mehr sagen), Eskimo (darf man wieder sagen), Zigeuner (manche sagen ja, manche nein), Häuptling (geht gar nicht) usw.; eigentlich wäre da mal eine In-&-out-Liste sinnvoll, die man als RSS-Feed abonnieren könnte, damit man in ethnologicis weiß, was man zum gegebenen Zeitpunkt überhaupt sagen darf, ohne sich unmöglich zu machen. --Hermine Tuzzi (Diskussion) 14:29, 3. Jan. 2014 (CET)
nicht enzyklopädetisch, aber dafür mit Quelle: http://cdn-storage.br.de/MUJIuUOVBwQIbtChb6OHu7ODifWH_-b6/_AJS/_y865yFd/131220_0900_BAYERN-3-am-Vormittag_Happy-End.mp3 die Franken machen den derzeitigen MP wahnsinnig, also weiter so --93.205.226.206 14:37, 3. Jan. 2014 (CET)
Also nehmen wir mal mich. Altstämmisch bin ich selbstverständlich Franke, worauf auch der Name einer Großstadt mit vielen Banken in meiner Nähe noch deutlich hinweist. Die "Franken" aber würden mich, so zerstritten sie untereinander (Oberfranken, Unterfranken, Maabrunzer) auch sind, einhellig nicht als einen der ihren ansehen, sondern als Hessen. Hesse bin ich natürlich auch, insofern ich so rede wie Babba Hesselbach und die Rodgau Monotones. Über lange Phasen der Geschichte jedoch verbindet mich und mein Kaff, heute durchaus noch spürbar, viel mehr mit Kurmainz, also Mainz, Rheinland-Pfalz, im Gegensatz zu der Stadt, die mein Kaff eingemeindet hat - zu der ich aber auch eine gewisse Affinität empfinde, wenn es um ihren (nur temporär viertklassigen) Fußballverein geht. Schwierig, das mit der Stammeszugehörigkeit. ;) Im Ernst aber: Die verlinkte Seite mit ihren holzhammerhaften Klischees von Stammeseigenschaften ist einfach absurd. Grüße Dumbox (Diskussion) 16:04, 3. Jan. 2014 (CET)
Es gibt keine einheitliche Definition dieses Begriffes, aber darum geht es in der Frage nicht. Die "bayerischen Stämme" sind eine Art Redewendung, die sich auf die Regionen Schwaben, Franken und Altbayern, die das heutige Bundesland Bayern ausmachen (manchmal werden die Sudetendeutschen, die sich nach 1945 in Bayern angesiedelt haben als "vierter Stamm" betrachtet. Wenn man die Franken als "Stamm" bezeichnen will, muss man diese Redewendung erwähnt werden. Ein Norddeutscher kennt das nicht.--Antemister (Diskussion) 17:16, 3. Jan. 2014 (CET)
In der Nazizeit wurde der Begriff "Stamm" als Teilmenge des Deutschen Volkes weithin verwendet (was nicht bedeutet, dass es sich um einen originären Nazi-Begriff handelt). Damit ist der Begriff wohl "verbrannt" und wurde deshalb gelöscht. Wollte man ihn heute wieder verwendungsfähig machen, bräuchte man zumindest eine einigermaßen gut definierte Begriffsbestimmung. --84.135.132.143 18:22, 3. Jan. 2014 (CET)
Entschuldige, aber daß eine auch in der Gegenwart weithin verwendete Bezeichnung durch die bloße Verwendung in der Nazizeit „verbrannt“ sein soll, ist eine absolut bescheuerte Begründung dafür, sie in der Wikipedia zu löschen. Die „Verwendungsfähigkeit“ (Was ist das?) des Wortes ist durch seine bloße Verwendung ja wohl hinreichend bewiesen. --Hermine Tuzzi (Diskussion) 20:16, 3. Jan. 2014 (CET)
Ich würde mich durchaus genieren ein Wort wie "Deutschtum" oder "Endlösung" zu gebrachen, obwohl es eigentlich harmlose Wörter sind. Der Bedeutungsinhalt, den die Nazis ihnen gaben, klebt nun mal fest dran. Sowas würde ich auch löschen. So "bescheuert" ist das gar nicht. Es ist natürlich Ansichtssache, wie weit mal dabei gehen soll. --84.135.139.254 22:35, 3. Jan. 2014 (CET)
In der Bayernhymne wurden die Bruderstämme jedenfalls afaik nicht gestrichen (aber ich bin nicht sicher ob die Franken da dazu zählen). --Ailura (Diskussion) 22:51, 3. Jan. 2014 (CET)
Absolute Zustimmung zu Hermine. Viele Leute sind sich nicht bewusst das die Nazis insgesamt 12 Jahre an der Macht waren. Wenn die eins wirklich gern getan haben, dann war es Reden halten. Es gibt ein mehr tausend Seiten starkes Geschichtswerk alleine über Hitlers Reden. Wenn man sich ernsthaft darüber Gedanken macht welche deutschen Worte jetzt durch die Nazis "verbrannt" wurden und welche nicht ... Die Antwort ist alle. Also sollte man sich sprachlich bei banalen Begriffen wie Stamm wirklich nicht damit einschränken indem man prüft ob irgendwann mal ein Nazi das Wort missbraucht hat. Ganz sicher hat das einer, die haben alle Worte missbraucht. Mit der Prüfung "Hat das mal ein Nazi gesagt oder getan" kann man alles in Grund und Boden reden. Kürzlich las ich einen sehr schlüssigen Artikel darüber, dass die heutigen Bio Bauern genaugenommen die Nachfahren der Bewegung zur gesunden Volksernährung aus der Nazizeit seien. :) Darauf im Jahre 2014 hinzuweisen ist nur genauso blödsinnig wie zu prüfen ob das Wort "Stamm" jetzt irgendwie nationalsozialistisch angehaucht sei. --85.181.211.239 11:30, 4. Jan. 2014 (CET)
Das Wort "Stamm" ist - "verbrannt" oder nicht - natürlich nicht zur Verwendung verboten. Nur wenn man es verwendet, sollte es irgendeinen greifbaren Sinn haben, d.h. was sagt es eigentlich aus, wenn ich von deutschen, analog auch von englischen , französischen, italienischen, niederländischen usw. "Stämme" rede? In der Eingangsfrage ging es nicht einmal um deutsche, sondern um bayrische Stämme, was bedeutet, dass nicht die Bayern, Hessen, Niedersachsen usw. die Stämme sind, sondern jeweils irgendwie aussortierte Teilbereiche derselben. Das müsste doch erst mal eine(r) plausibel erklären. --84.135.139.254 19:19, 4. Jan. 2014 (CET)
Nichts leichter als das: es handelt sich um drei Volksgruppen in Bayern, die sich in Siedlungsgebiet, Sprache und Traditionen unterscheiden und von einem gewissen historisch gewachsenen Zusammengehörigkeitsgefühl geprägt sind. --Hermine Tuzzi (Diskussion) 19:44, 4. Jan. 2014 (CET)

Suchmaschinen im Browser

Firefox hat dieses kleine Fenster rechts oben, wo schnell gewechselt werden kann zwischen verschieden Suchmaschinen (google, WP, commons, leo etc.) Gibt es so eine Einstellung auch für Google Chrome und Opera? Danke --Frze > Disk 04:35, 4. Jan. 2014 (CET)

Für Google Chrome siehe hier. --Rôtkæppchen68 08:02, 4. Jan. 2014 (CET)
Tutorials lesen! darin steht bei Opera, dass es Kürzeln für die Adressleiste gibt, und diese individuell erweitert werden können. [5] --80.108.60.158 09:22, 4. Jan. 2014 (CET)
Bei Opera 12 gibts das genau wie bei FF, kann über Rechtsclick auf die Adressleiste angepasst werden. Opera 15 hingegen ist ja kein Browser mehr und gehört daher nicht in diese Frage, bei Chrome brauchst du keine Suchfelder, weil Google eh schon weiss, wonach du suchen willst *SCNR* --92.202.90.81 17:42, 4. Jan. 2014 (CET)
Noch schneller, als einen entsprechenden Favorit / Lesezeichen / Verknüpfung / Link anklicken?! Solcher Schnickschnack verlangsamt doch insgesamt nur. --217.84.114.250 18:36, 4. Jan. 2014 (CET)

"Positive" Patientenverfügung

Kann man mit einer Patientenverfügung auch erklären, dass man möchte, dass man unter allen Umständen am Leben gehalten wird? --85.180.143.38 06:10, 4. Jan. 2014 (CET)

Warum denn nicht? Siehe auch Patientenverfügung und BGB § 1901a: ob er in bestimmte, zum Zeitpunkt der Festlegung noch nicht unmittelbar bevorstehende Untersuchungen ... , Heilbehandlungen oder ärztliche Eingriffe einwilligt oder sie untersagt (Patientenverfügung). Ferner Sterbehilfe und Medizinethik. --Vsop (Diskussion) 07:37, 4. Jan. 2014 (CET)
da wär ich aber ganz vorsichtig, weil schmerzmittelchen leicht mal lebensverkürzend sind... und wenn die ärzte n komischen tag haben, dann nehmen die die pat'enverfügung ernst... --Heimschützenzentrum (?) 08:18, 4. Jan. 2014 (CET)
in Österreich dient eine PatVerfügung ausdrücklich nur dazu, Behandlungen abzulehnen. Alles, was ansonsten medizinisch indiziert erscheint, wird ein Arzt versuchen, aber du kannst dir nicht darüber hinaus irgendwelche Sonderbehandlungen wünschen.--91.141.2.42 08:28, 4. Jan. 2014 (CET)
Hach ja, Deutschösterreich, du herrliches Land! Bei Euch ist die Welt in so vielem noch in Ordnung: Ärzte halten Patienten am Leben, Arbeitnehmer bekommen ein 14. Monatsgehalt... manchmal wünschte ich mir einen Anschluß der Bundesrepublik an Österreich... --77.12.105.212 08:39, 4. Jan. 2014 (CET)
*hust*Nazometer*hust*--88.67.153.104 12:06, 4. Jan. 2014 (CET)
Mit Nazivorwürfen schießen hier manche schneller als ihr Schatten... --Hermine Tuzzi (Diskussion) 13:12, 4. Jan. 2014 (CET)
Dient wahrscheinlich nur der Erfüllung von Godwin’s law und muß nicht weiter ernst genommen werden. --91.0.145.182 13:26, 4. Jan. 2014 (CET)
Die Ärzte werden sowieso alles tun, was finanziell vertretbar ist. Statt einer Patientenverfügung würde ich lieber eine private KV/Zusatzversicherung mit allem Pipapo, Chefarzt usw. abschließen. --91.0.145.182 11:24, 4. Jan. 2014 (CET)
Chefarzt? Bloß nicht. Maximal den Vizechefarzt, der steht wenigstens noch ein paar Stunden im OP und hat noch ein bisschen Einblick in die Praxis. Die Chefärzte, die ich bei den privatversicherten nahen Anverwandten erleben durfte, waren der Praxis so weit entrückt wie der Philosoph im Elfenbeinturm. Der Chefarzt mit seinem theoretischen Spezialwissen mag dann was helfen, wenn man an irgendeiner exotischen/seltenen Krankheit leidet - aber üblicherweise landet man wegen etwas ganz gewöhnlichem im Krankenhaus, was für den 08/15-Mediziner, der dort ständig an der "OP-Front" tätig ist, lässige Routine ist und aus dem effeff sitzt. Der Chefarzt dagegen steht vielleicht noch einmal die Woche oder einmal im Monat selbst im OP, damit er nicht ganz so abgehoben wirkt. Aber das ist nur ein Feigenblättchen.
Ja, sicher ist das jetzt etwas überspitzt und polemisierend dargestellt, aber ich denke, es ist klar, worauf ich raus will. -- 88.67.153.104 12:06, 4. Jan. 2014 (CET)
Nee, stimmt schon, Chefärzte machen fast nur noch Personal- und Verwaltungskrempel. Unter Umständen sind sie sogar an mehreren Kliniken Chefarzt und daher gar nicht da... --92.202.90.81 17:46, 4. Jan. 2014 (CET)
nur Amtsarztbehandlung ist lustiger (besonders die Visitenkarten (Postkartenformat und blass gelb mit braunen Streifen... jedenfalls in Goslar...))... Stationsärzte sind wohl auch lieber mit Verwaltungskrempel als mit Pat'en beschäftigt... --Heimschützenzentrum (?) 20:44, 4. Jan. 2014 (CET)

Zur Ausgangsfrage: Diese ist klar mit Ja zu beantworten. Das ist so ähnlich beim Organspendeausweis: man kann sich auch einen zulegen, um anzukreuzen, dass man Organspenden ablehnt. Ungeachtet von Willensbekundungen kann die Indikation zu medizinischen Maßnahmen kann jedoch nur vom Arzt gestellt werden.--Janden007 (Diskussion) 21:25, 4. Jan. 2014 (CET)

Geißelungskapelle in Jerusalem

Mag das mal jemand übersetzen?
Custodia Francescana Terrasanta = Kustodie des Heiligen Landes --Vsop (Diskussion) 12:24, 4. Jan. 2014 (CET)

Geißelungskapelle in Jerusalem Was bedeutet "Herzog Maximilian von Bayern (richtig wäre ... i n Bayern, denn er war nicht regierend) hat die Wiederherstellung ermöglicht ? Wiederaufgebaut wurde sie erst 1929, Herzog Max Joseph in Bayern verstarb aber bereits 1888 ! --84.153.40.118 08:52, 4. Jan. 2014 (CET)

Hm, als Quelle ist eine Website ein Baedeker angegeben, aber auch dort steht (noch eindeutiger): „Die heutige Kapelle […] ist eine Stiftung des Herzogs Maximilian von Bayern aus dem Jahr 1929“. Eine Stiftung kann natürlich deutlich vor der eigentlichen Bauumsetzung stattfinden, aber so lange? --elya (Diskussion) 09:47, 4. Jan. 2014 (CET)
Ergänzung: englische Wikipedia trennt die beiden Ereignisse. Mag jemand zur Erleuchtung mal die Gedenktafel für den bayrischen Max übersetzen? --elya (Diskussion) 10:31, 4. Jan. 2014 (CET)
Zeilenweise, daher etwas holprig:
Für Maximilian, Herzog von Bayern,
geboren am 4. Dezember 1808,
gestorben am 15. November 1888,
der diese heilige Stadt Jerusalem
vom 09. bis zum 15. Mai 1838
zur Zeit der Seuche
besuchte,
der,
dem gegeißelten Erlöser dankbaren Sinnes,
für die Erneuerung dieser Kapelle
zur Obhut des heiligen Grundes der Kustodie des Heiligen Landes
die Mittel zur Verfügung stellte,
(hat) in
Erinnerung an seine Tat
die katholische Bruderschaft der Bauleute,
aus dem Volk der Deutschen in Jerusalem versammelt,
ehrenhalber
(dies) errichtet
im Jahre 1900.
Grüße Dumbox (Diskussion) 10:59, 4. Jan. 2014 (CET)
(BK, Hmpf!) Maximilian, dem Herzog von Bayern / geb. den 4. Dezember 1808, / gest. den 15. November 1888, / der diese heilige Stadt Jerusalem / vom 9. bis zum 15. Mai 1838 / zur Zeit der Pest / besuchte, / der / aus Dankbarkeit gegenüber dem mit Geißeln geschlagenen Erlöser / um dieses Kapellchen wiederherzustellen / der Kustodie des Heiligen Landes / Mittel zur Verfügung stellte, / hat zur / Erinnerung an die Ausführung / die katholische Handwerkerzunft / die sich aus dem deutschen Volk in Jerusalem zusammenfand / zu Ehren / diese Tafel aufgehängt / im Jahre 1900. --Hermine Tuzzi (Diskussion) 11:13, 4. Jan. 2014 (CET) Diese Auskunftsseite ist nicht groß genug für uns beide. ε|;-)* --Hermine Tuzzi (Diskussion) 11:24, 4. Jan. 2014 (CET)
(BK) Ist doch schön, dass wir uns so einig sind! ;) Ja, an dem "custodiae" habe ich auch gezweifelt, mich dann aber für den dativus finalis entschieden. Ändert ja auch nicht viel. Grüße Dumbox (Diskussion) 11:25, 4. Jan. 2014 (CET)
Danke für den Hinweis, Vsop! Ich hab's in meine Übersetzung eingearbeitet. Die Geschichte des Heiligen Landes ist so interessant, und ich weiß so beschämend wenig darüber... :-( --Hermine Tuzzi (Diskussion) 12:39, 4. Jan. 2014 (CET)
Stell dir nur mal vor, wie unendlich öde es wäre, allwissend zu sein: Kein Forschen, kein Lernen, keine Überraschung, keine neuen Zusammenhänge erkennen, jede Pointe liegt schon vor - da stürzt man sich doch lieber gleich in das nächste Schwarze Loch... :-)) GEEZER… nil nisi bene 13:17, 4. Jan. 2014 (CET)
84.153.40.118 hat recht, der Link im Artikel Geißelungskapelle ist ohne Ver-Pipung treffender[6]. Danke für den Hinweis.--Der Harmlos (Diskussion) 20:50, 4. Jan. 2014 (CET)

Mit sicher nicht mehr, sondern eher weniger Ahnung als Hermine Tuzzi, habe ich mich trotzdem nicht von einer Überarbeitung/Ergänzung von Geißelungskapelle und https://commons.wikimedia.org/wiki/File:Iglesia_de_la_Flagelaci%C3%B3n_03.JPG abhalten lassen, die ich nun der kritischen Aufmerksamkeit der Interessierten empfehle. Über die Sodalitas catholica opificum habe ich bisher leider noch nichts herausfinden können. --Vsop (Diskussion) 13:48, 5. Jan. 2014 (CET)

Architekt gedenkstätte berlin

--178.27.62.112 16:28, 4. Jan. 2014 (CET)

Such Dir unter Kategorie:Denkmal in Berlin die richtige Gedenkstätte aus und lies den Artikel dazu. Mit etwas Glück findest Du dort auch den Architekten. Gruß, --DerFeigling (Diskussion) 16:38, 4. Jan. 2014 (CET)
Mein Tipp: Denkmal für die ermordeten Juden Europas, Peter Eisenman. --79.204.194.107 17:21, 4. Jan. 2014 (CET)

Dauermangnet abschirmen

Ich bin gerade dabei ein Perpetuum Mobile zu entwickeln. Ich brauche dazu nur noch ein Material, mit dem ich das Magnetfeld einens Dauermagneten effektiv abschirmen kann und das Material selbst darf nicht magnetisch sein.

Ich hab hier mal eine Illustration gemacht. http://s7.directupload.net/images/140104/xahej2ow.png

Das abschirmende Material rotiert unter dem Magneten im Kreis, so dass das Magnetfeld regelmäßig unterbrochen wird. Der Energieerhaltungssatz ist mir bekannt, ich prüfe ihn nur auf Richtigkeit.

Diverse PC-Speaker, magnetisch geschirmt und ungeschirmt, auch im Röhrenfernseher sind nur geschirmte Lautsprecher, da diese sonst Einfluss auf die Ablenkung des Elektronenstrahls nehmen würden und das Bild verfärben oder verzerren würden, bis zur Beschädigung der Loch- oder Schlitzmaske--Hans Haase (有问题吗) 11:42, 5. Jan. 2014 (CET)

--77.3.161.21 16:58, 4. Jan. 2014 (CET)

Mit einer geerdeten Keksdose aus Weißblech, siehe auch Permeabilität (Magnetismus). --Ohrnwuzler (Diskussion) 01:28, 5. Jan. 2014 (CET)
Mit Adamant geht das sicher. -- MonsieurRoi (Diskussion) 17:07, 4. Jan. 2014 (CET)
Unobtainium ist da noch besser! --Rubblesby (Diskussion) 17:10, 4. Jan. 2014 (CET)
Ich brauche ein reales Material. Was ist mit Blei? --77.3.161.21 17:11, 4. Jan. 2014 (CET)

(Trollerei entfernt. Rôtkæppchen68 11:29, 5. Jan. 2014 (CET))

Darf ich fragen was das mit meiner Frage zu tun hat? --77.3.161.21 17:42, 4. Jan. 2014 (CET)

(Trollerei entfernt. Rôtkæppchen68 11:29, 5. Jan. 2014 (CET))

Wer hat dir denn ans Bein gepinkelt? Das hier nennt sich "AUSKUNFT", nicht "Arena für Überheblichkeit". --77.3.161.21 18:51, 4. Jan. 2014 (CET)

(Trollerei entfernt. Rôtkæppchen68 11:29, 5. Jan. 2014 (CET))

Ärger Dich nicht, die können nichts dafür. Es hat nichts mit Dir zu tun. Sie sind nicht wirklich böse sondern fühlen sich nur klein oder mit privaten Problemen. Dann wirken sie unausgeglichen und beschimpfen sich gegenseitig. Es hat nichts mit Dir zu tun. 46.115.139.79 19:04, 4. Jan. 2014 (CET)
Sind "die" mehrere? --77.3.161.21 19:28, 4. Jan. 2014 (CET)
Na "die hier alle", die es betrifft. Keine Ahnung wie viele das abgezählt sind. Aber sie sind immer da, auch nachts und Weihnachten. 46.115.139.79 19:33, 4. Jan. 2014 (CET)
Es ist egal, was du nimmst: Um einen Leiter durch ein Magnetfeld durchzubewegen, brauchst du eine Kraft, die ziemlich genau dem entspricht, was du gewinnen könntest. Und Nichtleiter haben auf Magnetfelder überhaupt keinen Einfluss. Es ist zum Mäusemelken, es kann grundsätzlich nicht klappen --RobTorgel (Diskussion) 17:24, 4. Jan. 2014 (CET)
Verstehe ich das richtig, dass nur magnetische Materialien Magnetfelder abschirmen können? Also hätte Blei keinen Einfluss auf das Magnetfeld?--77.3.161.21 17:40, 4. Jan. 2014 (CET)
Im Artikel Abschirmung wird MU-Metall genannt, das für den von Dir beabsichtigten Zweck industriell eingesetzt wird. --84.178.28.108 17:48, 4. Jan. 2014 (CET)
Wo ist dann der Haken an meinem Modell? --77.3.161.21 18:52, 4. Jan. 2014 (CET)
Der Haken an deinem Modell ist prinzipiell, dass bereits wild geschätzt 850.000 schlaue Leute vor dir versucht haben, ein Perpetuum mobile zu bauen. Es ist aber kein funktionierendes bekannt. Zusammen mit der in der Wissenschaft nun mal anerkannten Tatsache, dass ein PM gegen, je nach Typ, einen oder mehrere nun mal als gültig angesehene Hauptsätze der Thermodynamik verstößt, schlägt dir die oben beobachtete Skepsis entgegen. Ich würde persönlich nicht unbedingt ausschließen, dass es möglich sein könnte ;-), aber sowas glaubt man erst, wenn man es sieht. Eine Skizze reicht da nicht. Ich hoffe dir ist damit geholfen. Solaris3 (Diskussion) 19:12, 4. Jan. 2014 (CET)
Hey, ich bin doch nur technikinteressiert und habe doch geschrieben, dass mir der Energieerhaltungssatz schon aus der Schule bekannt ist. Ein Perpetuum Mobile wird es nie geben können, zumindest laut GK Physik Oberstufe :). Dennoch habe ich Spaß an Gedankenexperimenten :) --77.3.161.21 19:30, 4. Jan. 2014 (CET)
Ich hatte ihn so verstanden, dass er neugierig und kreativ mit dieser Idee spielt. Das hast Du sicher früher auch mal gemacht, bevor Du selbst heraus gefunden hast dass etwas nicht geht. 46.115.139.79 19:19, 4. Jan. 2014 (CET)
(BK) Den erfährst Du, wenn Du die Elektrotechnik-Grundlagenvorlesung aufmerksam verfolgst. Zeichne einfach mal den Verlauf der Feldlinien in Deine Anordnung ein, einmal wenn der Supraleiter (habe ich jetzt zuviel verraten?) über dem Dauermagneten steht und einmal wenn nicht. Und vergiss nicht, dass Du eine Kraft aufwenden musst, den Supraleiter durch das Feld des Dauermagneten zu bewegen. --Rôtkæppchen68 19:16, 4. Jan. 2014 (CET)
Ich hatte zwar mal einen LK Elektrotechnik am Technischen Gymnasium besucht, aber da haben wir uns eher mit Phasendiagrammen und den Berechnungen von Schaltkreisen und Bauelementen beschäftigt. War wohl nicht so der Hit, denn magnetische Feldlinien waren dort nie Thema. Aber ich werde versuchen deinem Hint mit dem Supraleiter hinter her zu recherchieren :) --77.3.161.21 19:36, 4. Jan. 2014 (CET)
Du mußt Stabmagneten etwas schräg zum Radius auf einer Kreisscheibe im Nonius anordnen (zu einer entsprechenden Anordnung mit einem mehr um die Kreisscheibe herum, versteht sich). Einmal anstoßen, dann rotiert es immer weiter, weil die Ausrichtung geringere Abstoßung gegen und größere Abstoßung in Drehrichtung bewirkt. ;o]) --217.84.114.250 19:54, 4. Jan. 2014 (CET)
(BK)Der Gedankengang ist folgender: Um Magnetfelder abzuschirmen brauchst Du entweder das oben erwähnte Mu-Metall, mit dem die Feldlinien gezielt seitlich am abzuschirmenden Bereich vorbei durch das Mumetall hindurch abgeleitet werden. Du willst aber ein nichtmagnetisches Material. Dann kann es eigentlich nur ein perfekt diamagnetisches Material sein, das es gar nicht zulässt, von Magnetfeldern durchsetzt zu werden. Bis zu einer gewissen Grenze (siehe Artikel) sind das Supraleiter. Das Magnetfeld wird hier am Supraleiter vorbei vollständig zur Seite gedrängt (Meißner-Ochsenfeld-Effekt). Einen Magnetischen Kreis bilden die Feldlinien aber so oder so. --Rôtkæppchen68 20:01, 4. Jan. 2014 (CET)
Mit Oberstufenmathematik ist das Magnetfeld in diesem Fall wahrscheinlich schwer zu berechnen oder verstehen. Ich weiß noch, dass ich mir damals mit den ganzen feldtheoretischen Grundlagen unheimlich schwergetan hab. Um das wirklich ordentlich zu berechnen, brauchst Du mindestens höhere Mathematik auf Hochschulniveau. In der gymnasialen Oberstufe wurden mir einfach die fertigen Formeln vor die Nase gesetzt, ohne auf die die Oberstufenmathematik übersteigenden Hintergründe einzugehen. Erst später an der Uni kam dann die höhere Mathematik, die zum tiefen Verständnis erforderlich war. Bei Dir ist das wahrscheinlich ähnlich. Aber: Den Energieerhaltungssatz und den zweiten Hauptsatz der Thermodynamik lernt man auch im Gymnasium. Von daher ist Dein Ansatz richtig: Du weißt, dass Deine Vorrichtung kein Perpetuum mobile sein kann, möchstest aber ergründen, warum genau nicht. Obigen Angriff von 80.108. musst Du von Dir abprallen lassen. Zum Blei: Blei wird als Abschirmungsmaterial gegen Radioaktivität verwendet. Es wird nur deswegen ausgewählt, weil es verhältnismäßig günstig ist und man damit geringe Schichtdicken braucht. Zur Abschirmung gegen Radioaktivität braucht ein Material keine besonderen Eigenschaften. Es kommt dabei nur auf die Flächenmasse (kg/m²) an. Du könntest also auch Daunendecken nehmen, solange die abzuschirmende Fläche mit der gleichen Masse belegt wird. Nur wären Daunendecken bei derselben Masse eben sehr viel dicker als Blei und deswegen nimmt man sie nicht. Viele Kernkraftwerke sind auch mit Beton abgeschirmt, wenn es auf den Platz nicht ankommt. Die magnetische Permeabilität von Blei ist größenordnungsmäßig ungefähr die von Vakuum. Das Magnetfeld wird davon also so gut wie nicht beeeinflusst. --Rôtkæppchen68 23:38, 4. Jan. 2014 (CET)

(Trollerei entfernt. Rôtkæppchen68 11:29, 5. Jan. 2014 (CET))

Nochmal. Nimm erst mal eine geerdete Keksdose aus Weißblech, siehe auch Permeabilität (Magnetismus). Und schick mir ein e-mail, wenns funktioniert. Besser wäre wahrscheinlich amorphes Metall, auch Amorphe Lötfolie. Gruß vom --Ohrnwuzler (Diskussion) 01:30, 5. Jan. 2014 (CET)

(Trollerei entfernt. Rôtkæppchen68 11:29, 5. Jan. 2014 (CET))

Sieh das mal anders: Im Bereich geringer Feldstärken ist Keksdosenblech ein praxistauglicher Ersatz für Mu-Metall. Wenn man seine mit NFC oder RFID ausgestattete Bankkarte, Personalausweis, Reisepass, Betriebsausweis etc in Keksdosenblech verpackt, kann das Teil nicht unbefugt ausgelesen werden. --Rôtkæppchen68 07:44, 5. Jan. 2014 (CET)

(Trollerei entfernt. Rôtkæppchen68 11:29, 5. Jan. 2014 (CET))

Könntest Du nicht einfach mal stille sein, wenn Dir nichts Zielführendes einfällt? --Rôtkæppchen68 10:03, 5. Jan. 2014 (CET)

(Trollerei entfernt. Rôtkæppchen68 11:29, 5. Jan. 2014 (CET))

Übrigens, lieber Fragesteller. Genau dieses Perpetuum Mobile hatte glaube ich absolut jeder von uns schon mal im Kopf :) Das ist nämlich genau das Modell, dass jeden einfällt der drüber nachdenkt ob ein Perpetuum Mobile nicht doch irgendwie ginge. Wenn du ein wenig googlest und youtubst findest du ganz sicher bereits fertig gebaute Versionen deiner Konstruktion und kannst dir die Mühe sparen. --92.226.30.125 11:45, 5. Jan. 2014 (CET)

Kabel-Name

Wie heißt das Kabel, das am normalen Kopfhörer vom mp3-Player oder vom Laptop dran ist? Wie lang kann ein Verlängerungskabel (zum Beispiel in Metern) vom Laptop sein, ohne dass ein Zusatzgerät notwendig wird. 46.115.139.79 19:57, 4. Jan. 2014 (CET)

Du meinst einen 3,5er Klinkenstecker, dreipolig, oder? --Hans Haase (有问题吗) 20:01, 4. Jan. 2014 (CET)
Ja, genau. Danke. 46.115.139.79 20:09, 4. Jan. 2014 (CET)
Zur Länge: Wenn du damit einen Kopfhörer ansteuerst, kann die Länge recht groß sein. Du kriegst jedoch abhängig von deinem individuellen Setup (zB Qualität des Kopfhörerverstärkers im Laptop) möglicherweise ab einer gewissen Länge ein Brummproblem. 5-10 Meter sollten aber in jedem Fall drin sein, denke ich - ausprobieren. Du kannst ja bei einem Versand Kabel Klinken-Verlängerungskabel verschiedener Länge kaufen und die zurückschicken, bei denen es zu sehr brummt. Ein "Zusatzgerät" wird dir wohl eher nichts bringen. Alternativ könntest du einen Infrarot-Drahtlos-Kopfhörerverstärker verwenden, da brauchst du aber in der Regel Sichtkontakt. Oder noch besser einen Funkkopfhörer... Solaris3 (Diskussion) 20:21, 4. Jan. 2014 (CET)
Da es sich um ein Laptop handelt ist bestimmt Bluetooth an Bord: Ein Bluetooth-Kopfhörer würde sich deshalb anbieten und die Fernbedienung des Mediaplayers wäre schon eingebaut! --84.178.24.65 22:20, 4. Jan. 2014 (CET)
Bluetooth hat nur leider beschränkte Reichweite. Ein Kabel kommt da deutlich weiter. Mein Bluetoothkopfhörer reicht nicht vom Schreibtisch bis aufs Klo, mein Analogfunkkopfhörer schon. --Rôtkæppchen68 10:00, 5. Jan. 2014 (CET)

OK, danke. Ich denke 10 Meter Kabel müssten gehen. 46.115.137.210 22:51, 4. Jan. 2014 (CET)

Die Stecker sind oft schwer zu löten. Bei fertigen Kabeln sind die Stecker eingeschweißt zu zuverlässig, jedoch ist das Kabel irgend etwas zwischen Zwillingslitze und Koaxialkabel. Das der Kopfhörer mit 64 Ω abschließt, wird er nicht das Problem sein. Ein Line-in mit seiner höheren Impedanz und Verbindung zum Netz, dagegen schon. --Hans Haase (有问题吗) 11:31, 5. Jan. 2014 (CET)
Vielen Dank, aber ich fürchte fast, das ist etwas kompliziert. Ich habe einfach ein Verlängerungskabel mit Klinkensteckern für Aktivboxen unter dem Fenster bestellt. :-) 46.115.89.171 12:11, 5. Jan. 2014 (CET)
Für Aktivboxen ist das genau das Richtige. 10 Meter sind hier auch nicht kritisch. Ich habe auch schon eine zusammengestückelte 20-Meter-Verlängerung für so etwas verwendet, ohne Probleme. --Rôtkæppchen68 13:13, 5. Jan. 2014 (CET)

Kaputte Eingabetaste umgehen

Im Moment sitze ich an einem Rechner mit kaputter Eingabetaste. Reparatur hier und jetzt ist unmöglich, also muss ich sie irgendwie umgehen. Während man sich in den meisten Fällen mit Mausklicks oder dem Einfügen eines kopierten Returns helfen kann, hilft das z.B. bei Facebook-Einträgen nicht, die man ja mit der Eingabetaste bestätigen muss. Wie könnte ich das lösen? Gibt es ein "Allheilmittel" für alle denkbaren Fälle? (Übrigens scheint auch die übliche Eingabe von ASCII-Zeichen auf Facebook nicht zu gehen.) --KnightMove (Diskussion) 21:44, 4. Jan. 2014 (CET)

Unter Windows: Bildschirmtastatur, zu finden mit der Suchfunktion des Startmenüs. --RadiohörerDiskBewertung 21:48, 4. Jan. 2014 (CET)
Sonst Start, Ausführen: OSK (OSK.EXE) → on screen kezboard --Hans Haase (有问题吗) 21:59, 4. Jan. 2014 (CET)
Danke euch beiden, klappt! (Weitere Wege sind trotzdem willkommen.) --KnightMove (Diskussion) 22:00, 4. Jan. 2014 (CET)
Es gibt auch noch die Entertaste beim Nummernblock, die funktioniert genauso. --Cubefox (Diskussion) 22:05, 4. Jan. 2014 (CET)
Es gibt aber auch Tastaturen ohne Nummernblock und ohne zweite Entertaste. --RadiohörerDiskBewertung 22:10, 4. Jan. 2014 (CET)
Falls es ein Notebook ist, wovon ich aufgrund der fehlenden zweiten Eingabetaste im Num-Block ausgehe, kann man auch die Taste vorsichtig abhebeln und mit dem Finger auf den dann freiliegenden Bereich drücken. So habe ich mal eine defekte Alt-Gr-Taste bedienen müssen. --91.0.145.182 22:51, 4. Jan. 2014 (CET)
Alternativ soll oft, aber nicht immer Strg+M oder Alt+(13) auf dem Zehnerblock helfen. --Rôtkæppchen68 23:06, 4. Jan. 2014 (CET)
Oder die Eingabe-funktion auf einen andere Taste der Tastatur legen, zB mit folgendem Reg-Eintrag (auf die F1-Taste = 3B,00 wird die Funktion der Eingabetaste = 1C,00 gelegt):
[HKEY_LOCAL_MACHINE\SYSTEM\CurrentControlSet\Control\Keyboard Layout]
"Scancode Map"=hex: 00,00,00,00,00,00,00,00,02,00,00,00,1C,00,3B,00,00,00,00,00

(Anleitung siehe zB hier.) --TheRunnerUp 10:26, 5. Jan. 2014 (CET)

In der Kneipe einpennen

Stimmt es, dass der Kneipier ein Bussgeld zahlen muss, wenn (besoffene) Gäste wegpennen? Meiner weckt mit der Begründung regelmäßig Leute auf, die zuviel saufen - aber ich hab dazu noch nix juristisches gefunden. Das Bundesland wäre Bayern, interessant wäre natürlich auch wie andere Länder das handhaben.

--84.153.17.133 03:43, 5. Jan. 2014 (CET)

Nein, denn man nennt das eigentlich nur inoffiziell Bussgeld, statt Umsatzeinbuße, weil das eine erprobte Taktik ist.--Mr. Froude (Diskussion) 04:44, 5. Jan. 2014 (CET)
Trainingshilfe. In (Ober-)Bayern hat man erst das Recht ab 3 Promille abzutreten, man muss ja schließlich auch noch fahren können.--79.232.219.209 08:51, 5. Jan. 2014 (CET)
Etwas in der Art wird abgehandelt in populären Werken von Ralf Höcker. --Hans Haase (有问题吗) 10:06, 5. Jan. 2014 (CET)

Klimmzüge bei Frauen

Heute gab es die Nachricht, dass die US-Marines davon absehen wollen, von ihren Bewerberinnen drei(!) Klimmzüge zu verlangen, weil die Hälfte der weiblichen Marines das nicht schaffe. In einem Internet-Forum schreibt jemand, dass seine Freundin Turnerin mit viel Kraft im Oberkörper ist, aber gar keinen schafft. Die deutsche Polizei fordert laut [7] nicht in jedem Bundesland "richtige" Klimmzüge von den Bewerberinnen. Wir sprechen jetzt mal nicht von durchschnittlichen Frauen, sondern von richtig sportlichen - selbst denen scheinen Klimmzüge überraschend schwerzufallen. Gibt es dafür spezifische physiologische Gründe? --Eike (Diskussion) 14:26, 3. Jan. 2014 (CET)

Erster Antwortansatz, ohne Anspruch auf Kompetenz meinerseits: Dieser kleine Essay legt nahe, dass der eigentlich dafür zuständige Latissimus bei den meisten Menschen ohne spezielles Training und Aktivierung durch entsprechendes Aufwärmen nicht kräftig genug ist, um einen aus der gestreckten Haltung nach oben zu ziehen, die meisten Männer aber (im Gegensatz offenbar zu Frauen) in der Lage sind, das mit anderen Muskeln auszugleichen, oder sie aber gar nicht erst in der gestreckten Haltung anfangen. --YMS (Diskussion) 14:58, 3. Jan. 2014 (CET)
1 1/2 Stunden vorher ...  :-) GEEZER… nil nisi bene 15:02, 3. Jan. 2014 (CET)
SPON zur Frage (nicht signierter Beitrag von 2003:5F:2B38:864:12E:48B6:A198:3501 (Diskussion | Beiträge) 16:03, 3. Jan. 2014 (CET))
kleiner Beitrag: mW sind Frauen bei der US Armee für keinen Kampfeinsatz vorgesehen. (schon mal eine Frau mit zu viel Muskeln gesehen?) --80.108.60.158 16:11, 3. Jan. 2014 (CET)
(Oh ja.) --Eike (Diskussion) 16:19, 3. Jan. 2014 (CET)
ich hab auch schon welche gesehen, und dennoch ist mein konservatives Verständnis in einem anderen Bild verankert. So, oder so ähnlich wird es wohl auch im amerikanischen Verständnis sein. Von der Physiologischen Voraussetzung - also dem "angeblich" zierlicherem - steht uns vorerst ein Fragezeichen im Raum. Bis ein kleiner Bengel mit einem Lichtlein daher kommt. --80.108.60.158 16:25, 3. Jan. 2014 (CET)
(Nach BK)Um es einfach zu erklären: Frauen haben weniger Muskelmasse als Männer, das lässt sich auch nicht wegtrainieren. Da wundert es nicht dass so gut wie keine Frau in freiwillig in einem körperlich anstrengenden Beruf arbeitet. (Nebenbei: Beim Klettern kehrt sich dieser Nachteil um, Frauen kommen da fast an Männer ran weil sie weniger hier überflüssige Muskelmasse haben müssen. Es verblüfft, wie mühelos acht- bis zehnjährige Mädchen einer Kletterwand hochkommen.)--Antemister (Diskussion) 17:05, 3. Jan. 2014 (CET)
Der einzige Grund ist vermutlich das Frauen leichter Übergewicht bekommen, was speziell in den USA ein bekanntes Problem der dortigen Esskultur ist. --Kharon 17:30, 3. Jan. 2014 (CET)
Antemister, lies doch einfach mal vor dem Antworten die Frage. Es ging um durchtrainierte Frauen, um Sportler, um Frauen in Männerberufen. "Wir sprechen jetzt mal nicht von durchschnittlichen Frauen". --Eike (Diskussion) 18:09, 3. Jan. 2014 (CET)
Es geht doch gerade darum dass sich der Nachteil von weniger Muskeln nicht bei Frauen nicht durch mehr Training beheben lässt, sodass sie mit Männern gleichziehen können.--Antemister (Diskussion) 18:35, 3. Jan. 2014 (CET)
Gleichziehen verlangt ja keiner. Aber schau dir zum Beispiel Sprinter an, da sind die Frauen beim Weltrekord 10% hinter den Männern. Man kriegt das Gefühl, dass es beim Klimmzug Welten wären. (Natürlich ist Weltrekord und Marines wieder ein hinkender Vergleich.) --Eike (Diskussion) 18:44, 3. Jan. 2014 (CET)
Ich sage, es liegt -vor allem- an schlechten Trainern und schlechtem Trainingsverhalten. Wenn Mädchen in die Pubertät kommen, hören die meisten erst mal mit Sport auf, weil sie sich in ihrem (veränderten) Körper nicht wohlfühlen und vermittelt kriegen, dass sie mit Jungs eh nicht mithalten können usw. oder weichen in weniger kraftintensive Sportarten aus, während Jungs tendenziell erfahren, dass alles Körperliche leichter wird, ohne dass sie viel dafür tun müssen. Klimmzüge trainiert selbst eine sportliche Frau nur genau daurch, dass sie Klimmzüge oder darauf angepasste Übungen in der Muckibude macht - warum sollte sie? Auch sportliche Männer, sagen wir mal Läufer, können oft keine 3 Klimmzüge ohne extra trainieren. Mit Klimmzügen habe ich als Trainer konkret keine Erfahrungen, aber was das in den Links auch erwähnte Werfen angeht, kann ich sagen, Mädchen können technisch so gut wie Jungs werfen, wenn man ihnen die Technik beibringt - und zwar möglichst früh. Auch viele Jungs kriegen das nicht von selbst raus oder erst nach langem Üben. Hier versagen Turnlehrer und ehrenamtliche Vereinstrainer in der Regel komplett; entweder man kanns schon oder man guckts sich ab oder man hat Pech. Dabei bringt ein bisschen Anatomie, welche Muskeln benutze ich denn da, Hebelgesetz, das Veranschaulichen von Körperbewegungen in Zeitlupe usw. sehr viel. --92.202.57.120 17:51, 3. Jan. 2014 (CET)
Vielleicht unterschätzt man auch Klimmzüge. Als Kinder konnten wir welche (ich glaub, selbst ich konnte drei, und das will was heißen...), und als Erwachsener hat man's halt erst gar nicht mehr probiert... --Eike (Diskussion) 18:10, 3. Jan. 2014 (CET)
free re-fill? *rotfl* --Heimschützenzentrum (?) 18:19, 3. Jan. 2014 (CET)
Es kommt auch darauf an, wie breit man greift und mit welcher Griffart man die Klimmzüge zieht. Aus persönlicher Erfahrung kann ich sagen, dass es gar nicht sooo leicht ist, bei Klimmzügen gezielt den Latissimus anzusprechen. Das muss man üben. Wenn man die Klimmzüge nicht beherrscht bzw. man den Lat. nicht anzusprechen weiß, zieht man überwiegend mit dem M. biceps brachii (bei breitem Griff im Ristgriff). Auch meine ich, dass die Genetik eine Rolle spielt: Der Muskelansatz des Lat. dorsi ist nicht bei jedem Menschen gleich. Sieht man daran, dass manche einen "richtigen" V-Oberkörper ausbilden können, andere nicht. Nächste Woche lasse ich mir von unseren Mädels im Training mal Klimmzüge zeigen. Eine schafft auf jeden Fall welche; das habe ich schon mal gesehen. Ich schaffe etwa 25 Klimmzüge ohne Gewicht, etwa 10 mit 10 kg-Gewichtsweste. *scnr* --Turnstange (Diskussion) 01:16, 4. Jan. 2014 (CET)
eventunell hilft es den Radius weiter zu werfen. Wie war das eigentlich damals bei den Velociraptoren-Weibchen? --80.108.60.158 11:06, 4. Jan. 2014 (CET)
Au ja, experimentieren! :o) Wie alt sind denn deine "Mädels"? Bei Kinder scheint es ja noch leichter zu sein... --Eike (Diskussion) 13:07, 4. Jan. 2014 (CET)
bei meinem Feldversuch, reicht es, wenn man gedanklich eine bis max. - durchschnittlich eher weniger - Hürden nehmen kann. Insofern ist ein Beitrag aus einer Herzensseite wesentlich stämmiger als ein kraftsprotzendes KlimmzugAnimationsTraining. Wobei wir auch die Wertigkeit als solche wohl eher bei Sanftmut und Toleranz gelten lassen sollten, als ein aufgeblasenes Stangen-erhaschendes KriegsWeibleinFähigkeiten-zeigendes ..... --80.108.60.158 13:19, 4. Jan. 2014 (CET)
Vom Hörensagen kenne ich dazu die Aussage dass es Frauen vor allem an Muskulatur in Armen und Oberkörper fehlt, weswegen das Tragen schwerer Lasten oder Erdarbeiten für Frauen eine Qual sind. Beim Laufen etwa wirkt sich der Unterschied weit weniger aus. Keine Sportlerin hier die hier die Details kennt, etwa für Disziplinen wie Gewichtheben?--Antemister (Diskussion) 13:25, 4. Jan. 2014 (CET)
möglicherweise, oder, besser gesagt: witziger Weise sollten wir das neueste Barbiemodell abwarten um eine neue Referenz zu bekommen. Als natürlicher Fan der überwachbaren Unterhaltungselektronik könnte man natürlich auch bei der NSA nachfragen, was ihnen schon alles untergekommen ist, vorausgesetzt natürlich, dass diese wirklich so gut sind wie sie in ihrer Bewerbung angegeben haben. --80.108.60.158 14:40, 4. Jan. 2014 (CET)
@Eike: Die Mädels sind so zwischen 20 und 26, also im besten Soldatinnen-Alter. ;-) Könnte das Experiment auch noch mit jüngeren Mädels durchführen lassen; die wären dann 14/15. Ich bin der Meinung, dass fast jede Kraftsportlerin Klimmzüge schafft. Daher werden das vermutlich auch alle unsere Gerätturnerinnen schaffen. Turner, Kletterer usw. sind für solche Übungen geradezu prädestiniert. Viele andere Sportler schaffen dies eben nicht, Männer wie Frauen gleichermaßen. Grund: Man benötigt, vereinfacht gesagt, viel Kraft im gesamten Oberkörper, vor allem Zug- und Griffkraft. Diese haben Läufer, Ball- und Rückschlagsportler etc. eben überwiegend nicht. Kraft resultiert aus dem Muskelquerschnitt. Entsprechendes Training führt zur Muskelhypertrophie und damit zu einem Kraftzuwachs. Genetische Dispositionen spielen aber, wie gesagt, sicher ebenfalls eine Rolle. Frauen sind schon aufgrund der geringen Testosteron- bzw. Wachstumshormonsekretion gar nicht in der Lage, vergleichbar viel Muskulatur wie Männer aufzubauen. --Turnstange (Diskussion) 10:29, 6. Jan. 2014 (CET)

Prepaid-3G für Ukraine und Rumänien

Moin!

Ich bin auf der Suche nach (natürlich frei und unkompliziert erhältlichen) Prepaid-SIM-Karten, mit denen ich günstig in der Ukraine und in Rumänien ins Netz kann. Schön wäre folgendes:

  • Für eine ukrainische Karte
    • gute bis sehr gute Netzabdeckung in Metropolregionen (v.a. Lwiw, Kiew und Odessa),
    • aber auch Datennetz (z.B. Fallback auf GPRS/EDGE) in dünner besiedelten Gebieten,
    • und grundsätzlicher Empfang zum Telefonieren möglichst (fast) landesweit,
    • vielleicht sogar (minimalistisches?) Roaming in Moldawien (ohne Transnistrien) möglich.
  • für eine rumänische Karte
    • gute bis sehr gute Netzabdeckung in Städten (v.a. Bacău und Cluj-Napoca),
    • aber auch Datennetz (z.B. Fallback auf GPRS/EDGE) in dünner besiedelten Gebieten,
    • und grundsätzlicher Empfang zum Telefonieren möglichst (fast) landesweit,
    • vielleicht sogar (minimalistisches?) Roaming in Moldawien (ohne Transnistrien) möglich.

Ich würde mich besonders über Antworten aus eigener (jüngerer) Erfahrung freuen, da (A) die entsprechenden Webseiten der Unternehmen nicht besonders aussagekräftig sind zu einigen meiner Fragen, (B) die Entwicklung auf diesem Gebiet in den genannten Ländern äußerst rasant und dynamisch ist und (C) zumindest für die Ukraine die wenigen ausführlichen Foren-Beiträge uralt und damit vermutlich für die Tonne sind.

Vielen Dank im Voraus! —[ˈjøːˌmaˑ] 12:33, 4. Jan. 2014 (CET)

In der Ukraine bin ich zuletzt im November mit den Prepaid-Tarife von МТС (im Westen) sehr gut gefahren. Die Tarife verändern sich natürlich schon mal, aber die Beratung in den etwas größeren Läden sehr gut (die aktivieren Dir auch die Karte auf dem Gerät usw.). Smartphone/Tablet-Tarife, bis 3GB zu sehr kleinen Preisen. Verbindung war stabil bis auf einzelne östliche Karpatennester, da ist Kievstar oder djuice etwas stärker. --elya (Diskussion) 14:28, 4. Jan. 2014 (CET)
Danke, Elya! Die Angebote von MTS/МТС sehen gut aus – zusammen mit Deinen Erfahrungen (auch ich werde fast nur in der West-Ukraine unterwegs sein) scheint das einen Versuch wert zu sein. Werde mich gleich in Lwiw danach umsehen. —[ˈjøːˌmaˑ] 16:17, 4. Jan. 2014 (CET)
2010 so kenne ich es noch, war eine Prepaid 3G/UMTS Karte nur über ein Kundencenter vor Ort unter Angabe einer moldawischen Adresse möglich und in dünner besiedelten Gebieten ging nichts. --Search and Rescue (Diskussion) 14:21, 4. Jan. 2014 (CET)
Ja, dass das mit der Republik Moldau nicht so einfach wird, hatte ich mir schon gedacht. Eine eigene Karte dort beschaffen muss ich auch nicht unbedingt: Ich werde nur einen Tag durchs Land fahren (Одеса(UA)–Palanca(MD)–Bacău(RO)), daher wäre ein mögliches MD-Roaming einer ausländischen SIM-Karte ein nettes „Sahnehäubchen“ gewesen. ;) Nach erstem Überfliegen könnte das mit dem МТС-Angebot sogar möglich, wenn auch teuer sein. —[ˈjøːˌmaˑ] 16:17, 4. Jan. 2014 (CET)

Noch jemand ’n Tip für Rumänien? —[ˈjøːˌmaˑ] 16:17, 4. Jan. 2014 (CET)

Ist das überhaupt eine Wissensfrage? --RadiohörerDiskBewertung 22:21, 4. Jan. 2014 (CET)
Ja: Ich weiß nicht, welche Predaid-SIM man am besten für gute 3G-Netzabdeckung in Rumänien kauft. Wer weiß Rat? ;) Beste Grüße —[ˈjøːˌmaˑ] 18:07, 5. Jan. 2014 (CET)

Entfernen von Exif-Tags

Wie entferne ich Exif-Tags? Was ist von JPEG & PNG Stripper zu halten? In c’t Digitale Fotografie I/2014 heißt es nur „In den meisten Linux-Dristributionen [...]“. Habsch nicht als DAU. Was tun? Ich würde Wikipedia Fotos zur Verfügung stellen, aber hier werden zu viele Daten gesammelt. --80.187.109.29 17:15, 4. Jan. 2014 (CET)

Unter W7 kannst Du mit Rechtsklick auf die Bilddatei deren Eigenschaften öffnen. Im Reiter „Details“ gibt es ganz unten den Link „Eigenschaften und persönliche Informationen entfernen“, welcher ein weiteres Fensterchen öffnet. In diesem lassen sich einzelne Eigenschaftsmerkmale (oder alle) zur Entfernung auswählen. --84.178.28.108 18:01, 4. Jan. 2014 (CET)
Ich nutze hierfür seit Jahren das Programm jpegtran.exe der IJG, siehe Fußnote im Artikel JPEG und en:libjpeg. --Rôtkæppchen68 19:25, 4. Jan. 2014 (CET)
ExifTool. Mit exiftool -all= Datei.jpg sind alle(!) Metadaten weg. --Tors (Diskussion) 19:37, 4. Jan. 2014 (CET)
Die Exifs ganz zu löschen, wäre kein feiner Zug, weil einige sich gern die Kameraeinstellungen gelungener (oder auch nicht gelungener) Fotos anschauen. Mit dem Löschen des Datums geht ein Teil des enzyklopädischen Wertes verloren - zumindest das Jahr wäre nett. ;-) Ich verwende zum Bearbeiten der Exifs den Geosetter, der vor allem zum Positionieren der Fotos gedacht ist, aber auch das Datum kann man ändern oder andere Dinge hineinschreiben. Noch mehr läßt sich mit PhotoME machen, abgesehen vom Ändern einzelner Einträge kann man z.B. auch die Exifs exportieren und importieren (wenn man z.B. vermeiden will, daß das Bearbeitungsprogramm einen Eintrag hinterläßt - Exifs abspeichern, nach der Bearbeitung wieder einfügen). Liebe Grüße, --Häferl (Diskussion) 19:42, 4. Jan. 2014 (CET)
Nur sollte das eine Digitalkamera sein, die ihre Zeiteinstellung mit Datum vom GPS nimmt. Zum entfernen tut es auch ein Bildbearbeitungsprogramm, mit dem das Bild in Teilen oder voll ausgeschnitten und in ein neues eingefügt wird. Das Ergebnis sollte dennoch überprüft werden. --Hans Haase (有问题吗) 20:05, 4. Jan. 2014 (CET) ↓↓↓↓ Ich koch doch nur mit Wasser.
Zauberer verraten aber doch ihre Tricks nicht ;-) ExifTool (das übrigens auch von GeoSetter zum Schreiben der Exif-Daten genutzt wird) kann selbstverständlich auf Wunsch auch nur bestimmte Metadaten entfernen. --Tors (Diskussion) 20:04, 4. Jan. 2014 (CET)
Ich machs mit „exiftool -all= -overwrite_original Datei.jpg“. Einfach in der Kommandozeile. ExifTool kannst du dir, siehe obiger Link, auch auf Mac-OS-X- und Windows-Betriebssystemen installieren, am einfachsten ist es natürlich über den Paketmanager deiner Linux-Distribution (und nein, Linux ist nicht nur für „Experten“). --Chricho ¹ ² ³ 23:57, 4. Jan. 2014 (CET)
Welche Daten sollen denn nicht gesammelt werden? Wenn man seiner Kamera Adresse und Telefonnummer verrät, dann erzählt sie das weiter, das ist normal. Ist ja schließlich _die_ Kamera. Soll nicht bekannt werden, wann du wo gewesen bist? Nun, das kann man am Motiv auch erkennen. EXIF sind etwas nettes für die Nachwelt und sie werden wohl in Zukunft noch umfangreicher und wichtiger werden. Andererseits gibts die erst ein paar Jahre, früher kam die Fotografie auch ohne aus. --Pölkky 10:26, 5. Jan. 2014 (CET)
Früher wurden auch [Panzer nach Holzmodell] gekauft. ;-) --Hans Haase (有问题吗) 11:47, 5. Jan. 2014 (CET)
Es geht niemanden etwas an, welche Kamera und welche Objektive ich besitze. Und an welchem Tag zu welcher Uhrzeit ich wo genau war. Monat reicht. Und manchmal auch das Jahr. Oder was immer ich geruhe anzugeben. --AndreasPraefcke (Diskussion) 23:49, 5. Jan. 2014 (CET)

Bibliothekswesen in der DDR

Mal wieder eine Frage der Sorte "Wessi an Ossi": Gab es denn in der DDR/im Ostblock allgemeine einen freien Zugang zu Bibliotheken? Konnte man sich dort Privatmann ein Werk aus dem Westen, das nicht etwa unverfängliches Fachbuch zu naturwissenschaftlich-technischen Themen war, einfach so ausleihen oder zumindest anschauen? Es geht gar nicht mal um wirklich subversive Werke, sondern wissenschaftliche Literatur zu politischen, historischen oder wirtschaftlichen Themen, oder auch an sich harmlose Unterhaltungsromane, die nichts weiter tun als das Leben im Westen realistisch darstellen.--Antemister (Diskussion) 22:15, 4. Jan. 2014 (CET)--Antemister (Diskussion) 22:15, 4. Jan. 2014 (CET)

Bibliotheken gab es in großem Maßstab und sie wurden viel mehr als heute genutzt. Sie enthielten so ziemlich alle verfügbare Literatur, prinzipiell auch solche aus dem Westen. Es war aber nicht alles "verfügbar". Zum Beispiel wenn sie gespendet wurden. Wörterbücher, Kochbücher und sowas, aber keine Schundliteratur wie Konsalik. Allerdings ab es auch Bücher, die in abgetrennten Räumen standen und nicht jedermann zugänglich waren. 46.115.137.210 22:47, 4. Jan. 2014 (CET)
Es gab schon freien Zugang zu den Bibliotheken, nur waren dort eben nicht alle Bücher zu finden. Oder sie waren in Giftschränken verstaut (Siehe dort "Sperrbibliothek" war der amtliche Begriff). Im Wissenschaftsbetrieb wurden auch mit Einschränkungen Westzeitschriften zugänglich gemacht, aber es gab nur wenige Exemplare, weshalb sie immer weitergereicht werden mussten. Darum wurden HIWIs dazu abbestellt mit Fotoapparat alles manuell zu kopieren, damit man die Bücher/Magazine immer vorrätig hatte im Institut, auch wenn das Original schon längst weiter gegeben wurde. Mein Vater erzählte mir, dass es auch oft schwer war, den zuständigen Gremien zu vermitteln, warum eine interessante Zeitung weiterhin abonniert werden sollte. Interessant waren sie ja schließlich alle, weshalb das manchmal ausuferte. -- sk (Diskussion) 22:51, 4. Jan. 2014 (CET)

Ich kenn nur "Milde Sorte", was aber im Buchladen nicht zu ordern war kam auch nicht in die Ausleihe, wozu auch?--Mr. Froude (Diskussion) 05:18, 5. Jan. 2014 (CET)--Mr. Froude (Diskussion) 05:24, 5. Jan. 2014 (CET)

Rein technische Informationen waren auch in Form von Westliteratur verfügbar. Bauvorschriften, Normen und Fachbücher habe ich z. B. jederzeit in der Bibliothek der Bauakademie in Berlin einsehen können. Verleih ging nur auf Anmeldung, weil die Westwerke begehrt waren und nur in wenigen Exemplaren verfügbar. Es gab ebenfalls gute Literatutr zur Programmierung, oft in Form von Ormig-Kopien. Dabei war es bei einigen Exemplaren schon seltsam, daß die im Osten überhaupt exisatierten. Der komplette Assembler-Quelltext des 8087 war vielleicht doch nicht so ganz legal aus dem Westen importiert worden. Es gab auch reichlich sowjetische Literatur (bis hin zu Promotionen), was bei speziellen Suchen sehr hilfreich war. In der Geschichtsbibliothek der Humboldt-Uni gab es auch einen Giftraum, so ganz einfach kam man an die Werke dort nicht ran. Bei jeder einzelnen Ausleihe mußte man schriftlich begründen, warum und wofür man gerade dieses Buch braucht. Ein "interessiert mich" interesasierte nicht. Die Handhabung war also grundlegend anders. Während ich in der Bauakademie als Baustudent gemeldet war und alles bekam, was ich wollte, war das an der Uni fast unmöglich. Wenn dort etwas gut begründet wurde, dann wurde der Grund nämlich erstmal überprüft. Es gab sogar "Naziliteratur" ;) Bauvorschriften zur Autobahn beispielsweise, die gab es sogar in der kleinen Bibliothek unserer Schule frei verfügbar. Katasterpläne mit unterirdischen Leitungen der Berliner Wasserbetriebe, der U-Bahn usw. wurden etwas besser bewacht, die gabs aber auch. Ohne konnte man ja nichts in Grenznähe projektieren und iun Berlin war quasi fast alles grenznah. --Pölkky 10:13, 5. Jan. 2014 (CET)
Vielleicht dazu interessant: Liste der auszusondernden Literatur --84.135.157.112 14:28, 5. Jan. 2014 (CET)
Um zu den Schätzen & dem Giftraum der (Humboldt)-Uni vorgelassen zu werden musste man idR zuvor aber noch 3 Jahre bei der NVA gedient haben, oder?--Mr. Froude (Diskussion) 20:08, 5. Jan. 2014 (CET)
Nein, das ist Quatsch. Ich habe als Lehrling 1980 eine Belegarbeit zur Geschichte der Schorfheide geschrieben und habe dafür auch Werke aus dem Giftschrank bekommen. Ich brauchte freilich eine Bestätigung , daß dies wirklich meine Aufgabe ist. --Pölkky 20:24, 5. Jan. 2014 (CET)
Ich bin (in den 80ern) ein "klassischer" Bibliotekskunde (Stadtteilsbibliothek) gewesen, weil man dort "alles" bekam; Literatur, Sachbücher, Musik, Erotik usw.. Glasnost und ähnliche Themen konnte man dort weit vor der "Wende" nachlesen. Ich war auch in der Berliner Unibibliothek (ohne Student zu sein) und da gab es gefühlt dann wirklich alles. Bibliotheken hatten einen sehr hohen Status und er Beruf der Bibliothekarin (kein Gender) war sehr beliebt (auch aufgrund von weiteren Entwicklungsmöglichkeiten).--Wikiseidank (Diskussion) 20:32, 5. Jan. 2014 (CET)

Lautsprecher-System

Ich habe nun schon wiederholt Berichte über die Audioengine 5+ gelesen, zuletzt in der FAZ ( http://www.faz.net/aktuell/technik-motor/audio-video/lautsprecher-und-funkbruecke-von-audioengine-musik-spielpartner-fuers-notebook-12724882.html ). Ich verstehe aber nicht, was ich mir außerdem bestellen muss, damit ich meinen Laptop über Funk an die Lautsprecher anschließen kann. Im FAZ-Artikel ist von der Audioengine D2 die Rede, an anderer Stelle lese ich von Audioengine W3. Letztere scheint man nur in den USB-Schlitz des Klapprechners schließen zu müssen. Was also muss ich mir nun alles bestellen? Vielleicht kennt sich jemand von Euch ja mit Lautsprecher-Laptop-Schnittstellen besser aus als ich. 130.149.169.40 13:41, 5. Jan. 2014 (CET)

Brauchst du nichts. Es ist für den Sender ein USB-Kabel dabei und für den Empfänger ein RCA-Kabel. Eventuell mußt du es an einem Ende abschneiden, damit du die Boxen anschließen kannst, viele haben nur Schraub- oder Klemmenanschluß.--84.187.81.130 18:19, 5. Jan. 2014 (CET)

Bezeichnung von Fotos

Hallo, wie würdet ihr Fotos nennen, die zwar nicht inszeniert sind, aber doch bei einem besonderen Anlass entstanden sind? In einem Interview werden einerseits die (komplett inszenierten Bundesratsfotos) thematisiert, andererseits aber auch die Bilder von der Bundesratsreise. ‚Schnappschüsse‘ finde ich unpassend, weil die Fotografen ja schon konkret zu dem Anlass mitgereist sind, habt ihr eine Idee? --Port(u*o)s 14:31, 5. Jan. 2014 (CET)

Bei einem Artikel habe ich sie in offiziell und inoffiziell unterschieden. GEEZER… nil nisi bene 14:37, 5. Jan. 2014 (CET)
Ich meine, die angereisten Fotografen machen Bilddokumente von dem Anlass.--79.232.219.209 14:54, 5. Jan. 2014 (CET)
@Grey Greezer: Ja, das offiziell/inoffiziell wird in dem Interview auch breit besprochen - es geht um Inszenierung von Politik bzw. die Kleidung von Politikerinnen und Politikern. In der Einleitung zu dem Interview werden nun die Fotos erwähnt. Was ich suche, ist eine Bezeichnung, die das ‚nicht extra so Hingestellte‘ charakterisiert, Teil einer Inszenierung ist das ja auch, wenn der Bundesrat eine Reise macht und Profis zum Fotografieren hinzubittet. Schnappschüsse wiederum sind (für mich) eher, wenn jemand zufällig Prinzessin Kate im Gespräch mit Prinz Charles an der Supermarktkasse entdeckt und draufhält - oder wenn man gerade einen spielenden Hund sieht, den man ablichten möchte. Wenn man die Kamera also eher zufällig dabei hat.
@IP: Ja, Bilddokumente ist vielleicht die richtige Richtung – etwas griffiger wäre schön … Port(u*o)s 14:59, 5. Jan. 2014 (CET)

Ich könnte mir "Situationsfotos" denken.--G-Michel-Hürth (Diskussion) 15:48, 5. Jan. 2014 (CET)

Ja, das wäre vielleicht eine Idee gewesen. Meine Frau hats jetzt weggeschickt und einfach ‚Pressefotos‘ genannt. Geht dann aus dem Zusammenhang im Interview hervor. Gruss Port(u*o)s 16:08, 5. Jan. 2014 (CET)
Schon weg - trotzdem.
Pressefoto ist "zweischneidig". Unternehmen (Regierungen) benennen auch hübsch gemachte, die Schokoladenseite zeigende Photos, die dann an die Medien zur Verwendung weitergeleitet werden, als "Pressefotos". GEEZER… nil nisi bene 10:47, 6. Jan. 2014 (CET)

Autobahnknoten

Laut Bundesautobahn#Organisation verlaufen in Deutschland die Autobahnen mit geraden Nummer in Ost-West-Richtung, die mit ungeraden Nummern in Nord-Süd-Richtung (als Ausnahme fallen mir der Berliner Ring und der Autobahnring München ein). Mit würde deshalb interessieren, ob es (abgesehen vom Berliner/Münchner Ring und eventuell anderen Städte-Ringen) Autobahnkreuze und Dreiecke gibt, an denen sich nur Autobahnen mit gerader /ungerader Nummer schneiden. Rein geometrisch betrachtet sollte es das ja nicht geben, das sich Parallelen ja nicht schneiden ;-). 213.54.83.210 15:21, 5. Jan. 2014 (CET)

Wenn Du mehrmals auf die Karte im Artikel Bundesautobahn klickst, kommt eine Version mit Beschriftung (besser gesagt: Man kann die Nummern dann lesen, weil sie groß genug sind). Da kannst Du - falls hier niemand die ultimative Antwort hat - selbst nachsehen. Gruß, --DerFeigling (Diskussion) 15:31, 5. Jan. 2014 (CET)
Verläuft eine BAB ringförmig, könnte sie nur nach einem Abschnitt nummeriert werden. Das ist ist Auslegungssache und Theorie. Interessant ist auch, dass die A8 nicht ununterbrochen verläuft und die A81 durch die A8 verläuft oder unterbrochen wird. Je nach Bauform eines Autobahnkreuzes kann ein TOTSO entstehen. --Hans Haase (有问题吗) 15:38, 5. Jan. 2014 (CET)
Aus dem Kopf: A1/A3, A1/A7, A3/A5, A3/A7, A3/A9, A5/A7, A1/A45, A2/A30, A2/A352, A7/A37, A1/A29, A5/A45, A45/A485, A66/A648, A3/A81, A5/A659, A5/A67, A14/A2, A14/A24, A14/A4, A4/A72, A9/A93, A6/A8, A6/A620, A8/A820, A1/A61, A61/A63, A61/A65, etc... --Baladid Diskussion 15:41, 5. Jan. 2014 (CET)
Ich hab mal nur um Köln herum geguckt und auf die A1 gekommen. die verläuft hier teilweise mehr in Ost-West- als in Nord-Süd-Richtung und hat viele Dreiecke und Kreuze mit "ungeraden" Autobahnen. Wenn man die rausnimmt, die am Kölner Autobahnring liegen, finde ich schnell das Autobahnkreuz Bliesheim, das Autobahndreieck Erfttal und, etwas weiter weg aber dafür wohl am bekanntesten, das Westhofener Kreuz. --dapete 15:58, 5. Jan. 2014 (CET)
Es gibt sogar zwei Autobahnen mit ungerader Nummer, die sich zweimal schneiden: Die A1 und die A43 in Wuppertal Ost und in Münster Süd. --80.187.101.115 16:58, 5. Jan. 2014 (CET)
Erwähnenswert ist da auch das Autobahndreieck Hockenheim mit der A6 in Nord-Süd-Richtung und der A61 von Westen, also genau verkehrt herum. --mfb (Diskussion) 20:07, 5. Jan. 2014 (CET)

Alkohol/Wasser Lösung entfernen

Ich habe (hatte) zwischen zwei Folien in einer nicht demontierbaren Tastatur eine größflächige Wasseransammlung. Ich habe dann die Tastatur in Spiritus eingelegt in der Hoffnung, dass das das Wasser verdrängt und ich das dann durch Erhitzung verdampfen kann. Wenn ich eine Taste entferne kann ich sehen, dass in der Folie immer noch eine Flüssigkeit ist. Intensiver Einsatz eines Heizlüfters hat auch nicht geholfen. Wie bekomme ich den Alkohol oder das Wasser oder die Mischung aus beidem möglichst effektiv aus dem Zwischenraum der Folien? Silica Gel eignet sich ja für Wasser, wie ist es mit Alkohol? --93.132.159.174 15:48, 5. Jan. 2014 (CET)

Könnest höchstens versuchen eine Tissue auf den unteren Folienrand zu pressen, um des mit Kapillarität zu versuchen, aber das sieht schlecht aus. Andererseits sollte der Spirtus nicht leitend sein, was Kurzschlüsse verhindern würde, außer das Wasser ist noch drin und hat diverses angelöst, so dass die Tastschalter als betätigt erkannt werden. Bei einem Laptop, tausche es aus, eine herkömmliche PC-Tastatur ist billiger in der Anschaffung, aber oft dennoch reparierbar. Kompressor, Pressluft? Schutzbrille tragen!!! Hält die Tastatur 110°C aus? Wenn ja, in den geregelten Umluft-Ofen damit. --Hans Haase (有问题吗) 15:58, 5. Jan. 2014 (CET)
@Hans: Wie kommst Du da drauf, dass Spiritus nicht leitend sei? Ethanol ist polar, durchaus elektrisch leitend und ein Wasser/EtOH-Gemisch sowieso. Nur den Wasser- oder Ethanolanteil da raus zu bekommen ist sowieso unmöglich, auch durch Hitze; siehe Azeotropisches Gemisch. Eine Tastatur (geschweige ein Notebook!) mit 110°C zu malträtieren... naja! Um was für eine "nicht demontierbare Tastatur", welche "in Spiritus eingelegt"wurde handelt es sich da denn? Ohne Details zu kennen, kann man nicht mehr empfehlen, als bei moderaten Temperaturen (so etwa 45°C können die meisten elektronischen Geräte gerade so noch ab) zu trocknen, evtl auch über längeren Zeitraum. Vorsichtiges evakuieren im Exsikator mit Trockenmittel kann evtl auch helfen, wenn man dazu Zugang hat! Na, sonst gibt's noch Fachbetriebe, die sich für wenig oder eher mehr Geld darum kümmern... --Lexx105 (Diskussion) 17:46, 5. Jan. 2014 (CET)
Dazu kommt, dass die Flüssigkeit durch die Kapillarwirkung zwischen die beiden Folien gesaugt wird und dann dort bleibt, weil da kein Luftraum ist, in den die Flüssigkeit verdunsten könnte. Sie diffundiert höchstens ganz langsam heraus. Die Diffusion könnte man evtl durch behutsames Erhitzen im Vakuum beschleunigen. 110 °C würde ich einer Folientastatur nie zumuten wollen. Danach sind die Kontakte wohl unbrauchbar. --Rôtkæppchen68 19:13, 5. Jan. 2014 (CET)
Also bevor man das Ding wegwirft, würde ich wie Lexxx oben eine schonende "Niedertemperatur-Therapie" versuchen - entweder im Keller neben die Heizung, oder auch vor einen mittelheiß eingestellten (noch mit der Hand gut ertragbar) Heizlüfter stellen. Und zwar für 24 h oder länger. Das kostet beim Heizlüfter natürlich schon locker etwa um die 10 Euro/Tag, was eine Abwägungsfrage zum Neukauf wäre... Aber ich würd die Chance recht hoch einschätzen, dass sie danach wieder geht. Solaris3 (Diskussion) 21:13, 5. Jan. 2014 (CET)
1) Löschpapier, Küchenkrepp in Schnipseln an die Flutstelle bringen. 2) Es saugt und bläst der Heinzelmann... Hoher Luftvolumenstrom trocknet recht gut auch an schwer zugänglichen Stellen. --M.Bmg 21:49, 5. Jan. 2014 (CET)
Bau sie mal raus und schau, was auf der Rückseite an Aufklebern und Nummern steht. Füttere damit eine Suchmaschinen (auch von Verkaufsportalen) und mit etwas Glück ist es mit 20 … 40,-- erledigt. Achte aber auf das Sprachlayout QWERTY QWERTZ usw. --Hans Haase (有问题吗) 14:07, 6. Jan. 2014 (CET)

Regio SBahn Lübeck

ich vermisse einen Wikepedia Artikel zur geplanten Regio SBahn Lübeck

--217.6.52.114 15:56, 5. Jan. 2014 (CET)

Dann schreib einen! So funktioniert das hier!!! (nicht signierter Beitrag von 188.97.128.157 (Diskussion) 16:20, 5. Jan. 2014 (CET))

Ein Artikel zur S-Bahn Lübeck wurde anno 2007 wegen Glaskugelei gelöscht. Für den Fall, dass sich seither die Sachlage geändert hat, etwa, weil Mittel für das Projekt bereitgestellt wurden, es eine konkrete Bauplanung gibt, etc., kann ein Artikel erstellt werden, aber nur dann. Über Spekulationen ohne konkrete Aussicht auf Realisierung gibt's bei uns keine Artikel. --88.73.52.20 16:23, 5. Jan. 2014 (CET)

S-Bahn Lübeck wurde wie Du dort sieht mehrfach gelöscht. Das würde nicht in WP:RK passen. Um das zu erfüllen könnte höchstens im Artikel S-Bahn ein entsprechender Abschnitt erstellt werden, der die geplanten Projekte in ihren wesentlichen Kriterien abhandelt. Ich denke alles andere fällt unter WP:WWNI. --Hans Haase (有问题吗) 16:35, 5. Jan. 2014 (CET)
Hier Steht was von Konzept 2020. Wenn sich ein oder zwei Jahre vorher heraus kristallisiert das das was wird bitte wiederkommen. --Mauerquadrant (Diskussion) 18:23, 5. Jan. 2014 (CET)
Deswegen ist sie zur Stunde nicht gebaut. Das bedeutet keinen eigenen Artikel. --Hans Haase (有问题吗) 20:36, 5. Jan. 2014 (CET)

Lied gesucht

Hallo! Mal wieder die Suche nach einem Ohrwurm. Hymnenartiger Pop aus den 70ern, englischer Text, Stil Donovan und Cat Stevens, einziges Prägnante an was ich mich erinnere ist das langezogene Wort "Ocean", jedoch ob into, under oder over, keine Ahnung. Weiß jemand, was gemeint sein könnte?Oliver S.Y. (Diskussion) 22:37, 5. Jan. 2014 (CET)

Atlantis von Donovan meinst du aber nicht, oder?--Hokanomono (Diskussion) 23:04, 5. Jan. 2014 (CET)
Nee, oder? "Way down, below the ocean..." Dumbox (Diskussion) 23:06, 5. Jan. 2014 (CET)
Danke vielmals, das wars. Oliver S.Y. (Diskussion) 23:56, 5. Jan. 2014 (CET)

Material verdichten

Mal eine technische Frage. Angenommen man presst mit einer großen schweren Exzenterpresse das Material, das zum Zusammenbau von A-Batterien verwendet wird, auf das Volumen einer AAA-Batterie zusammen und baut damit eine AAA-Batterie, die aber so viel Masse enthält wie eine A-Batterie. Funktioniert diese verdichtete Batterie dann? Den Atomen dürfte so eine geringe Verdichtung doch eigentlich egal sein.

Oder mal anders herum gefragt, könnte man ein Stück Würfelzucker zum Süßen von Nahrungsmitteln verwenden, das durch Zusammenpressen von 10 Stück Würfelzucker angefertigt wurde?

(Zu Vernachlässigen sei hier der Umstand der Erhitzung oder anderer technischer Probleme, da die Presse über eine Kühlung verfüge und hinreichend langsam presse, wobei es zu keiner termisch bedingten chemischen Reaktion komme.)2001:4C28:4000:721:185:26:182:29 23:45, 5. Jan. 2014 (CET)

Wenn der Druck nachlässt, dehnt sich das meiste Material wieder aus, außer es war vorher sehr porös. Mehrere Würfelzucker zu der Größe eines einzelnen komprimieren kann also gut klappen (ob das Ergebnis dann zusammenhält, ist eine andere Frage, allerdings kann man auch gleich massiven Zucker nehmen), mit der Batterie sieht es aber schlecht aus. Die hat keine freien Hohlräume, die man einfach mal so entfernen könnte. --mfb (Diskussion) 23:58, 5. Jan. 2014 (CET)
Nur teilweise korrekt. Die festen Teile der Batterie bestehen sogar zu einem großen Teil aus Hohlraum, um die Oberfläche der Elektroden zu vergößern. Durch Zusammenquetschen verkleinert man die Hohlräume wieder, wodurch die Batterie aus gepresstem Material eine geringere Energiespeicherfähigkeit hat. -- Janka (Diskussion) 00:09, 6. Jan. 2014 (CET)
Im Falle des Würfelzuckers würde das Zusammenpressen nur begrenzt funktionieren. Selbst wenn Du es schaffst, sämtliche Luft aus dem Zucker herauszupressen, kommst Du nicht auf mehr als 1570 kg/m³. Das Ergebnis wäre polykristalliner Kandiszucker. In der Handhabbarkeit wäre so ein Zucker eingeschränkt, denn er wäre vermutlich brüchig und würde sich aufgrund der geringen Oberfläche nicht so schnell auflösen, hätte aber sonst keinen höheren Nutzwert als dieselbe Masse üblichen Würfel- oder Kristallzuckers. Die allermeisten Feststoffe sind so gut wie inkompressibel. Man braucht also immens hohe Drücke, um sie auch nur ein bisschen zusammenzuquetschen. Sobald der Druck weggenommen wird, dehnen sich die Stoffe wieder auf ihr voriges Volumen aus. --Rôtkæppchen68 00:22, 6. Jan. 2014 (CET)
In der Regel werden die "Hohlräume" der Batterie aber durch eine Flüssigkeit ausgefüllt, und diese Hohlräume mit der Flüssigkeit sind relevant für den Betrieb der Batterie. --mfb (Diskussion) 00:26, 6. Jan. 2014 (CET)
Und das Problem bei Batterien/Akkus ist heutzutage auch weniger das Volumen als das Gewicht bzw. die benötigte Materialmenge an z. T. teuren Stoffen. --PeterFrankfurt (Diskussion) 02:26, 6. Jan. 2014 (CET)
Wenn ein Geräteentwickler will, dass dessen Batterie besonders lange hält, dann verpasst er den Gerät gleich Lithium- oder Silberoxidbatterien und keine AA- oder AAA-Rundzellen. Bei AA- und AAA-kompatiblen Rundzellen verschenkt man die Hälfte des möglichen Energieinhalts an die übliche Nennspannung von 1,5 Volt gegenüber 3 Volt bei von vorneherein als Lithiumbatterie konzipierten Batterien (z.B. CR2032). --Rôtkæppchen68 06:43, 6. Jan. 2014 (CET)
Die meisten Feststoffe und Flüssigkeiten sind praktisch inkompressibel (es sei denn du hast Schäume, lockeres Pulver, poröses Material oder dergleichen), da wäre so etwas schon vom Prinzip nicht möglich. Bei Funktionsmaterialen kommt noch hinzu das sich deren Eigenschaften unter Druck ändern bzw. chemische Reaktionen ausgelöst werden können.--Antemister (Diskussion) 11:34, 6. Jan. 2014 (CET)

Gefahrengebiet in Hamburg

Kann mir jemand erklären, wie ein "Gefahrengebiet"[8] genau definiert ist, welches die Rechtsgrundlage dafür ist, was die Polizei dort tun darf was sie sonst nicht darf, und ob es vergleichbares auch in anderen Bundesländern gibt? Der Artikel Gefahrengebiet hilft leider, ähm, "nicht so ganz" weiter. --80.187.102.179 20:10, 3. Jan. 2014 (CET)

PolDVG § 4 Abs. 2. --Vsop (Diskussion) 23:26, 3. Jan. 2014 (CET)

Da kommt aber weder dieser Begriff vor, noch wird aus dem Text klar, was der Unterschied zu den normalen Befugnissen der Polizei ist. --80.187.102.152 23:38, 3. Jan. 2014 (CET)

(BK) Das (daß der Artikel Gefahrengebiet weiterhilft) kannst du bei der deutschen Wikipedia und dem einflußreichen Hamburger Stammtisch auch nicht unbedingt erwarten. Die Rechtsgrundlage für die vermeintlichen Gefahrengebiete in Hamburg ist seit 2005 im Gesetz über die Datenverarbeitung der Polizei (HmbPolDvG) versteckt. Dort heißt es im § 4 II HmbPolDvG: „Die Polizei darf im öffentlichen Raum in einem bestimmten Gebiet Personen kurzfristig anhalten, befragen, ihre Identität feststellen und mitgeführte Sachen in Augenschein nehmen, soweit auf Grund von konkreten Lageerkenntnissen anzunehmen ist, dass in diesem Gebiet Straftaten von erheblicher Bedeutung begangen werden und die Maßnahme zur Verhütung der Straftaten erforderlich ist.“ In einem Kommentar von 2011 im Newsletter Presback lese ich: „Seit 2005 wur­den in Ham­burg etwa 40 Ge­fah­ren­ge­bie­te ein­ge­rich­tet. Ei­ni­ge von ihnen be­tref­fen lang­fris­tig ganze Vier­tel, an­de­re wer­den zu be­stimm­ten An­läs­sen, zum Bei­spiel Demos, für kurze Zeit an­be­raumt.“
Hier ergänzend das hamburger SOG. Die Kernprobleme sind sicherlich nicht nur in der Fragen der Verhältnismäßigkeit und der Verfassungsmäßigkeit zu finden sondern vor allem auch in der Problematik, mit welchen Mechanismen die Intention von Ausnahmerecht in der Praxis durch die normative Kraft des Faktischen zu Gewohnheitsrecht, der Ausnahmefall zum Regelfall verwandelt wird. Auf der anderen Seite überrascht das auch nicht mehr, wenn man bedenkt, daß aktuell die SPD die Bedenken gegen eine große Koalition mit dem Hinweis zu zerstreuen sucht, man habe sich ja auch bereits im Dezember 1966 in eine Große Koalition begeben. Wer genauer hinhört denkt dann natürlich sofort auch daran, daß diese Große Koalition mit 53 Stimmen aus der SPD und gegen den erheblichen Protest eines relevanten Teils der Bevölkerung, der die freiheitliche Demokratie zu schützen suchte, 1969 die unseligen Notstandsgesetze beschlossen hat. Auch dieser Geist ist offenbar noch fruchtbar und einen bei einer Diskussion um sozialdemokratische Staatsraison naheliegenden Bogen von 2014 zu den Kriegskrediten von 1914 führe ich lieber nicht näher aus.
hier eine Diskussion zu den Gefahrengebieten bei jurawelt.com schon aus 2009. Darin verlinkt eine Gutachterliche Stellungnahme zur Verfassungsmäßigkeit von § 4 Abs. 2 HmbPolDVG (Gesetz der Datenverarbeitung der Polizei) von RA Dirk Audörsch (undatiert) und die Bürgerschafts-Drs. 18/1487 vom 14.12.2004 unter dem Aspekt der Genesis der Gesetzesänderung. In dem Abschnitt 2 aa seines Gutachtens schreibt Rechtsanwalt Audörsch: „ (...) Diesbezüglich lässt sich § 4 Abs. 2 HmbPolDVG entnehmen, dass die Polizei im öffentlichen Raum in einem bestimmten Gebiet Personenkontrollen durchführen kann. Wie dieses Gebiet des öffentlichen Raumes umschrieben ist, lässt sich dem Wortlaut nicht entnehmen. (...) Eine Ermittlung des Personenkreises welcher potentiell betroffen ist oder eine konkrete örtliche Eingrenzung des Wirkungskreises der Norm, ist auch nach allen Mitteln der juristischen Auslegungsmöglichkeiten unmöglich.“ Von den hier zusammengestellten Informationen sollte eine weitergehende eigenständige Recherche möglich sein. --84.191.134.175 23:36, 3. Jan. 2014 (CET)

Erstmal danke, obwoh mir immer noch nicht klar ist, was der Unterschied zu den normalen Polizeibefugnissen ist - die nicht mehr vorausgesetzte Notwendigkeit eines konkreten Anlasses für bestimmte Handlungen? Ist das alles? Irgendein Grund findet sich doch immer. --80.187.102.152 23:50, 3. Jan. 2014 (CET)

Siehe auch Dokumente zur Einrichtung des "Gefahrengebiets Sternschanze" am 23. Mai 2013 https://fragdenstaat.de/anfrage/gefahrengebiet-sternschanze/ am Ende. --Vsop (Diskussion) 23:59, 3. Jan. 2014 (CET)
Naja, man würde hoffen, dass bei sagen wir 500 systematisch durchsuchten Personen selbst ein Hamburger NaziRichter irgendwann fragt "und die sahen alle verdächtig aus?" Da besorgt man sich also lieber vorher noch einen Extra-Blankofahrschein --92.202.57.120 00:06, 4. Jan. 2014 (CET)
@ IP 80.187.: Das Kernproblem ist die legitimierte Unbestimmtheit und Beliebigkeit. Ansonsten muß eine Maßnahme konkret hinsichtlich der Person gerechtfertigt werden. Du hast sicher recht, ein Vorwand findet sich immer, aber die Übergriffe können ansonsten konkreter hinterfragt und angegangen werden als bei einer Generallizenz der Verdächtigung. (Beipiel: nachträglich erfolgreiche Klagen gegen Einkesselung.) Der Grund oder Vorwand wird ja nicht mehr mit der konkreten Person sondern über das von ihre frequentierte Gebiet hergestellt. Du bist also nicht mehr aus deiner Person heraus verdächtig sondern weil du irgendwo bist und für diesen Ort ein Generalverdacht konstruiert wurde. Juristisch m. E. eine höchst fragwürdige Methode. Wer zum Fußballspiel geht ist automatisch verdächtigt, ein Hooligan zu sein, wer über einen Platz mit Junkies gehen muß ist automatisch des Drogenmißbrauchs verdächtigt. Manchmal weißt du garnicht mehr, daß du gerade verdächtigt wirst und was für Folgen das hat (Videoüberwachung, Eintrag in Datenbanken etc.). Was soll zum Beispiel jemand machen, der auf dem Weg zur Arbeit immer an Junkies vorbeigehen muß und in dessen Akte dann ohne sein Wissen steht: "Hält sich häufig in Gefahrengebieten (BTM-Mißbrauch) auf". Daß du den Begriff "Gefahrengebiet" nicht findest ist eine verständliche, faktisch jedoch unsinnige juristische Fliegenbeinzählerei. Der Senat verfügt mit der Formel des SOG "(...) zum Schutz der Allgemeinheit oder des Einzelnen erforderlichen Maßnahmen, um bevorstehende Gefahren für die öffentliche Sicherheit oder Ordnung abzuwehren oder Störungen der öffentlichen Sicherheit oder Ordnung zu beseitigen (Maßnahmen zur Gefahrenabwehr)." über eine Leerstelle, in die nach Belieben erstmal alles reindefiniert werden kann und bis zu den letzten Instanzen ist es ja dann immer ein weiter und auch sehr langer Weg... Wenn es um die Wurst geht wird die "Sicherheits"politik immer gerne sehr unbestimmt. Man denke nur an die Formel "Gefahr im Verzug", mit der in der Praxis dann der Art. 13 GG ausgehöhlt wurde. --84.191.134.175 00:12, 4. Jan. 2014 (CET) Nachtrag: Noch ein Satz zu der Fragwürdigkeit des Vorgehens: Aus meiner Sicht hat der Staat überall Rechtssicherheit herzustellen. Nun wird aber argumentiert: In bestimmten Gebieten kann ich das nicht mit den zur Verfügung stehenden Mitteln, also möchte ich da ein Ausnahmerecht haben. Man ändert also das Recht, statt für die nötige Ausstattung (z.B. Personalschlüssel) zu sorgen. Weil die polizeilichen Maßnahmen nicht an den Ursachen ansetzen (in der Anti-Drogen-Politik meist ohnehin nur in andere Gebiete verdrängen) und die ansonsten tatenlose Politik sich dahinter verschanzt, es geschehe ja bereits etwas, wird das als Ausnahmerecht etablierte Gesetz in einem bedenklichen schleichenden Prozeß zum alltäglichen Gewohnheisrecht. Die Entrechtung wird nicht ins Bewußtsein gehoben und vor der Verabschiedung der Gesetze gesellschaftlich kontrovers diskutiert sondern durch den scheinbar unbehebbaren Mißstand dauerlegitimiert. Wenn das nicht ausreicht, macht man noch zusätzlich Angst, z.B. vor rumänischen Arbeitskräften oder so... --84.191.134.175 00:37, 4. Jan. 2014 (CET)
Hauptsache der Terror hört wieder auf. 3x vor der Haustür gewesen heute, zweimal durchsucht worden. Meine Bereitschaft zur Kriminalität im Beleidigungsbereich ist spürbar gestiegen. --Hinnerk11 (Diskussion) 03:08, 5. Jan. 2014 (CET)
Es sieht aber eher so aus, daß wohl noch längere Zeit der Polizeistaat geübt werden soll. Es gibt aber offenbar auch viele Stimmen, die die Prinzipien der Gewaltenteilung mißachtet sehen, weil es ja keinerlei richterliche Prüfung oder Anordnung gibt und die Polizei sich nach eigenem Gutdünken selbst kontrolliert. Zunächst muß man wohl abwarten, ob rechtliche Schritte gegen diesen Skandal Erfolg haben. --84.191.138.77 22:46, 6. Jan. 2014 (CET)

Entfernungsfrage

Ich wüsste gern, welche beiden Punkte auf der Erde, die miteinander Sichtkontakt haben, am weitesten voneinander entfernt sind. Ich vermute, dass das zwei Hochgebirgsgipfel sind (Himalaya?); Wolkenkratzer oder Fernsehtürme gelten auch, dürften aber wohl in diesem Sinne nicht an Berge heranreichen. Also, gesucht ist ein Punkt A, von dem aus ich bei idealen Sichtbedingungen (mit Fernglas) einen möglichst weit entfernten Punkt B - und umgekehrt - wahrnehmen kann. Gemäß der Formel in Horizont wäre etwa der Horizont vom Gipfel des Mount Everest knapp 340 km entfernt; die gesuchten beiden Punkte sollten dann noch knapp über diesem Wert voneinander entfernt liegen. --Proofreader (Diskussion) 20:19, 3. Jan. 2014 (CET)

Auf viewfinderpanoramas.org wird auch darüber nachgedacht. Die Seite schätzt sogar gut 500 km zwischen der westlichen Andenkette und der Sierra Nevada de Santa Marta in Kolumbien - allerdings nur theoretisch und mit Hilfe von Lichtbeugung (siehe hier). --80.218.227.50 21:48, 3. Jan. 2014 (CET)
Na, da werden ja durchaus auch reale Beobachtungen angegeben. Die 532 km etwa in Bezug auf die Berge Alaskas erscheinen durchaus noch im Bereich des Möglichen, wenn ich mir das wirklich unglaubliche von Thorsten verlinkte Bild aus 270 km Entfernung anschaue, wo wirklich jeder Gipfel absolut klar zu erkennen ist. Bei den 750 km in Turkestan lässt der Text ja leichte Zweifel durchblicken, will das aber auch nicht ausschließen. --Proofreader (Diskussion) 03:29, 4. Jan. 2014 (CET)
In Berliner Fernsehturm steht drin: Bei guter Sicht reicht der Blick bis zum rund 70 Kilometer entfernten Freizeitpark Tropical Islands. Wer bietet mehr? Vergiss aber nicht die Sichtbarkeitsbeschränkungen der Erdatmosphäre. Siehe auch Sphärische Trigonometrie. --sk (Diskussion) 21:36, 3. Jan. 2014 (CET)
270 km. --тнояsтеn 21:47, 3. Jan. 2014 (CET)
Vom Mount Kenya zum Kilimandscharo: 323 km. --тнояsтеn 21:53, 3. Jan. 2014 (CET)
In Eiffelturm: Bei gutem Wetter kann man von hier aus bis 80 Kilometer weit sehen. --sk (Diskussion) 22:22, 3. Jan. 2014 (CET)
In Burj Khalifa: Die Gebäudespitze lässt sich bei klarer Sicht aus über 100 Kilometern Entfernung erkennen. Berggipfel scheine für die Frage interessanter zu sein. --sk (Diskussion) 22:24, 3. Jan. 2014 (CET)
Im ersten Augenblick hatte ich an Dominanz (Geographie) der Berggipfel gedacht, aber das scheint für den Sichtkontakt nur nebensächlich. -- sk (Diskussion) 22:28, 3. Jan. 2014 (CET)

Im Juni 1906 hat Robert Edwin Peary vom Gipfel des Cape Thomas Hubbard aus das geschätzt 130 Meilen (210 km) entfernte Crocker Land gesehen (-> en:Crocker Land Expedition) :-) --80.187.102.152 23:22, 3. Jan. 2014 (CET)

Ende des 19.Jh. wurde bei einer Triangulation das Mittelmeer überbrückt. Das Netz umfasste zwei Stationen in Algerien und zwei in Spanien. Von jeder Station aus wurden die anderen angepeilt. Es ergaben sich maximale Peilstrecken von 300 bzw. 270 km: http://www.nature.com/nature/journal/v20/n518/abs/020541a0.html. Die genaue Länge der Strecken müsste in der Literatur ermittelbar sein. --Dioskorides (Diskussion) 00:13, 4. Jan. 2014 (CET)
Dort a.E. wird eine Peilung über eine Distanz von 309 km erwähnt. --Hermine Tuzzi (Diskussion) 09:21, 4. Jan. 2014 (CET)

Das sind doch schon mal interessante Antworten. Die Idee zur Frage kam mir übrigens tatsächlich beim Blick auf den Berliner Fernsehturm vom Stadtrand aus - was Türme und Gebäude angeht, können der Berliner Fernsehturm und der Eiffelturm gegen den Burj Khalifa nicht wirklich anstinken, aber wie ich schon gedacht hatte, sind Berge da eindeutig im Vorteil. Also, die genannten Werte 270 km, 323 km, 210 km, bis 300 km zeigen, dass bei optimaler Sicht der geometrische Horizont tatsächlich beobachtet werden kann. Was die Verringerung des Kontrastes in der Entfernung angeht, ließe sich das durch folgendes Experiment umgehen: Man installiere auf den Berggipfeln etwa besonders starke Scheinwerfer oder Lasergeräte, dann müsste nicht die Geländekontur selbst erkannt werden und man hat trotzdem den in der Frage geforderten Sichtkontakt. --Proofreader (Diskussion) 03:04, 4. Jan. 2014 (CET)

Hier in der Gegend kann man beispielsweise von Herchenhain im Vogelsberg aus bis zum Donnersberg in der Pfalz sehen, das sind rund 140 km. --PeterFrankfurt (Diskussion) 03:36, 4. Jan. 2014 (CET)
Ich habe mal als Kind von einem Berg in der Nähe von Malaga aus den Hafen von al-Hoceima in Marokko erblickt, das dürften etwa 175 km sein; allerdings ging das nur, weil da eine bestimmte Wetterlage eine Art Luftspiegelung hervorrief, genau weiß ich das nicht mehr. --Hermine Tuzzi (Diskussion) 09:16, 4. Jan. 2014 (CET)
Vom Feldberg im Taunus aus kann man immer wider mal (vor allem im September) die Alpenkette sehen, ebenso wie von Arber oder Lusen im Bayrischen Wald aus. Luftlinie in km? Gruß -- Dr.cueppers - Disk. 11:01, 4. Jan. 2014 (CET)
Nur mal so als Anhaltspunkt, ab welchen relativen Höhen man suchen muss bzw. wo man sich die Suche gleich sparen kann: Wenn zwei Berge mit Höhen im Meer stehen und man vom einen Gipfel gerade noch den Gipfel des anderen sehen kann, dann ist ihre Distanz (ohne Lichtbeugung und sonstige Schikanen) mit R Erdradius (siehe [9]). Für R=6370 km ergibt das bei je 8 km Höhe eine Sichtweite von 638 km, bei je 4 km Höhe eine Sichtweite von 451 km. Wenn der zweite Punkt auf Meereshöhe ist, dann jeweils nur die Hälfte. Um die in [10] genannten 506 km zu übertreffen, bräuchte man also, wenn sie gleich hoch sind, schon zwei Berge mit mindestens 5025 Metern Höhe. Entsprechend höher, wenn zwischendrin nicht das Meer, sondern eine höhere Landfläche liegt. Der genannte Pico Ojeda hat 5490 m, die Sierra Nevada de Santa Marta 5770 m. --Grip99 01:24, 7. Jan. 2014 (CET)
Weil der Link noch nicht kam (das Lemma ist wohl nicht so naheliegend): Erdkrümmung. --Neitram  11:49, 7. Jan. 2014 (CET)

"sollte"-Präteritum... enzyklopädarisch?

Häufig wird in Erzählungen ein Vorausgriff in die Zukunft oder auf eine abschließende Analyse mit einer "sollte"-Konstruktion vorgenommen, um zum Ausdruck zu bringen, dass das zum Zeitpunkt des Geschehens noch nicht klar war ("Dies sollte sein letzter Ausflug nach Südamerika sein"). Was sind eigentlich die Hintergründe dieser Formulierung, wo kommt sie her? Und: Ist es in Ordnung, sie hier in Wikipedia zu verwenden? Insofern ich sie in der Vergangenheit verwendet habe, wurde sie von anderen Usern geändert, nicht zuletzt mit dem teils berechtigten Hinweis auf Zweideutigkeit - das "sollte" kann auch ausdrücken, dass es dann doch nicht so kam ("Er sollte in seines Vaters Fußstapfen treten, doch..."). --KnightMove (Diskussion) 22:02, 3. Jan. 2014 (CET)

Die Formulierung ist m. E. eindeutig und, wenn richtig gebraucht, nicht zu beanstanden. Allerdings (und dieses allerdings ist schwerwiegend) wird die Konstruktion wohl von vielen Leuten schlichtweg nicht verstanden. Aus diesem Grund ist sie möglichst zu vermeiden. --AndreasPraefcke (Diskussion) 22:08, 3. Jan. 2014 (CET)

Ist ähnlich problematisch wie ein Finalsatz in derselben Funktion: "Er schaute kurz vorbei, um bald darauf wieder nach Hause zu gehen." Habe mich auch immer gefragt, wo das herkommt. Hat es mal als Übersetzung eines lateinischen Participium futuri activi angefangen? --Laurentianus (Diskussion) 23:44, 3. Jan. 2014 (CET)
Man kann ja unmißverständlich präzisieren: "sollte in der Tat; sollte tatsächlich; sollte de facto; sollte dann auch; usw" und "um dann auch wieder nach Hause zu gehen; um dann auch wirklich(?)" oder so .. allerdings schaut man ja auch nicht kurz vorbei, weil man sonst nicht danach wieder nach Hause könnte, normalerweise .. O.-o "sollte sein letzer Ausflug sein" könnte so für sich auch heißen, "daß es möglichst nicht noch mehr werden sollten" (lt. Arbeitgeber; Eltern; heimgebliebener Frau mit Kindern; Anweisung des Orakels; ..) oder, daß es der letzte werden würde, weil er auf dem Ausflug umkommt .. Man sollte nicht isolierte Sätze ohne ihren Kontext künstlich mißverstehen. Sprache ist von Natur aus dehnbar und bedarf immer der Übereinkunft der Beteiligten, sollte sie auch noch so logisch aufgebaut erscheinen. "Dies sollte sein letzter Ausflug nach Südamerika  w e r d e n."  bestärkt wiederum o.g. Zweifel, während "hätte es werden sollen." offen läßt ob der Ausflug danach geschah oder weitere folgten. --217.84.114.250 18:09, 4. Jan. 2014 (CET)
Da ist was dran. Ich meinte mit meinem Statement auch eher "ist für mich eindeutig, wenn der Kontext eine andere Interpretation ausschließt". Man kann das sicherlich auch zweideutig verwenden. --AndreasPraefcke (Diskussion) 14:10, 5. Jan. 2014 (CET)
Ich könnte mir vorstellen, dass es mit der Annahme eines Schicksals (von göttlichen Mächten vorherbestimmt) zu zun hat. ("Es hat nicht sollen sein.") --Neitram  11:57, 7. Jan. 2014 (CET)

Lichtgeschwindigkeit

Bestimmt ganz einfach erklärt, aber ich bin ja nicht der einzige, dem es bei Lichtgeschwindigkeit an Vorstellungsvermögen fehlt: Wenn ein Objekt zwei Lichtjahre weit weg ist, sehe ich das Objekt zwei Jahre in der Vergangenheit. Was sehe ich aber, wenn sich das Objekt direkt auf mich zubewegt, also ein Jahr später schon ein Stück näher war?

--85.180.197.116 10:10, 4. Jan. 2014 (CET)

„sehe ich das Objekt zwei Jahre in der Vergangenheit“ ist physikalisch gesehen ungenau, da es keinen absoluten (d.h. bezugssystem-unabhängigen) Zeitmassstab zur Bestimmung dieses Zeitunterschieds gibt. Aber das war ja nicht die Frage. Das Bild des Objekts, das Du siehst ist unabhängig von seiner Geschwindigkeit, wenn es sich direkt auf Dich zubewegt. Es ergibt sich lediglich eine Farbverschiebung durch den Dopplereffekt.--Belsazar (Diskussion) 10:23, 4. Jan. 2014 (CET)
die Frage ist etwas ungenau, da zu einer genaueren Berechnung die "angenommene" Geschwindigkeit des Lichtgebenden Objektes fehlen würden dürfte. --80.108.60.158 10:37, 4. Jan. 2014 (CET)
Ich will es ja gar nicht berechnen. Ich wollte nur verstehen, ob ich dann ein 2 Jahre altes oder ein 1 Jahr altes Objekt bzw. irgendetwas dazwischen sehe, oder alles auf einmal. Aber das ist, wie Belsazar andeutet, wahrscheinlich einfach zu naiv gedacht. Vielen Dank jedenfalls! 85.180.197.116 11:32, 4. Jan. 2014 (CET)
Du siehst ja gar nicht das Objekt selbst sondern nur das Licht, das es ausstrahlt. Selbst wenn sich das Objekt rasend schnell auf Dich zubewegt, ist es immer noch langsamer als das Licht und Du siehst immer nur das Licht, was es in der Vergangenheit ausgestrahlt hat. Erst nach zwei Jahren wird sich das ändern, vorausgesetzt es stand bis vor 2 Jahren still. Das Licht wird dann eher bei Dir sein, wenn es auf Dich zurast, aber das kannst Du ja nicht sehen. --91.0.145.182 12:09, 4. Jan. 2014 (CET)
nach BK: Du siehst das Objekt an sich genauso als ob es in Ruhe wäre (auf einem Foto kannst du auch nicht untersscheiden ob und wie schnell sich ein entgegenkommendes Auto in deine Richtung bewegt). Du nimmst freilich den Dopplereffekt war (und sofern sich das Objekt schnell genug bewegen würde, würdest du bei einer Wiederholungsbeaobachtung feststellen, dass sich seine Position relativ zu anderen Objekten verändert hat).--Antemister (Diskussion) 12:22, 4. Jan. 2014 (CET)
Aussage der IP drüber kommt natürlich auch noch dazu, wollte ich auch noch dazuschrieben.--Antemister (Diskussion) 12:22, 4. Jan. 2014 (CET)
Wenn sich das Objekt z.B. ein Erdjahr lang mit halber Lichtgeschwindigkeit auf Dich zubewegt hat, also nur noch 1 1/2 Lichtjahre entfernt ist, dann siehst Du dasjenige Licht, was 1 3/4 Jahre zuvor vom Körper kam. Wobei dieses "wenn, dann" sich natürlich auf Dein Koordinatensystem auf der Erde bezieht, siehe Relativität der Gleichzeitigkeit. --Grip99 01:20, 7. Jan. 2014 (CET)

Ist doch nicht so schwierig. Bei einer Entferung von 2 Lichtjahren siehst Du das Objekt, wie es vor 2 Jahren ausgesehen hat. Wenn es sich auf eine Entfernung von einem Lichtjahr genähert hat, siehst Du es, wie es vor 1 Jahr ausgesehen hat und bei der Entfernung dazwischen eben dem Alter zwischen einem und zwei Jahre. Je nach Geschwindigkeit um einen Bruchteil früher als 2 Jahre, also 2 Jahre minus x Sekunden. --Ohrnwuzler (Diskussion) 02:09, 5. Jan. 2014 (CET)

Das kann trotzdem zu interessanten Konsequenzen führen. Wenn wir sehen wo es vor 2 Jahren war und es mit 90% der Lichtgeschwindigkeit auf uns zukommt, dann werden wir in gut einem Monat sehen, wo es vor einem Jahr war, und im März sehen wir wie es im März aussieht (weil es dann die Erde erreicht hat). Dopplereffekt bedeutet auch, dass wir Prozesse auf dem Objekt beschleunigt sehen, und dass es so wirken kann, als wäre es schneller als Licht (weil unser Bild davon in 2,5 Monaten 2 Lichtjahre näher kam). Zwar nicht bei Sternen, aber bei Jets, ist das ein beobachteter Effekt. --mfb (Diskussion) 03:43, 5. Jan. 2014 (CET)


Man sieht das 2-Jahre alte Licht, das das Objekt zeigt, wie es vor 2 Jahren war. Wenn sich das Objekt auf einen zubewegt, sieht man das Bild, das hier ankommt, schneller altern. Wenn das Objekt schließlich hier ist, stimmt gesehenes und tatsächliches Alter überein. Wenn man zum Beispiel einen Mann in 20 Lichtjahren Entfernung erblickt, dann sieht man zunächst den Mann von vor 20 Jahren (jünger), obwohl er zum Zeitpunkt, an dem man das Licht von ihm sieht, schon 20 Jahre älter ist. Wenn sich der Mann nicht bewegt, altert er gesehen normal. Wenn er auf einen zukommmt, dann altert das Bild schneller und wenn er hier ist, sieht er genau so alt aus wie er ist. 46.115.99.236 20:54, 6. Jan. 2014 (CET)

Trimalchio und Bedeutung von 'malchio'

Moin, der Name des aus dem Satyricon von Petron stammende Trimalchio wird in der englischen Wikipedia so hergeleitet: "The name Trimalchio is formed from the Greek prefix τρις and the Semitic מלך (melech) in its occidental form Malchio or Malchus. The fundamental meaning of the root is "King," and the name Trimalchio would thus mean "Thrice King," "greatest King." (hier). Jetzt habe ich hier allerdings einen Verweis (hier) gefunden, der das malchio von einem griechischen Wort mit der Bedeutung 'geschmacklos, unangenehm, widerlich' herleitet. Ist dieser Weg der Wortbedeutung auch möglich? Vielleicht kann mir jemand helfen, denn ich habe keine Ahnung von Griechisch.

--Involuntarius (Diskussion) 12:22, 4. Jan. 2014 (CET)

Hermine??? An der zitierten Stelle wird das Wort nicht aus dem Griechischen hergeleitet, sondern als lateinisches Pendant zu dem griechischen aedes bezeichnet. Jetzt müsste man wissen, woher der große Friedländer hier seine Weisheit hat; da müsste man genauer nachforschen. Die weiteren Herausgeber scheinen ihm ja offenbar nicht gefolgt zu sein. Denkbar scheint mir, dass M/malchio als Ethnophaulismus für einen "Ossi", eben als typischer Name, diese negative Bedeutung angenommen haben könnte. Dann wäre der Ursprung aber immer noch semitisch, und die Groß- oder Kleinschreibung Geschmackssache. Grüße Dumbox (Diskussion) 13:12, 4. Jan. 2014 (CET)
Etymologisch möglich ist vieles; der Name Malchus ist ja bekannt: so hieß der, dem Petrus das Ohr abhieb, und wenn ich mich recht entsinne auch ein Karthager aus den Punischen Kriegen. Das würde zu einer Herkunft von der Wurzel mlk passen. Malchio als Name ist mir noch nicht untergekommen; der Neue Pauly erwähnt ihn s. v. Cognomen (Rix) als semitisches Kognomen (Ich halte es nicht für unmöglich, daß Rix sich auch nur auf Mart. 3,32,82 bezieht), RE habe ich gerade nicht zur Hand. Denkbar wäre m. E. auch, daß malchio irgendein italisch-volkssprachliches Wort ist, das uns nur an dieser Stelle bei Martial und im Namen Trimalchio überliefert wäre; dessen Verwendung als Kognomen wäre ja dann ebenfalls wahrscheinlich. Der große Friedländer – dem neben Obermayer ja immerhin auch der große Katzenliebhaber aus Cambridge in seiner zweisprachigen Ausgabe folgt – schöpft die Bedeutungsangabe ἀηδής aus einer Glosse, nämlich Philox. Mart. 38 bei Lindsay, Glossaria Latina, Paris 1926, Ndr. Hildesheim 1965. Der Thesaurus kennt auch nur diese beiden Fundstellen und stellt keine Vermutungen zur Etymologie an. Schade, daß der Namen-Thesaurus ruht -- der hätte uns zu Trimalchio sicherlich was sagen können. --Hermine Tuzzi (Diskussion) 14:00, 4. Jan. 2014 (CET)
In der Fußnote geht es hautsächlich um Groß-/Kleinschreibung: ist malchio ein Appellativum oder ein Eigenname? Übersetzt wird "hos malchionis patimur improbi fastus" schon oben mit: "Diese Arroganz des unverschämten Widerlings müssen wir uns gefallen lassen." .gs8 (Diskussion) 14:09, 4. Jan. 2014 (CET)
Danke für eure Anmerkungen. Mir geht es um die Bedeutung von malchio und nicht um die Groß-/Kleinschreibung. Nun habe ich bei Gottwein noch eine Bedeutung mit "weichlich" gefunden: http://www.gottwein.de/Lat/petron000.php gefunden- μαλακός : malakós , hier wäre die Bedeutung ja nicht nur ein Pendant, sondern eine Herleitung, weil hier auch die Wurzel mlk vorkommt, oder nicht? --Involuntarius (Diskussion) 23:57, 6. Jan. 2014 (CET)

Fachausdruck für ausgehöhltes Buch gesucht

Ich habe Buchtresor, Buchsafe u.ä. für ein ausgehöhltes Buch gefunden. Gibt es dafür einen Fachausdruck? --80.187.109.184 14:14, 4. Jan. 2014 (CET)

Die Begriffe sind doch OK. auch en:WP Hollow book liefert nicht mehr. GEEZER… nil nisi bene 16:06, 4. Jan. 2014 (CET)
"Buchattrappe" nennt man das schlicht. --Kharon 20:12, 4. Jan. 2014 (CET)
Als Buchattrappe hätte ich diese Dinger bezeichnet, die es in Möbelhäusern immer gibt, wenn es so aussehen soll, als stünden Bücher in einem Regal. --88.130.114.151 12:59, 5. Jan. 2014 (CET)
Geheimversteck, Buchversteck, Buchtresor, Buch mit Geheimfach oder schlicht und einfach Versteck. --80.108.60.158 07:40, 5. Jan. 2014 (CET)
Es gibt also keinen Fachausdruck aus dem Bibliothekswesen dafür? --80.187.109.135 18:54, 5. Jan. 2014 (CET)
Bei den Bibliothekaren bist du da eher falsch. Passender wären die Schlapphüte. Die RAF-Anwälte haben übrigens Mitte der 1970er in prinzipiell ähnlichen ausgehöhlten Akten alle möglichen Sachen in den Knast zu ihren Mandanten geschmuggelt, darunter Waffen. Da ist in Presse/ Literatur auch immer nur von "ausgehöhlten/hohlen Akten" die Rede, und das ist glaub ich die bisher bekannteste Anwendung von so etwas gewesen. Solaris3 (Diskussion) 23:47, 5. Jan. 2014 (CET)
Könnte man auch als Toter Briefkasten bezeichnen. -- sk (Diskussion) 16:20, 6. Jan. 2014 (CET)

Herz stehen bleibt einfach so ohne Ankündigung

Ich sehe gerade The Ramen Girl, eine Komödie bei der leckere japanische Nudelsuppen eine gewisse Rolle spielen. Und als ich bei der Schauspielerin Brittany Murphy nachschauen wollte habe ich gelesen, dass sie schon tot ist, weil bei ihr einfach so das Herz stehen geblieben ist. Oh mein Gott das ist ja entsetzlich. Gibt es denn keine irgendwelchen Möglichkeiten, dass nicht bei jeder x-beliebigen Person unangekündigt der Tod einfach so von hinten zuschlagen kann? Wieso kann man in der heutigen Zeit nichts gegen sowas unternehmen? 46.115.139.79 14:29, 4. Jan. 2014 (CET)

eine sehr ernste Frage. Dazu muss man schon auch sagen, dass der Tod zum Leben dazu gehört. Natürlich wäre ein erfülltes Leben mehr als wünschenswert. Ein langsames dahin-siechen in einer Krankheit ist weniger wünschenswert als eine plötzliche Todesart. Ein Herzstillstand kann auf viele verschiedenen Ursachen zurückgeführt werden. Um solchen Dingen präventiv vorzubeugen empfiehlt es sich - in den meisten Fällen, auch äusserst wirksam - einen gesunden Lebenswandel zu führen. Wie so ein gesunder Lebenswandel auszusehen hat, ist Gegenstand zahlreicher Diskussionen. --80.108.60.158 14:48, 4. Jan. 2014 (CET)
Es ist mir nicht interessant genug, das tiefer zu untersuchen, aber der deutsche Artikel Brittany Murphy behauptet derzeit: "Am 4. Februar 2010 teilten die Gerichtsmediziner in Los Angeles mit, dass der Tod auf eine Lungenentzündung, auf die Einnahme mehrerer Medikamente und auf eine Eisenmangelanämie zurückzuführen sei." und belegt dies mit dieser Meldung von AFP vom 4. Februar 2010. Die Behauptung des Fragestellers, daß "bei ihr einfach so das Herz stehen geblieben ist" erscheint mir daher substanzlos und für die daraus abgeleiteten Schlußfolgerungen und Emotionen wäre vielleicht ein anderer Todesfall sinnvoller heranzuziehen. --77.12.35.119 15:05, 4. Jan. 2014 (CET)
Lungenentzündung und Medikamente; im englischen Artikel heißt es momentan "drugs", was auch auf Drogen hinweisen könnte, aber an sich steht das da ganz ähnlich. Warum es jetzt unbedingt nötig ist, die Allerweltsaussage des Leichenbeschauers, dass sie an einem Herzstillstand gestorben sei, zu zitieren, ist mir ein Rätsel. Jeder, der tot ist, hat einen Herzstillstand. --88.130.114.151 15:19, 4. Jan. 2014 (CET)
Die hatte keine Drogen genommen, sie hatte einfach nur eine Erkältung bekommen. Das kann jedem mal passieren und dass man dann Medis nimmt ist auch normal. Das macht jeder. Was für mich nur so krass ist, dass der Tod wirklich ständig sehr sehr nah ist. Sogar bei denen die es überhaupt nicht verdient haben kommt der Tod so überraschend. 46.115.139.79 15:33, 4. Jan. 2014 (CET)
<quetsch> :o Es nimmt nicht jeder Medis. (Schon gar nicht jeder wegen einer Erkältung.) Es ist auch nicht "normal", sondern "medizinisch angezeigt". Wenn Du einfach so Sachen behauptest, darfst Du Dich nicht wundern, daß es Dich wundert lol? .. Gemeint hast Du: Von Erkältungsmedis fällt man ja nicht plötzlich um mit Herztod. --217.84.114.250 18:57, 4. Jan. 2014 (CET) </quetsch>
Ja, das habe ich gemeint. Ich finde es krass dass man einfach tot umfallen kann, ohne sich beispielsweie was zu Schulden kommen lassen zu haben. 46.115.137.210 22:55, 4. Jan. 2014 (CET)
Wer hat in Deinen Augen denn den Tod verdient? Das ist eine sehr seltsame Bemerkung. --91.0.145.182 15:43, 4. Jan. 2014 (CET)
Bei denen die Drogen nehmen kommt das dann wegen Drogen. Oder wer raucht braucht sich auch nicht zu wundern wenn was passiert. 46.115.139.79 16:51, 4. Jan. 2014 (CET)
Wenn also jemand jung seine Mutter verloren hat oder seine langjährige Freundin sich gerade von ihm getrennt hat und er deswegen mal Drogen nimmt, dann sagst du, ist es OK, wenn er stirbt? Sehr komische Weltanschauung - was ist nur los mit dir? --88.130.114.151 17:23, 4. Jan. 2014 (CET)
Das soll man jetzt auch nicht überanalysieren. Ich meinte wenn es jemand schon kommen hat, wo man sagen kann, dass sowieso bald der Mann mit der Kutte an die Tür klopft wie bei Amy Winehouse, die ja noch ein Lied (rehab) darüber gemacht hat, dass sie keine Hilfe annehmen kann, she had it coming. Oder wie bei manch anderem, wo man keinen Wissenschaftler braucht um rauszukriegen was bald kommt. Oder wo man einfach nur denkt mann dass jemand überhaupt so lange durchhalten kann. Das meinte ich. Aber das muss man jetzt nicht moralphilosophisch betrachten. 46.115.139.79 17:43, 4. Jan. 2014 (CET)
Wo wir gerade schon mitten in den wildesten Verschwörungstheorien drin sind, wünsch ich dann schonmal Charles Fox Good bye. Hoffentlich nicht zu hart. --88.130.114.151 19:17, 4. Jan. 2014 (CET)
Der englische Artikel spricht nicht nur von Lungenentzündung und Blutarmut, sondern auch von "multiple drug intoxication", also Arzneimittelvergiftung oder Drogenrausch. Wie man es auch dreht und wendet, dass die ggf. genommene Medizin ordungsgemäß benutzt worden sei, kann man jedenfalls nicht sagen. Wird übrigens langsam ziemlich trollig hier; es bleibt dabei: Warum es jetzt unbedingt nötig ist, die Allerweltsaussage des Leichenbeschauers, dass sie an einem Herzstillstand gestorben sei, zu zitieren, ist mir ein Rätsel. Jeder, der tot ist, hat einen Herzstillstand. --88.130.114.151 17:14, 4. Jan. 2014 (CET)
Sollte kein Arzt in der Nähe sein, dann führt ein Herzstillstand ganz sicher zum Tode. Und hier will auch auch gleich noch Lebensrettende Maßnahmen, die "Erste Hilfe" ins Gedächtnis rufen. Denn eine sachgemäße Herzmassage und einem Notruf per Phone wäre smart. --80.108.60.158 17:30, 4. Jan. 2014 (CET)
Wären sehr smart und können vor allem auch von Nicht-Ärzten durchgeführt werden (das ist ja gerade Sinn der Sache). Dass also ausschließlich ein Arzt nach einem Herzstillstand den Tod vermeiden kann, würde ich so nicht unterschreiben. Aber andersrum ist es natürlich wahr: Wer tot ist, der hat auch einen Herzstillstand. Insofern ist es bei einem Toten witzlos zu sagen, er habe einen Herzstillstand - natürlich hat er den! --88.130.114.151 19:17, 4. Jan. 2014 (CET)
Man muß sich nicht absichtlich mißverstehen. Der Fragesteller schreibt: "(...) weil bei ihr einfach so das Herz stehen geblieben ist." Daraus wird in der ersten Antwort "Herzstillstand", verlinkt auf den gleichlautenden Artikel. Tatsächlich geht es aber um eine Insuffizienz, umgangssprachlich als Herzversagen bezeichnet. Das kann man in der Diskussion unter AFG mitdenken. (Wenn man will.) Ich verstehe auch nicht mehr, in welche Richtung (=> Erkenntnisgewinn) gerade geredet wird. Der Fragesteller ist im konkreten Fall von falschen Voraussetzungen ausgegangen und im Allgemeinen gehört ein Diskurs über die Unausweichlichtkeit des Todes und den Umgang mit der eigenen und der Anderen Endlichkeit ins Cafe, oder? --84.191.149.187 22:25, 4. Jan. 2014 (CET)
(BK) Darauf wollte ich auch gerade hinweisen. Was den Fragesteller so erschüttert hat, war sein (wie oben nachzulesen, unzutreffender) Eindruck, bei einem jungen, gesunden Menschen sei ohne erkennbaren Grund plötzlich das Herz stehengeblieben. Aber es ist sicher nicht verkehrt, sich hin und wieder einmal zu vergegenwärtigen, dass es für jede/n von uns im nächsten Moment vorbei sein kann – aus welchem Grund auch immer. --Jossi (Diskussion) 22:56, 4. Jan. 2014 (CET)
Genau das meine ich, das ist ja das Makabere. Es kann für jeden sofort und ohne Angabe von Gründen zu Ende sein. Sogar so eine liebe Schauspielerin, die eben noch einen harmlosen Film über Nudelsuppe gemacht hat und dann schaut man in den Artikel: ist schon weg, weil mit 30 tot umgefallen. 46.115.137.210 23:10, 4. Jan. 2014 (CET)
Ja, es gibt Sternenkinder, die sich nichts haben zu schulden kommen lassen, und andere werden 100, obwohl sie rauchen, trinken und Burger und Fleisch nur so in sich reinschaufeln. Was soll man machen? --88.130.114.151 00:42, 5. Jan. 2014 (CET)
Bazon Brock ist 1936 geboren und lebt noch.
Nicht mit 30, mit 32. Franz Schubert starb mit 33 Jahren. Und Georg Büchner starb schon mit 23 Jahren, Sid Vicious mit 22, Jim Morrison mit 27, Franz Kafka mit 41 Jahren, Alexandra und Janis Joplin mit 27, die Monroe und Bob Marley mit 36, Amy Winehouse mit 28, James Dean mit 24, Kurt Cobain und Jimi Hendrix mit 27, Fassbinder mit 37, Diana mit 36 und Mozart mit 35. Und Jesus soll angeblich auch bloß 33 Jahre alt geworden sein. Wir müssen alle sterben (und wer es besser weiß schweigt wohlweislich) und ob es einen Grund hat oder nicht hilft da auch nicht weiter. Auch ein äußerer Grund wie Autounfall, Fischgräte, Vulkanausbruch, US-Militärdrohne, elektrischer Stuhl, sich aus dem 13 Stock stürzen oder der legendäre Blumentopf begründet nichts, weil man fragen kann: "Warum ausgerechnet der, warum ausgerechnet ich, warum hat es oder haben sie nicht jemand anderen erwischt?" Wir sterben früher oder später, manche eben früher. Nebbich. Es gibt keine Antwort, es gibt keine Erklärung. Wir sterben und das ist einfach so. Wir wissen viel darüber, unter welchen Voraussetzungen wir sterben, wir wissen ein bißchen darüber, wie wir sterben, wir wissen nicht, warum wir sterben. Wir suchen Halt und Trost, irgendetwas, an das wir uns vermeintlich klammern können, irgendeine Konstruktion wie Religionen oder andere Beliebigkeiten des Glaubens ohne Wissen, verzweifelte Erfindungen, die unsere Ratlosigkeit vor unserer Endlichkeit nur noch deutlicher hervortreten lassen, nichts als seelische und intellektuelle Verrenkungen der Selbstlüge. Auch die Konstruktion der Gerechtigkeit ist eine Konstruktion. Es gibt keine Hoffnung, keinen Ausweg. Nur Selbsthypnose, Verdrängung, Totenkult und Sepulkralkultur. Man kann den frühen Tod als makaber bewerten (lauf mal über einen Kinderfriedhof), als ungerecht, als Strafe der Götter, als Menetekel oder als was auch immer. Alles ist richtig und falsch, je nachdem, wie wir uns in unserer Hilflosigkeit die Dinge zurechtgelegt haben. Das ist genauso beliebig wie die Regel, daß du erst jemand bist, wenn du mehr als 500 Gramm wiegst. Wir schlucken die rote Kapsel und gelangen in "die tiefsten Tiefen des Kaninchenbaus". Dort sehen wir: Das einzige, was der Tod uns gnadenlos lehrt ist, daß es nicht auf alles eine Antwort gibt. So ist das. Und damit muß man leben lernen. Irgendwie. Jeder so, daß er es aushält. --84.191.149.187 00:55, 5. Jan. 2014 (CET)
Er zitiert Matrix. --88.130.114.151 02:28, 5. Jan. 2014 (CET)
Zurück zum Tod: Es stimmt: Vor allem wenn man plötzlich stirbt, weiß man das vorher oft nicht (besagter Blumentopf) und kann man sich insofern auch nicht "gezielt" drauf vorbereiten. Was man aber durchaus machen kann, ist das Risiko zu verringern, an einer bestimmten Ursache zu sterben: Nichtraucher kriegen signifikant seltener Lungenkrebs, Nichtalkoholiker seltener Leberzirrhose usw. Ernähr dich gesund, hol dir vll. eine private Krankenversicherung und mach keinen Mist im Leben (Extremsportarten, gefährliche Dinge, z.B. Train Riding usw.). Pass einfach auf dich auf. --88.130.114.151 02:28, 5. Jan. 2014 (CET)

...Und geh nicht ins Bett, da sterben die meisten.--79.232.219.209 10:27, 5. Jan. 2014 (CET)

Herje und das stimmt auch noch alles! Es ist glaube ich ein Irrtum, dass man für den Tod irgendwie eine Vorbedingung braucht wie "Warte, ich bin noch nicht fertig" oder "Darf ich das noch zu Ende machen?". Sowas ist dem völlig egal. Es gibt weder ein Recht auf Leben noch auf eine Mindestlänge. Seit Milliarden von Jahren sterben die Menschen und es kam noch kein einziges Mal vor, dass der Tod auf irgend etwas Rücksicht genommen hätte. Als allmächtiger Vorgang tritt er einfach ein und ist plötzlich da und macht keinen Unterschied zwischen jung und alt, reich und arm, gut und böse oder fleißig und faul. Es kommen alle dran und manchmal auch welche die herzensgut sind und bei denen es schade ist wenn sie weg sind, weil man wünscht, dass alle so wären wie die, damit die Welt ein besserer Ort ist. Aber selbst das ist im Tod schnurz. Er macht einfach was er will. 46.115.89.171 12:01, 5. Jan. 2014 (CET)
„YOU FEAR TO DIE ?“
„It's not that I don't want... I mean, I've always...it's just that life is a habit that's hard to break...“
― Terry Pratchett, Reaper Man
Ich empfehle (noch) wärmstens, sich mit dem TOD bei Pratchett anzufreunden, damit man nicht überrascht ist, wenn man ihn eines Tages hinter sich im Rückspiegel sieht. Pratchett beschreibt auch das vollkommene Chaos, als der Grim Reaper eines Tages entscheidet, die Sense an den Haken zu hängen... 's passt scho'. Ich kann es kaum erwarten. Warum ich trotzdem auf ein langes Leben hinarbeite? Na ja, delayed gratification (Belohnungsaufschub), die Vorfreude, die lange Vorfreude... GEEZER… nil nisi bene 12:27, 5. Jan. 2014 (CET) Wenn man wirklich mal genau nachdenkt und eine Liste macht, hat "tot sein" viel mehr Vorteile als "lebendig sein"...
Ganz zu schweigen davon, dass zwar (fast) jede/r noch ein bisschen länger leben will, dass aber ein EWIGES Leben, wenn man es genau durchdenkt, so ziemlich die grauenhafteste Vorstellung überhaupt ist. Man muss sich das mal klar machen: ES HÖRT NIEMALS MEHR AUF! 150 oder 200 Jahre lang leben klingt ja ganz nett (vorausgesetzt, man ist noch halbwegs beieinander), bei 10000 oder 50000 Jahren schluckt man schon, aber eine Milliarde Jahre leben? 750 Trillionen Jahre leben? Immer in dem Bewusstsein, dass das ja erst der Anfang ist und es UNENDLICH weitergeht, ob man will oder nicht? Nee, danke ...
Natürlich kann man sich denken, dass man in einer ganz anderen (unzeitlichen, multidimensionalen oder was auch immer) Weise fortexistiert, aber das wäre von unserem jetzigen menschlichen Bewusstsein so fundamental verschieden, dass es nicht vorstellbar ist. Somit bleibt es dabei: Dieses Leben endet, und das ist das einzige, das wir haben. Und es ist gut so. --Jossi (Diskussion) 17:09, 5. Jan. 2014 (CET)
Aber das meinte ich ja auch garnicht so philosophisch mit "Weiterlebenmüssen für immer". Ich finde es nur etwas übertrieben viel Einfluss, wenn der Tod einen schlichtweg überall abholen kann, nicht nur beim Schlafen, sondern dass es schon beim nächsten Gang ins Bad soweit sein kann. Muss das denn sein? Wenn man die gefragt hätte, hätte sie sicher geantwortet, dass sie es nicht will aber dass sie später irgend wann mal schon sterben würde, wenn es soweit ist. Philosophisch mit "für immer Leben" ist vielleicht etwas überanalysiert.
Ich meine nur, kann da beim Sterben nicht mal ... wie soll ich sagen ... eine Grenze eingehalten werden oder ein Anstand oder gewisses Maß an Vernunft? Ja klar, das ist vermenschlichte Sichtweise ... 46.115.91.197 19:27, 5. Jan. 2014 (CET)
Da steht "pneumonia, Lungenentzündung; multiple drug intoxication, Medi-Vergiftung; elevated level, erhöhte Menge; Eisenmangel anaemie" und Du machst daraus "einfach ohne Ankündigung tot umgefallen mit Erkältung und Erkältungs-Medis". Das ist keine Sichtweise. Das ist grob fahrlässig - wenn nicht absichtlich - dramatisierend falsch übersetzt. Zumindest selektiv dramatisierend wahrgenommen. --217.84.100.251 16:58, 6. Jan. 2014 (CET) Du wolltest hier nur sagen, daß Du Brittany magst .. ährrh .. mochtest (lol) und es todschade fandest .. ährrh .. findest, daß sie so jung gestorben ist. (Für so Plaudereien gibt`s übrigens das Café. Sei`s drum.) --217.84.100.251 17:03, 6. Jan. 2014 (CET)
Einer hatte ein Rezept dagegen: der Brandner Kaspar. Gruß -- Dr.cueppers - Disk. 18:21, 6. Jan. 2014 (CET)

Artikel Astrologie

Vorweg: mir persönlich ist egal, ob Astrologie in irgendeiner Weise Belang beanspruchen darf. Vor einigen Jahren befaßte ich mich aber intensiv mit einer Form retrospektiver Prognostik auf der Basis der Überlegungen von Alexander Marr (1919-2000) und einer Forschergruppe um ihn herum. Diese äußerst differenzierte und komplizierte - und von wenigen Astrologen beherrschte - Variante von Astrologie taucht in Ihrem Artikel nicht auf. Daß diese auch für die Überprüfer schwierig zu erlernende Methode mit ihren genau ineinandergreifenden Variablen jemals unabhängig überprüft worden wäre, ist mir nicht bekannt. Einen Kurzüberblick finden Sie auf meiner Website www.autoren-theater.de unter der Rubrik Essayistik. --77.11.167.60 23:31, 4. Jan. 2014 (CET)

1. ist das sowas: [11]? 2. man kann den Artikel ja einfach erweitern, wenn es eine wichige Form der Astrologie ist... --Heimschützenzentrum (?) 23:49, 4. Jan. 2014 (CET)
Nicht jede esoterische Spielerei ist enzyklopädisch relevant. von daher ist es IMHO ganz richtig, dass diese Methode nicht in der Wikipedia nachzulesen ist. Verbreiteter esoterischer Quatsch hat natürlich seinen Platz in der Wikipedia. Deswegen gibt es Artikel über Homöopathie, Astrologie, Kartenlegen, Erdstrahlen, Heilsteine und mancherlei anderen esoterischen Quatsch, aber nicht über jeden. --Rôtkæppchen68 23:58, 4. Jan. 2014 (CET)
Man könnte es auch so aufziehen: Wenn das mit den Büchern stimmt, wäre er relevant. Dann könnte man den Krempel erst einmal im Personenartikel darlegen. GEEZER… nil nisi bene 00:06, 5. Jan. 2014 (CET)
Hmmm ... Das ZVAB hat ein 68-Seiten Heftchen und ein Tabellenwerk, die StaBi Berlin nichts. Hätte ich das voraussehen müssen? GEEZER… nil nisi bene 00:09, 5. Jan. 2014 (CET)
OK, GoogleBooks hat etwas mehr, also wohl relevant. In einem davon scheint seine Vita zu stehen... GEEZER… nil nisi bene 00:13, 5. Jan. 2014 (CET)
Etwas so kompliziert zu machen, dass es nicht sinnvoll überprüft werden kann, macht etwas nicht besser, relevanter, oder gar richtiger. Ansonsten siehe Rotkaeppchen68. Wenn genügend Leute dem nachlaufen, kann es erwähnt werden. --mfb (Diskussion) 03:45, 5. Jan. 2014 (CET)
Marr arbeitete mit Primärdirektionen, einer Prognosemethode, die in der Astrologie vor allem vom 15. bis 17. Jahrhundert gängig war und gute Kenntnisse in sphärischer Trigonometrie verlangte. Nach dem Niedergang der Astrologie im 17. Jahrhundert gab es nur noch vereinzelte Spezialisten in Kreisen pensionierter Marineoffiziere, die ihr nautisches Handwerk beherrschten, oder bei den Kalendermachern. Die modernen Astrologen nutzten meist grob vereinfachte Methoden, symbolische Schlüssel und dergleichen. Das änderte sich mit der Entwicklung von Computern. Marr scheint wohl der erste Pionier der astrologischen EDV zu sein, Jahre vor Michael Erlewine[12] oder Alois Treindl[13]. Dennoch arbeiten heute nur wenige Astrologen mit Primärdirektionen, im deutschsprachigen Raum dürften es höchstens einige Dutzend sein, die sich wirklich damit auskennen. Mir ist nicht bekannt, dass durch die Verwendung von Primärdirektionen besonders signifikante prognostische Aussagen möglich werden. - Eine mögliche Überprüfung der Marrschen Methoden wird auch dadurch erschwert, dass er ein ganzes Bündel zusätzlicher Tools einsetzte, insbesondere ein von ihm postuliertes Empfängnishoroskop. Als eigenes Lemma für Wikipedia hat er wohl keine Bedeutung, man könnte ihn im Artikel Astrologie in einem noch zu schreibenden Absatz zu Astrologie und Computer nennen. --Stobaios?! 04:27, 5. Jan. 2014 (CET)
Wenn er mehr als 2 (oder 3 ?) Bücher in Verlagen geschrieben hat, ist er relevant. von Däniken ist auch relevant, und auch die Katzenberger oder jeder, der seinen Schniedel 5 x auf DVD verewigt hat - und dafür evtl. einen Preis bekommen hat. Du und ich sind (noch) nicht relevant ... :-) GEEZER… nil nisi bene 11:54, 5. Jan. 2014 (CET)
*quetsch* Schreibst du den Artikel und stellst die Relevanz dar? :) --Stobaios?! 18:36, 5. Jan. 2014 (CET)
Beim Relevanzcheck] nachfragen! GEEZER… nil nisi bene 14:41, 5. Jan. 2014 (CET)
retrospektive Prognostik ist bei jeder Chartanalyse Usus. --Mr. Froude (Diskussion) 04:30, 5. Jan. 2014 (CET)

Was war hier eigentlich die "Allgemeine Wissensfrage"?--77.119.133.31 09:48, 5. Jan. 2014 (CET)

zwischen den Zeilen: 1. Was ist „retrospektive Prognostik”? 2. Warum steht davon nix im Artikel Astrologie? :) --Heimschützenzentrum (?) 10:10, 5. Jan. 2014 (CET)
Retrospektive Prognostik ist sicher sowas wie Zurück in die Zukunft. --91.0.149.27 10:34, 5. Jan. 2014 (CET)
... oder die Meinung und Einschätzung eines Proktologen... GEEZER… nil nisi bene 14:43, 5. Jan. 2014 (CET)
S. a. A posteriori. Dumbox (Diskussion) 14:51, 5. Jan. 2014 (CET)
@Heimschützenzentrum: Retrospektive Prognostik ist eine Methode zur Überprüfung der Validität von Prognosemethoden - man testet eine Methode im nachhinein, ob sie bei bekannten Ereignissen korrekte Ergebnisse liefert. Da allerdings in der Astrologie der Interpretationsspielraum ziemlich groß ist, sind bislang keinerlei wissenschaftlich verwertbaren Ergebnisse bekannt. --Stobaios?! 18:17, 5. Jan. 2014 (CET)
verstehe... --Heimschützenzentrum (?) 13:15, 7. Jan. 2014 (CET)

Gibt es den italienischen Schriftsteller Giorgio Pasetti?

Im Netz kursiert vielfältig das Bonmot „Die gesündeste Turnübung ist das rechtzeitige Aufstehen vom Eßtisch.“ Als Urheber wird durchgehend ein gewisser Giorgio Pasetti angegeben. Er soll italienischer Schriftsteller sein, es handelt sich also offenkundig nicht um den Schauspieler Giorgio Pasotti. Im Zusammenhang mit dem Aphorismus lese ich auch keine anderen Namen für den Urheber. Nun habe ich längere Zeit die Suchmaschinen, auch GBS und Ngram befragt, die it.wp besucht und ebenso ergebnislos in den Datenbanken der DNB, der LoC , des KVK und auch in eurobuch.com gesucht. Mir scheint, daß der Mann als Phantom ein schönes Beispiel für die Eigendynamik von verantwortungsloser Falschinformation im Web durch blindes Kopieren abgeben könnte (gibt es für dieses Phänomen eigentlich schon einen in den Wissenschaften benutzten Begriff?). Ich finde jedenfalls kein einziges Buch, das von einem Schriftsteller Giorgio Pasetti stammt. Hat jemand noch eine Idee? Habe ich etwas übersehen? --84.191.134.175 10:40, 3. Jan. 2014 (CET)

Über Zitatsammlungen im Netz könnte ich nicht aufhören zu jammern (und tue es auch nicht, ehrlich gesagt). Ja, die sind alle abgekupfert, frei von Quellenangaben, und man könnte, wenn man nichts Besseres zu tun hätte, sein Leben mit Nachrecherchieren verbringen. Die italienische WP kennt als Autor einzig einen Giovanni Pasetti; der taucht aber nicht im Zusammenhang mit diesem Bonmot auf. Ich bin mir sicher, es ist in Wahrheit von Sokrates. Grüße Dumbox (Diskussion) 10:58, 3. Jan. 2014 (CET)
Danke für deinen Hinweis. Wie geschrieben hatte ich die it.wp schon befragt. Andere Pasettis gibt es ansonsten noch eine Menge, aber darum geht es mir ja nicht. Auffällig ist, daß der Name Giorgio Pasetti nicht in Varianten auftaucht. Auch das schürt den Verdacht des copy&paste. Es geht mir aber zunächst um den angeblichen Schriftsteller. Unter der Annahme, daß Schriftsteller normalerweise ganz gut im Netz zu finden sind. Und meine Frage war, ob ich da eine passable Methode der Suche nach Schriftstellern übersehe. Mehr nicht. Im nächsten Schritt würde ich wohl - und das ist an dieser Stelle nicht meine Frage - nach den frühesten Auftreten des Bonmots suchen, das nach meinem Gefühl eher deutsche Fälscher-Wurzeln haben könnte. Manchmal kann man, wie hier sogar die Fälscher und Täter namentlich dingfest machen. Ich finde noch englische Varianten wie „The healthiest form of gymnastics is to leave the table at the right time.“ (und auf norwegisch (?): „Den sunneste gymnastikk er å reise seg fra bordet i rett tid.“ [14] oder: „The healthiest gymnastic exercise is getting up on time from the dining table.“ [15] (auch ein möglicherweise deutscher Name, der diese Fassung verantwortet.) --84.191.134.175 11:23, 3. Jan. 2014 (CET)
Die jüngste Verwendung kam mir gestern im Deutschlandfunk unter -- Dir womöglich auch? --, aber das wird Dir nichts nützen. Ich bin mir aber sicher, daß ich den Spruch schon in den Akten des Konzils von Ephesus gefunden habe; dort wird er aber Goethe zugeschrieben, der das schon 1356 an Martin Luther geschrieben haben soll. Noch früher hatte es nur der Postillon. --Hermine Tuzzi (Diskussion) 12:07, 3. Jan. 2014 (CET)
Hmmmm ... Bis zum Ende des 19. Jahrhunderts (und darüber hinaus) galt "Wampe macht Ohsan!" (Eine kräftige Statur beeindruckt die Mitmenschen).
Neben der Existenz des Giorgio Pasetti - ab wann findet man den Satz im deutschsprachigen Web?
Im Englischen finde ich ihn 2004-03-19 und hier Hier 2004-04-02. GEEZER… nil nisi bene 12:38, 3. Jan. 2014 (CET)
Auf deutsch: 21.I.2002; mal sehen, ob ich noch früher komme. --Hermine Tuzzi (Diskussion) 12:53, 3. Jan. 2014 (CET) Hmm, es gibt noch ein paar Ergebnisse von 2001, die sind aber nicht sicher datiert. --Hermine Tuzzi (Diskussion) 12:55, 3. Jan. 2014 (CET)

1974: http://books.google.de/books?id=2CEeAQAAMAAJ&q=rechtzeitige. --Vsop (Diskussion) 13:05, 3. Jan. 2014 (CET)

Hmpf, und ich war gerade so stolz auf meinen 1998er... --Hermine Tuzzi (Diskussion) 13:11, 3. Jan. 2014 (CET)
GIORGIO PASETTI (1916–2006), ITALIENISCHER SCHRIFTSTELLER - Ernährung, Gesundkrempel und so. GEEZER… nil nisi bene 13:16, 3. Jan. 2014 (CET)
Hmm, in italienischen Bibliothekskatalogen finde ich ihn nicht, und er hat auch keinen VIAF-Eintrag... Wo hast Du ihn gefunden? Ich nur hier. --Hermine Tuzzi (Diskussion) 13:23, 3. Jan. 2014 (CET)
Auch die StaBi Berlin, das italienische Amazon und das ZVAB kennen ihn nicht. Es könnte einer dieser unglücklichen Schriftsteller sein, die im ganzen Leben nur einen einzigen erhaltenswerten Satz gesagt haben. Welch ein dramatisches Schicksal... GEEZER… nil nisi bene 13:33, 3. Jan. 2014 (CET)
Wenn er Schriftsteller war, muß er ja irgendwas veröffentlicht haben -- sonst wäre er ein wahrhaft unglücklicher Schriftsteller. --Hermine Tuzzi (Diskussion) 13:37, 3. Jan. 2014 (CET)
Ruck-Zuck ... neue Spur! :-) [TF] Google => "Peter Pasetti" esstisch <= oder => "pasetti" "beste Turnübung" -giorgio <= (da zitiert sogar die Kirche) Der Mann war humorvoll (* 1916 ... würde oben teilweise passen...)), vielleicht hat er mal diesen Aphorismus aus dem Gehege seiner Zähne entlassen ? GEEZER… nil nisi bene 13:40, 3. Jan. 2014 (CET)
Das würde mich nur überzeugen, wenn ein Nachweis vor 1974 erbracht werden könnte (auch wenn das angeblich gemeinsame Geburtsjahr 1916 (unbelegt bei Giorgio) und 1916 (belegt bei Peter) zur Spekulation einlädt). Handfest ist bisher das Ergebnis von Vsop: Ronner und Ott in Thun sind seriöse Adressen (wobei die Ausgabe in der Indiana University nach der ISBN witzigerweise eine Ausgabe des VEB Verlag Sprache und Literatur ist, die an den gestrengen Immigration Services vorbeischleichen konnte :-). Die Person Ronner spricht eher gehen eine Fälschung, aber man weiß ja nie. Ansonsten scheinen mir die paar Treffer zu Peter eher aus der Ecke zu kommen, in der man ein unbelegtes und gefälschtes Zitat lustvoll und hemmungslos nochmals unbelegt fäscht. Hermine bringt es auf den Punkt: Was hat der Giorgio denn geschrieben? Er wird mal als Schriftsteller, mal als Dichter ausgegeben. Wo steht denn nun das eine Buch, das ihn dazu macht? Und en passant: Seit das ZVAB von amazon gekauft wurde wird es übrigens von eurobuch.com mitdurchsucht, man muß also nicht mehr unter beiden Adressen wühlen. StabBi ist auch im Berliner Gesamtkatalog. Und Goethe ist immer eine sichere Bank, aber mir doch zu sehr erforscht. Die ideale Person, der man irgendeinen Aphorismus anhängen kann ist m. E. Peter Ustinov. Das glaubt jeder und nichts ist belegt... --84.191.134.175 14:41, 3. Jan. 2014 (CET) Streichung, s.u. --84.191.134.175 15:07, 3. Jan. 2014 (CET)
*lach* Ja, stimmt: Hätte man dem das Zitat zugeschrieben, wäre es niemals aufgefallen. Peter Pasetti halte ich auch für eine falsche Spur: Der ist halbwegs bekannt, und wenn man ein Zitat von eitel Pasetti vorfindet, verfällt man leicht auf den Gedanken, es dem ???-Erzähler zuzuschreiben, einfach weil er der einzige bekannte Pasetti ist. Für ganz unwahrscheinlich halte ich aber auch nicht, daß der angebliche Aphorist so zu seinem Giorgio gekommen ist, einfach weil's so schön italienisch klingt. Übrigens enthält das Buch von Ronner auch Zitate von Peter Pasetti. Vielleicht handelt es sich ja auch um eine Verwechslung von Ronners Seite. Sind die Zitate bei ihm im allg. belegt? Dann würde ich bei Gelegenheit meiner Anwesenheit dort, wo man ihn lesen kann, einfach mal einen Blick reinwerfen. --Hermine Tuzzi (Diskussion) 14:56, 3. Jan. 2014 (CET)
In dem Buch kommt - neben mehrfach P.P. auch ein "Giorgio Prati" vor. Man müsste wissen, was der Drucksetzer am Vorabend gemacht hat. Wie oben "im Scherz" (aber nie ohne Absicht!) erwähnt: Gibt es neben "keinem Buch" von ihm auch überhaupt ein anderes Zitat von "Giorgio Pasetti"? Wie wahrscheinlich ist es, dass von einem Autor von vor 40 Jahren nur ein einziger Satz übrig bleibt? :-) GEEZER… nil nisi bene 16:59, 3. Jan. 2014 (CET)
Google => "Giorgio Pasetti" -"rechtzeitige Aufstehen" <= zeigt
  • (a) Dass auch die Dänen him diesen Satz zuschreiben und
  • (b) dass ihm auch der Carl-Zuckmayr-Satz (?) "Die eine Hälfte des Lebens ist Glück, die andere Disziplin. (Giorgio Pasetti)" 1 x zugeschrieben wird.
Es bleibt somit bei "Der Schriftsteller ohne Buch" und "Der Schriftsteller, von dem nur ein Satz bekannt ist, der aber in mindestens 2 Sprachen...". GEEZER… nil nisi bene 17:18, 3. Jan. 2014 (CET)
Vier Sprachen: Englisch, Deutsch, Norwegisch, Schwedisch. Mit einem signifikanten Übergewicht in Deutsch, das sicher nicht an unserer Büchmann- und Kaiserreden-Kultur liegt. (Apropos und OT: Ist eigentlich der Poesiealbum-Kult der kleinen Mädchen mittlerweile untergegangen oder Facebook gewichen?) Daß wir den Satz nicht in Italienisch finden mag uns zu denken geben, vielleicht aber auch an seinem Inhalt liegen. In dieser Kultur liegen Eßtisch und Turnübung gedanklich wohl zu weit auseinander und der soziale Akt des gemeinsamen Essens steht der disziplinierten Selbstbeschränkung entgegen. Und die Franzosen? Die Kultur, die uns einerseits die Raffinesse und andererseits mit Gargantua einen der größten Fresser und Säufer der Literaturgeschichte geschenkt hat? Ich würde meinen Hut verwetten, daß dieses sachlich durchaus richtige, aber im Grund des Herzens auch urpreußische, protestantische Vater-Jahn-Bonmot deutsche Wurzeln hat. --84.191.149.187 21:55, 4. Jan. 2014 (CET)
Zu Ist eigentlich der Poesiealbum-Kult der kleinen Mädchen mittlerweile untergegangen oder Facebook gewichen?): Nein, es gibt sie wieder vermehrt im Stile dieser modischen Monster High-Figuren. --Tommes  04:59, 8. Jan. 2014 (CET)
Korrektur, Die StaBi ist doch nicht im BGK enthalten, der stand nur (oder steht noch?) in der StaBi. Sorry. --84.191.134.175 15:03, 3. Jan. 2014 (CET)
Danke für die Hilfen. Gibt es eigentlich einen Ort, wo fälschlich zugeschriebene oder nicht nachzuweisende Zitate gelistet sind? Wir haben ja solche Fälle öfters hier. Ist eigentlich schade um die Arbeit, die im Archiv untergeht. Bei Wikiquote gibt es zwar hin und wieder einen entsprechenden Abschnitt, aber in diesem Fall hätte man ja eine mutmaßlich fiktive Person, von der es mutmaßlich kein Zitat sondern nur eine Zuschreibung gibt... --84.191.138.77 00:56, 7. Jan. 2014 (CET)
Da greift das Kosten-Nutzen-Prinzip: Jedes falsch zugeordnete Zitat - interessant wäre, welches Zitat/welcher Allgemeinplatz der grössten Anzahl an Personen untergeschoben wurde... - kann nicht dokumentiert werden, weil dann am nächsten Dienstag Karl-Heinz kommt und eine weitere Falschmeldung auf seine Website schreibt. ABER: Die Werke berühmter Menschen - ich habe es für Einstein und Darwin gesehen - werden analysiert und da werden nicht nur die Hammerzitate, sondern auch die fälschlicherweise zugeordneten gelistet und erklärt (woher sie kommen). GEEZER… nil nisi bene 17:24, 7. Jan. 2014 (CET)

Augenarztuntersuchung

Welche Untersuchungen sollten Bestandteil eines Routinebesuchs bei einem Augenarzt sein? --Balham Bongos (Diskussion) 19:43, 4. Jan. 2014 (CET)

1. durch die Helferin: Autorefraktometer... 2. durch den Facharzt: (a) Fein-Tuning der Werte durch den Facharzt, (b) Netzhaut begucken (wohl auch mit Augentropfen, die die Iris erweitern)... --Heimschützenzentrum (?) 20:30, 4. Jan. 2014 (CET)
Das hängt davon ab, ob es ein Erstbesuch ist oder ein Wiederholungsbesuch, ob ein Kind untersucht wird oder ein Erwachsener: 1. eine allfällige Fehlsichtigkeit überprüfen (Sehtest); 2. Hirschberg-Test, 3. Messung des Augeninnendrucks (Tonometrie); 4.Netzhautkontrolle. 5. Sonstiges siehe auch Kategorie:Diagnostisches Verfahren in der Augenheilkunde--Ohrnwuzler (Diskussion) 01:12, 5. Jan. 2014 (CET)
Anders gedacht und gefragt: Wohl alles das um irreversible und teure Erkrankungen des Auges zu vermeiden. Beim Lesen des Artikels kommt man auf Wirbeltierauge und findend dort die Kategorie:Krankheitsbild in der Augenheilkunde. --Hans Haase (有问题吗) 10:13, 5. Jan. 2014 (CET) Bitte vor Bedarf lesen
Es ist ein Wiederholungsbesuch und es geht um einen Erwachsenen. Man soll ja einmal im Jahr zum Augenarzt gehen, heißt es. Ich werde dann um "Feintuning der Werte", "Netzhaut begucken mit Augentropfen" und "Messung des Augeninnendrucks" bitten, das restliche vorschlagen. Gibt es weitere sinnvolle Standarduntersuchungen? --89.0.228.187 17:53, 7. Jan. 2014 (CET)
Bei "Netzhautuntersuchung mit Augentropfen" nicht mit dem Auto anreisen oder genügend Zeit danach einplanen, bis man wieder fahrtauglich ist. --TheRunnerUp 17:58, 7. Jan. 2014 (CET)

Modems in Österreich und Schweiz

Da es mich persönlich interessiert: Gibt es hier noch jemand aus der Vor-Internet-Ära, die vor 1980 mit Modems gespielt haben? In Deutschland gab es die Datentaste zum Anschalten ans Telefonnetz, wie war es in den anderen Ländern? Gruss --Nightflyer (Diskussion) 21:00, 5. Jan. 2014 (CET)

In der Schweiz gab es da keine Tasten, sondern Wählscheiben, und Modems waren eh verboten :) --85.0.126.226 14:49, 6. Jan. 2014 (CET)
Akustikkoppler gab es auch, ein Modem das nicht wählen kann hat keinen Vorteil.--79.234.114.238 21:16, 6. Jan. 2014 (CET)
Sind wohl alle hier zu jung hier, oder haben nicht in wichtigen Industrien oder Abteilungen gearbeitet. Eine Bank in der Schweiz ohne zentrale Datenverarbeitung ist auch in diesen Jahren nicht vorstellbar, zumal in Deutschland schon 1969 Apotheken durch Parallelmodems vernetzt waren. Vielleicht hat ja der Postillon nicht Unrecht :-) Gruss --Nightflyer (Diskussion) 23:51, 6. Jan. 2014 (CET)
Du kannst ja einmal bei der Kapsch AG nachfragen, die haben früher solches Equipment für die Post gebaut. Übrigens Bergstämme: die LKW-Maut mittels GO-Box von Kapsch funktioniert schon länger und besser, als das Toll Collect-System bei den Flachlandgermanen. Nur so als Beispiel. --El bes (Diskussion) 15:16, 7. Jan. 2014 (CET)
In den 1980er-Jahren bot Interdiscount Modems an, mit der Warnung, dass deren Betrieb in der Schweiz verboten wäre. Schreibtelefone dagegen durften in der Schweiz schon in den 1970er-Jahren verwendet werden (mit 110 baud), damals aber alle mit Akustikkoppler. Etwas OT: Tritel ist entsetzlicherweise noch rot! Den beigen Tritel Elm dürfte ja jeder Schweizer kennen. --Filzstift  15:43, 7. Jan. 2014 (CET)
„Nur für den Export“-Modems gab es auch in der Bundesrepublik Deutschland. Meine ersten zwei Modems besaßen keine FTZ- bzw BZT-Zulassung und ich hab sie trotzdem ohne Probleme betreiben können. Allerdings hab ich mir die Anschlusskabel selbst zusammenlöten müssen. --Rôtkæppchen68 21:27, 7. Jan. 2014 (CET)

Reform der Musik?

Gibt es eigentlich eine Sammelbezeichnung für künstlerische (oder sonstige) Bewegungen, die die Musik ihrer Meinung nach positiv verändern, moderner machen wollen? Z. B. durch die Einführung von sowas wie der Zwölftonmusik? Die Bezeichnung "neue Musik" scheint mir irgendwie ein bisschen wage. Schließlich wird es solche Ansätze schon in der Vergangenheit gegeben haben, oder? --188.100.178.39 11:03, 5. Jan. 2014 (CET)

Ja, Die Kunst der Fuge --Hans Haase (有问题吗) 11:25, 5. Jan. 2014 (CET)
Guckst du hier : Avantgarde#Avantgarde_in_der_Musik --Anachron (Diskussion) 11:56, 5. Jan. 2014 (CET)
Avantgarde, Reformer, Neuerer, Revolutionäre: Nein, einen eigenen Begriff gibt es wohl nicht. Für einen kurzen Überblick zu diesem Thema seit Bach empfehle ich Paul Draytons "Masterpiece", vorgetragen von den King's Singers. Grüße Dumbox (Diskussion) 12:19, 5. Jan. 2014 (CET)
Vor Bach: Ars nova (14. Jh., siehe auch Ars antiqua) und Claudio Monteverdi (17. Jh.) sowie Monodie, Seconda pratica. --Motmel ♫♫♪ 12:37, 5. Jan. 2014 (CET)
experimentelle Musik? Neue_Musik? (Neue_Improvisationsmusik (John Cage free Jazz), Intuitive_Musik (Stockhausen ff)) .. Neue Musik scheint es zu treffen (Sammelbegriff), vielleicht noch allgemeiner, als Avantgarde (dissonant, elektronisch, unkonventionell) (?) .. --217.84.100.251 17:23, 6. Jan. 2014 (CET)
Neue Musik trifft es nicht, es ist vielmehr zur Epochenbezeichnung der Musik von etwa 1950 bis 1990(?) erstarrt. --FA2010 (Diskussion) 12:27, 7. Jan. 2014 (CET)
ah ok. Dann gibts vllt kein Begriff und schlage vor es "Reformer" oder "Reformbewegung" oder "positiv-moderne Musikreformbewegung" neu zu nennen. ;o]) Oder bei "Avantgarde" bleiben? O.-o --217.84.101.47 12:57, 9. Jan. 2014 (CET)

Möge der richtige Oudemans aufstehen!

Jean Abraham Chrétien Oudemans (1884)
Oudemans und andere auf der Insel Réunion (1874)

Folgende - besonders das rechte - Oudemans-Foto ist verfügbar. Ich glaube, ihn ausgemacht zu haben, hätte aber gerne noch ein paar Meinungen. Danke!

Bonusrunde: Kann jemand (eine) weitere Person(en) namentlich identifizieren? Es geht um die Expedition bezüglich des Venustransits 1874). GEEZER… nil nisi bene 15:00, 5. Jan. 2014 (CET)
Ich würde meinen, die weissgekleidete Person links - beim Porträtphoto war er Mittfünfziger, beim Gruppenfoto zehn Jahre jünger. Hier suchen noch andere nach den Identitäten, vielleicht kann man die fragen, ob sie bereits Erfolg hatten. Gruss --Port(u*o)s 15:12, 5. Jan. 2014 (CET)
Eine Weitere Website sucht danacht und sie erwähnen folgende Personen:
Nous souhaiterions échanger des documents avec l'Université d'Utrecht qui nous a fait parvenir les deux photos ci-dessous. On sait par ailleurs que les Hollandais ont rencontré le gouverneur M. de Lormel, le maire de Saint-Denis M. Le Siner, le directeur de la B.R. M.Bridet, l'horloger M. Chailliey, le médecin M. Muircec. Pour l'hébergement, ils se seraient adressés à MM. Bertho, Hugot et De Touris, Pezzani, T.F. Blanken GEEZER… nil nisi bene 15:42, 5. Jan. 2014 (CET)

Doch, Oudemans ist ganz klar der, der so souverän danebensteht. Gleiche Nase, gleiche Lippen, zieht die Augenbrauen gleich hoch, sogar Barttracht und Frisur sind gleich. Der Gouverneur fr:Louis Hippolyte de Lormel ist auf dem Bild meines Erachtens nach nicht abgebildet. Der in der Mitte, der seiner Frau beinahe an den Busen grapscht, ist bestimmt ein örtlicher Würdenträger, links daneben (rechts im Bild) dito (mit seiner Gattin vornedran, die noch einen Kater vom Vortag hat). Im Hintergrund der Hilfswilly der Expedition, rätselhaft ist der Bärtige. Schliesslich vorne das Ehepaar links, beide sitzend, da würde ich auf den Arzt tippen. Gruss vom Hellseher --Port(u*o)s 16:00, 5. Jan. 2014 (CET)

Ta-DAH! Rätselhaft war der Bärtige. Wie kann ich das Foto hier - unten halblinks - downloaden? Da sind die Vögel mit Namen drauf. GEEZER… nil nisi bene 17:39, 5. Jan. 2014 (CET)
Cursor aufs Bild halten, dann auf die Vergrößerung gehen (Javascript nötig), Rechtsklick: Bild-URL kopieren, in neuem Tab öffnen, speichern. Grüße Dumbox (Diskussion) 17:49, 5. Jan. 2014 (CET)
Ich hab Screenshot versucht und kann in etwa lesen. Kriegst du das besser hin und kannst du das Bild hier hochladen? GEEZER… nil nisi bene 17:52, 5. Jan. 2014 (CET)
Habe ich schon lange nicht mehr gemacht. Ich hab dir eine URL zu meiner Dropbox gemailt. Grüße Dumbox (Diskussion) 17:56, 5. Jan. 2014 (CET)
Prima! Zwei sind identifiziert (und selbst die Niederlender erkennen ihren O. nicht im WP-Bild). So, heute genug Niederländisches gehabt.  :-) GEEZER… nil nisi bene 18:38, 5. Jan. 2014 (CET)

URL zum Bild - keine tolle Qualität, aber erkennbar. --mfb (Diskussion) 20:03, 5. Jan. 2014 (CET)

Also sind im Hintergrund die beiden
  • F. F. van de Sande Bakhuyzen (direkt hinter Oudemans, ohne Bart)
Einen F.F. habe ich nicht gefunden, aber "Ernest-Friedrich van de Sande Bakhuyzen" passt - sowohl visuell als auch professionell (im Artikel berits eingetragen...) GEEZER… nil nisi bene 23:46, 5. Jan. 2014 (CET)
Google => "H. T. Soeters" dutch_expeditions.ppt <= liefert hoch aufgelöstes "2. Bild". GEEZER… nil nisi bene 23:52, 5. Jan. 2014 (CET)
(BK) Für mich ist das zweifellos Mynheer Soeters: Kopfform, Schnurrbart, Augenbrauen und Briskfrisur passen. --Hermine Tuzzi (Diskussion) 23:56, 5. Jan. 2014 (CET)
Sind mehr der Meinung? Man könnte es mit "?" (schon im Ausgangsbild !) in den Artikel setzen... GEEZER… nil nisi bene 00:02, 6. Jan. 2014 (CET)
Cave: ich bezog mich auf das kleine Bild; die Powerpointpräsentation kann ich nicht öffnen. --Hermine Tuzzi (Diskussion) 00:06, 6. Jan. 2014 (CET)
Könnte der Herr auf dem Stuhl nicht Dr. Auguste Vinson sein? --Laurentianus (Diskussion) 00:27, 6. Jan. 2014 (CET)
Und der Herr rechts der Direktor des Observatoriums Bridet? --Laurentianus (Diskussion) 01:16, 6. Jan. 2014 (CET)
Die Franzosen haben jedenfalls ihr Wissen hier her. --Laurentianus (Diskussion) 01:33, 6. Jan. 2014 (CET)
Ja, der Herr rechts ist Hilaire Gabriel Bridet, das glaube ich auch. Vinson würde ich nahezu ausschliessen, Soeters glaub ich immer noch nicht so recht - in der vergrösserten Version sogar eher weniger. Port(u*o)s 04:26, 6. Jan. 2014 (CET)
Soeters war ja auch krank, litt schon seit der Überfahrt (Aden) an der Leberzirrhose, an der er dann 1879 starb … Port(u*o)s 10:50, 6. Jan. 2014 (CET)
Ich habe nachgetragen.
Nun Logikfrage: Wenn man so ein Foto aufnimmt und die Gäste aus fremden (Nieder)Landen und auch der Observatoriumsdirektor drapiert sind ... WEN stellt man da in die Mitte? Es müsste das lokale Alphatier sein (mit Körperkontakt zum seinem Weibchen). Gouverneur? Bürgermeister? Evtl. ein niederländischer Botschafter. Woher kennen wir die Eierschale, die er auf dem Kopf trägt? Das ist kein einfacher Tropenhelm, oder? GEEZER… nil nisi bene 09:22, 6. Jan. 2014 (CET)
Ja, ich denke auch, daß es vielleicht doch de Lormel sein könnte, eben forscher und glatter als auf der Abbildung, die es von ihm gibt; oder doch Le Siner. Die drei Damen würde ich tendenziell drei Einheimischen zuordnen wollen, eben Bridet, dem Herrn in der Mitte, und dann bliebe eigentlich nur nur der Herr links auf dem Stuhl als Einheimischer übrig. Das ist aber eben spekulativ.--Laurentianus (Diskussion) 09:30, 6. Jan. 2014 (CET)
Hmmmm Louis Hippolyte de Lormel (* 1806) der müsste dann nahezu 68 sein. Mit so einer jungen Dame ...? GEEZER… nil nisi bene 09:33, 6. Jan. 2014 (CET)
Ne, de Lormel würde ich ebenfalls ausschließen. Ist der Bürgermeister fr:Louis Le Siner als Alphatier denn wirklich schon vom Tisch? Im Netz finde ich sonst keine Bilder von ihm. Die "Eierschale" ist schon ein normaler Tropenhelm, aber eher altmodisch. Die mit der breiten Krempe haben sich wohl erst im 20. Jh. durchgesetzt, und ich denke der Herr hat über seinen Helm noch eine Schutzhülle (gegen Staub, Möwenscheiße, etc.) drübergezogen. Geoz (Diskussion) 10:49, 6. Jan. 2014 (CET)
Ich habe auch nur [das] gefunden. Finde ich nicht verkehrt. --Laurentianus (Diskussion) 11:00, 6. Jan. 2014 (CET)
Ich habe auf der obigen Réunion-Site Paraden von Vögeln mit exakt solchen Helmen (warum muss ich dabei immer an Mel Brooks denken ??) gefunden. ich habe das Bild weiter an den Nachbarn hier in F geschickt, der Militaria sammelt. Der weiss sowas. Geduld... :-) GEEZER… nil nisi bene 12:22, 6. Jan. 2014 (CET)
So. Er sagt "casque colonial". Das kommt im Prinzip hin - aber die sind alle mehr oder weniger "abgesetzt" (Hut + Krempe). Der ähnlichste ist noch der => Casque colonial modèle 1886 <=. Aber der hier im Bild ist nicht abgesetzt und irgendwie breiter.
Zu den Alpha-Tieren sagt er, es könnte auch ein "Colon" sein, das war eine Art ökonomischer Entwickler (Managertyp) der Kolonie... GEEZER… nil nisi bene 13:34, 8. Jan. 2014 (CET)
Australische Pferdehändler in Indien, 1929
Das Ähnlichste, was ich bei Commons an Tropenhelmen gefunden habe, hier rechts (da ist, glaube ich, auch zu erkennen, dass die Schutzhüllen haben). Aber Du hast Recht, derartig calimereske Helme sind nicht normal... Sammeln wir die Fakten: 1) Zivilist, aber herrschaftsgewohnt (grimmig runtergezogene Augenbrauen, verächtlich hochgezogene linke Oberlippe), 2) hat einen ausgefallenen Modegeschmack, 3) kann den aber generell durchsetzen (Ich kann die von Dir erwähnten "Paraden" auf Réunion nicht finden. Meinst Du richtige Militärparaden?), 4) Wenn der Schnauzbart auf dem Stuhl ebenfalls ein Ausländer ist, zu wem gehört dann die dritte Frau, die viel näher beim Obermotz sitzt, als bei ihm? Wenn er ein Einheimischer ist, warum sitzt er dann auf einem Stuhl, wie die anderen Frauen? Abgründe tun sich hier auf... Geoz (Diskussion) 18:17, 8. Jan. 2014 (CET)

Zusatzfrage

In dem Bild oben ist ja der bärtige F-Graf identifiziert. Seit wann kennt man Selbstauslöser bei Kameras? Unser Artikel ... schweigt. GEEZER… nil nisi bene 00:02, 6. Jan. 2014 (CET)

Er sieht auch aus wie nachträglich einkopiert.--Laurentianus (Diskussion) 00:27, 6. Jan. 2014 (CET)
Datei:Bulb (photography).jpg
Fernauslöser gibt es ja schon fast solange es photographische Verschlüsse gibt. Bei einem pneumatischen Verschluss mit Gummiball und Gummischlauch musste man nur den Schlauch entsprechend verlängern. --Rôtkæppchen68 00:31, 6. Jan. 2014 (CET)
...was wegen der Kompressibilität der Luftsäule im Gummischlauch nicht unbegrenzt möglich ist. --79.216.212.64 16:39, 8. Jan. 2014 (CET)

Fehler bei Facebook korrigieren - wie?

Bei Facebook wird Ittigen inkorrekterweise als Duplikat oder Teil von Worblaufen geführt. Wie Ittigen bzw. Worblaufen zu entnehmen ist, trifft das Gegenteil zu. In der FB-Seite zu Worblaufen wird der WP-Artikel zu Ittigen angezeigt. Wer kann helfen, dieses Chaos zu korrigieren? FB ist da ein Schloss mit sieben Siegel für mich. :( P.S.: Zum Chaos gehört übrigens auch die redundante sowie falsch geschriebene Seite Ittigen Bei Bern. --178.83.122.77 23:31, 5. Jan. 2014 (CET)

Sollte erledigt sein. Wenn du eine Seite auf Facebook siehst mit "diese Seite ist ein Duplikat"... dann musst du mit dem Mauszeiger dort drüber gehen und kannst dann beeinspruchen, dass es ein Duplikat sei.--Hokanomono (Diskussion) 01:25, 6. Jan. 2014 (CET)

Ich sehe keinen Unterschied zu gestern. Bei "Ittigen" hatte ich angegeben, dass es sich NICHT um ein Duplikat von "Worblaufen" handle. Bei "Worblaufen" wollte ich dann angeben, dass DIESE Seite ein Duplikat von "Ittigen" ist. Das eine ging zwar, aber es muss wohl noch bestätigt werden. Das andere habe ich gar nicht hingekriegt. 178.83.122.77 07:29, 6. Jan. 2014 (CET)

Entweder Ittigen und Worblaufen sind dasselbe oder nicht. Ich gehe davon aus, dass "Duplikat" eine Äquivalenzrelation ist. Es kann nicht Worblaufen dasselbe wie Ittigen sein aber gleichzeitig Ittigen etwas anderes als Worblaufen. (Offenbar ist Worblaufen ein Teil von Ittigen; also nicht dasselbe.) --Hokanomono (Diskussion) 11:15, 6. Jan. 2014 (CET)

Wie mir scheint, verstehst du nicht mehr von FB als ich. So wie ich es interpretiere, ist es wie bei WP: Eine Weiterleitung von Worblaufen auf Ittigen ist nicht dasselbe wie eine Weiterleitung von Ittigen auf Worblaufen. Entsprechend müsste Worblaufen als Duplikat von Ittigen gekennzeichnet werden und keinesfalls umgekehrt. Falls ich mich irren sollte, lasse ich mich von einem FB-Experten gerne belehren. Mich erstaunt, dass es hier keine davon zu haben scheint. :( Die Hoffnung habe ich aber noch nicht aufgegeben. 178.83.122.77 21:14, 7. Jan. 2014 (CET)

Kein Wunder. Wikipedianer sind Tag und Nacht in der Wikipedia, sind demnach Wikipedia-süchtig und haben für nichts anderes mehr Zeit. Diejenigen, die sich in der WP als Facebookler outen, nehmen die WP also nicht ernst ;-). --Filzstift  10:20, 9. Jan. 2014 (CET)
Ich hab zwar einen Facebook-Account, aber mir sind diese Orte- und Personen-Seiten komplett wurscht. Ich will nicht kennzeichnen, wo ich geboren oder zur Schule gegangen bin oder zuletzt gepupst hab, und wenn ich Informationen über die Orte suche, geh ich auch bestimmt nicht zu Facebook. Vielleicht bin ich da ja nicht der einzige. --Eike (Diskussion) 10:29, 9. Jan. 2014 (CET)

Rückgabe defekter Rasierapparat

Hallo zusammen, habe im August bei Lidl einen Rotationsrasierer gekauft, für knapp 20 Euro (vielleicht 18,95 oder 19,80 oder so). Den Kassenbon habe ich nicht mehr (wer denkt schon an sowas!). Vorgestern habe ich das Ding (nach Vorschrift) aufgemacht und abgespült; das soll man alle zwei Monate machen, ich hatte ihn aber selten benutzt. Dabei ist eine der haarfeinen Befestigungsdöpse (offensichtlich gegossen und so dünn, dass "Sollbruchstelle" sich schon zu robust dafür anhört) abgebrochen. Komme in den nächsten Tagen in die Gegend und Filiale, wo ich das Ding gekauft hatte. Habe ich da eine Chance? Danke, --194.94.133.9 10:54, 3. Jan. 2014 (CET)

Liste geflügelter Worte/P#Probieren geht über studieren. --Vsop (Diskussion) 10:58, 3. Jan. 2014 (CET)
Eben. Hängt alles von der Dynamik des Augenblicks ab. Du kannst es mit Charme versuchen (treuer Blick, Hilflosigkeit) oder mit offensivem Auftreten (Filialleiter an Kassiererin: "Mach, dass der komische Typ Ruhe gibt!"). Viel Glück! Dumbox (Diskussion) 11:05, 3. Jan. 2014 (CET)
Und fürs nächste Mal, daran denken die Kassenzettel immer aufzuheben, daran denkt eigentlich jeder, weil gemeinhin bekannt ist, dass Rückgabe damit leichter geht und im Allgemeinen beim Kauf auch dazugesagt wird, aufheben falls mal was ist. --87.148.73.244 11:05, 3. Jan. 2014 (CET)
Also, ich denk da auch regelmäßig nicht dran. Wenn ich einen guten Tag hab, schmeiß ich den Kassenzettel in irgendeine Schublade, und es ist fraglich, dass ich ihn bei Bedarf wiederfinden würde. --Eike (Diskussion) 11:10, 3. Jan. 2014 (CET)
In meinem Haushalt gibt es da zwei Extreme. Ich werfe alles weg, und meine bessere Hälfte hebt alles auf: Plastiktüten voller ausgeblichener Zettel aus den letzten Jahrzehnten. Eines so unproduktiv wie das andere... Dumbox (Diskussion) 11:15, 3. Jan. 2014 (CET)
Ich bewahre alle Kassenzettel auf, sortiere aber regelmäßig. Wenn die Garantie vorbei ist oder der Händler pleite, dann fliegt die Rechnung in den Müll. --Rôtkæppchen68 11:37, 3. Jan. 2014 (CET)
So soll es ja auch sein. Nur gut, dass mir Neid fremd ist! ;) Dumbox (Diskussion) 12:08, 3. Jan. 2014 (CET)
ich scanne die alle ein und speichere sie auf 2 festplatten in 2 verschiedenen computern und einmal im Monat kommt alles auf die älteste von 5 DVDs... der Betreuungsrichter wär so stolz auf mich, dass ich gleich als Amtsarzt anfangen könnte... :) --Heimschützenzentrum (?) 12:16, 3. Jan. 2014 (CET)

Dass Federn für Rastnasen schraubbar und damit auswechselbar ausgeführt werden ist bei einem Endpreis unter 20 € für einen kompletten Rasierer nicht möglich. "Haarfein" ist ziemmlich unmöglich und an der beschriebenen Stelle auch äusserst unüblich. Und die Geschichte daher unglaubwürdig. Wahrscheinlicher ist, dass sich der Fragesteller ungeschickt angestellt hat (wie schon im Umgang mit der Rechnung) und nun versuchen will, die paar € für eine Neuanschaffung auf die Rücken aller anderen Konsumenten zu verteilen. --84.178.63.216 12:04, 3. Jan. 2014 (CET)

Es ist lieb von dir, dass du über den Fragesteller spekulierst. Um eine Feder geht es nicht; die einzige Feder saß in der Mitte und ist völlig intakt. Ob die Dinger "Rastnasen" heißen, weiß ich nicht. Wenn "Rastnasen" nicht haarfein sein können, geht es nicht um eine "Rastnase". Die genauen Bezeichungen für die Einzelteile des Rasierers kenne ich nicht. Deinem Konsumentenrücken mein herzliches Beileid zum eventuell bevorstehenden Verlust, allen anderen Dank für Trost und Tipps! 194.94.133.9 12:20, 3. Jan. 2014 (CET)
Da war nichts lieb gemeint. Es ging eher um Aufklärung, was ein gewisses Mass an Verständnisvermögen voraussetzt. Bei jemanden der spekuliert, dass ein Rasierer nur mit einer einzigen Feder gemacht sein kann, ist das nicht gegeben. --84.178.63.216 12:29, 3. Jan. 2014 (CET)
Er hat nichts von Feder geschrieben und das "lieb" war Sarkasmus. Vielleicht solltest du erstmal lesen lernen, bevor du hier rumpöbelst. Aber auch dann könntest du es uns einfach ersparen. Danke. --Eike (Diskussion) 12:31, 3. Jan. 2014 (CET)
Ich habe was von Feder geschrieben, quasi als Service (erst genauer lesen, dann verstehen). Und zu spekulieren gab es übrigens wenig, denn: „Den Kassenbon habe ich nicht mehr“ macht die Sache schon ziemlich klar. --84.178.63.216 12:40, 3. Jan. 2014 (CET)
Du hast dich verspekuliert: "Bei jemanden der spekuliert, dass ein Rasierer nur mit einer einzigen Feder gemacht sein kann, ist das nicht gegeben." --Eike (Diskussion) 12:46, 3. Jan. 2014 (CET)
Das war lediglich eine Wortwiederholung um in Deinem Wortschatzbereich zu bleiben. Quasi wieder eine Art Service um das schon vorher erkannte Verständnisproblem nicht noch weiter zu strapazieren. --84.178.63.216 12:49, 3. Jan. 2014 (CET)
Ziemlich klar: Ja - wenn ich schreibe, dass ich den Kassenbon nicht mehr habe, will ich damit ausdrücken, dass ich den Kassenbon nicht mehr habe.
Dass nur eine Feder in dem Apparat sei, hatte ich weder geschrieben noch vermutet. Da ich den Apparat nicht ganz demontiert habe, kann ich über die Gesamtfedernzahl keine wirklich fundierte Aussage treffen. Deshalb hatte ich zu diesem gesamtgesellschaftlich sicherlich relevanten, mir aber eben nicht zugänglichen Thema gar nichts gesagt. Auch allgemein hatte ich kein "Verständnisvermögen" meinerseits postuliert. Ich gehöre zu der Minderheit von Männern, die sich auch mal elektrisch rasieren, ohne wirklich Rasierapparatebau studiert zu haben. Jedenfalls werde ich nach Dumbox' Rat (ganz oben) es morgen versuchen. Gegebenenfalls wird der ADKR (Allgemeine Deutsche Konsumentenrücken) dann meine Ungeschicklichkeit mittragen müssen. Darüber, ob ich nun das Nicht-Aufheben des Kassenbons als moralisch minderwertiges Verhalten betrachten und bereuen soll, werde ich noch nachdenken. Danke, 194.94.133.9 12:57, 3. Jan. 2014 (CET)
Ich befürchte ja, sie werden ihn nicht umtauschen. Sie müssen wohl nicht, weil du nicht nachweisen kannst, dass du da gekauft hast, und viel Kulanz wird in dem Preis nicht inbegriffen sein. Kannst ja mal berichten, wie's gelaufen ist! --Eike (Diskussion) 13:01, 3. Jan. 2014 (CET) PS: Den mit den Verständnisproblemen lass ich mal außen vor (AKA EOD), das führt zu nichts, solange er daran nicht intensiv gearbeitet hat.
Na ja, wenn die Lidls nicht wollen, habe ich eben die knapp 20 Euro (minus Gegenwert des bisherigen Gebrauchs) verloren. Aber dafür den Korü nicht belastet...
Frohes Neues noch an alle, tschüs, 194.94.133.9 14:02, 3. Jan. 2014 (CET)
Und? Haben sie ihn zurück genommen? Manchmal sind sie da ja durchaus kulant. --StYxXx 02:59, 6. Jan. 2014 (CET)
Dem deutschlandweiten Interesse entsprechend zwischenmelde ich, dass ich noch gar nichts weiß. Ich war im Laden, wo man aber nach einer Sechs-Wochen-Frist keine Chance mehr hat und an die Hotline verwiesen wird. Die Hotline hat sich alle Daten geben lassen und will zurückrufen oder -mailen. Die Stimme klang eher kulant. Sonst ist es wie bei den Ratsnasen - keine Ahnung! Vielleicht schauen sie jetzt nach, ob ich (nein) oder jemand an der Adresse (auch nein) schon mal / schon öfter Sachen zurückgeben wollte. Ich fahre nun wieder aufs Land, wo ich kein Netz habe. Freitag melde ich mich wieder, wenn dann diese Frage noch hier steht... 194.94.133.9 15:16, 7. Jan. 2014 (CET)
Das Interesse an deinem Ding geht sogar über Deutschland hinaus. Schliesslich (nicht schließlich) gibt's 500m von mir entfernt auch einen Lidl ;-). --Filzstift  15:35, 7. Jan. 2014 (CET)
Zurück vom platten Lande, wo das einzige Netz das ist, das dazu dient, Rehe und Erdbeeren voneinander zu trennen (was auch nicht immer funktioniert, aber um die Jahreszeit sind eh mehr Rehe da), aber keine weiteren Nachrichten von Lidl's Kulanz. Tschüs, 194.94.133.9 10:45, 10. Jan. 2014 (CET)
Letztes Kapitel:
Lidl ist kulant. Ich bekam einen Brief, man entschuldige sich für die mangelnde Qualität des Geräts und hoffe doch, mich in Zukunft als Kunden zu behalten und so weiter; ich solle mit dem Brief den Apparat in einer beliebigen Filiale zurückgeben und bekomme auch ohne Kassenbon den Kaufpreis von 19.99 erstattet. Genau das ist dann geschehen. Wirklich reich bin ich dadurch nicht geworden; wirklich arm ist wohl auch Lidl dadurch nicht geworden; der Konsumentenrücken wird es ertragen. Danke für das allgemeine Interesse und tschüs, NeulichAls194941339 (Diskussion) 11:52, 13. Jan. 2014 (CET)
Ich find's gut; und unser Rücken wird's wohl aushalten. Und willkommen als neuer Wikipedia-Nutzer! ;o) --Eike (Diskussion) 14:15, 13. Jan. 2014 (CET)

Zusammenbruch des Chip-TAN-Verfahrens

Wir kennen alle das Chip-TAN-Verfahren aus den Online-Überweisungen. Es geht um die Sicherheit der User, die mit dem TAN-Generator überweisen.

Seit gestern werden z.B. bei der Sparkasse hilflose Hinweise angezeigt, dass man nur noch sicher sei, wenn man die im Generator angezeigten Informationen mit den realen Überweisungsdaten abgleicht. Zitat: "Vergleichen Sie beim chipTAN-Verfahren immer die im Display des TAN-Generators angezeigten Empfänger-daten mit den Auftragsdaten Ihrer Überweisung. Wenden Sie sich im Zweifelsfall bitte an unsere Hotline."

Eigentlich dürfte beim regulären Funktionieren des Verfahrens keinerlei Abweichungen zwischen den am Bildschirm eingegebenen Daten und den vom Generator angezeigten Daten entstehen. Wenn die Banken darauf hinweisen, die Übereinstimmung zu kontrollieren, sollte das doch auf einen ernsten Fehler im System hinweisen. (Bitte keine Sinnlos-Antworten, nur Leute die wissen was sie sagen.) 88.73.205.89 16:43, 29. Dez. 2014 (CET)

Der Hinweis war doch schon immer da?! Der Vorteil des ChipTAN-Verfahrens ist ja gerade, dass das Gerät die Überweisungsdaten nochmal anzeigt, d.h. selbst wenn es jemand schafft, durch einen Man-in-the-Middle-Angriff einem über die Webseite falsche Daten unterzujubeln (also eine manipulierte Flicker-Animation), hast du immer noch die Möglichkeit der Datenverifikation. --Magnus (Diskussion) 16:48, 29. Dez. 2014 (CET)
(BK) Dieser Vergleich ist von Anfang an der Sinn des Systems. Es sind vermehrt Schadprogramme im Umlauf, die versuchen, Überweisungen per Browser zu manipulieren, und die können durch den Vergleich nicht zum Zuge kommen. Hintergründe finden sich z. B. in c't 25/14 (hier Auszüge). --Eike (Diskussion) 16:50, 29. Dez. 2014 (CET)
Wenn man nicht vergleichen müsste brauchte man die Daten gar nicht anzeigen. --Mauerquadrant (Diskussion) 16:55, 29. Dez. 2014 (CET)
Äh. OK. Ich verstehe. Die Frage ist beendet. 88.73.205.89 17:25, 29. Dez. 2014 (CET)
Archivierung dieses Abschnittes wurde gewünscht von: --Eike (Diskussion) 17:41, 29. Dez. 2014 (CET)

komische Zeichen

Welche Zeichen sind auf dieser Briefmarke vom Patentamt zu sehen? 88.73.205.89 22:51, 29. Dez. 2014 (CET)

Datei:DBP 1981 1088 Patentamt.jpg
Ich erkenne diverse Symbole aus Mathematik, Physik und Ingenieurwissenschaften, dazu Symbole aus der Elektrotechnik, lateinische und griechische Buchstaben. Und ein Textilpflegesymbol. --Rôtkæppchen₆₈ 23:16, 29. Dez. 2014 (CET)
Symbole würde ich jetzt darauf antworten. Rotkäppchen hat schon gesagt woüberall die eingesetzt werden. Eben das typische Beigemüse wenn irgendwelche Belege, Zeichungen, Schemas und Berechnungen bei Patenten gebraucht werden.--Bobo11 (Diskussion) 23:24, 29. Dez. 2014 (CET)
Ach so, okay. Ich dachte es sind Zeichen irgendeiner technischen Sprache. --88.73.205.89 02:58, 30. Dez. 2014 (CET)
Archivierung dieses Abschnittes wurde gewünscht von: Yoursmile (Diskussion) 08:07, 30. Dez. 2014 (CET)
kl erg. --Advanceddeepspacepropeller (Diskussion) 08:13, 30. Dez. 2014 (CET)

Nehmen wir mal an, ich sei Ausländer und bekomme die Deutsche Staatsbürgerschaft...

... aber ich will noch mehr. Denn, wir haben ja fünf anerkannte Minderheiten (die verorteten Sorben, Friesen und Dänen und dazu noch die Sinti und Roma) und die Frage ist nun: Kann ich zusätzlich zu meiner neuen Staatsangehörigkeit auch noch eine Zugehörigkeit zu einer der fünf Gruppen anmelden? Wenn Ihr antworten darauf habt, könnten wir zwei Fallkonstellationen unterscheiden: 1. Den Spaßvogel, der unten in der Pfalz unbedingt auch ein schleswigscher Däne sein will und 2. den ehemaligen Argentinier der mit seiner sorbischen Frau im Spreewald lebt. Le Duc de Deux-Ponts (Diskussion) 09:40, 30. Dez. 2014 (CET)

Anmelden? Bei wem? Du darfst jederzeit sagen, dass du Sorbe bist, wenn du das gerne möchtest. Du kriegst deswegen keinen Sorbenausweis oder sowas. --King Rk (Diskussion) 09:51, 30. Dez. 2014 (CET)
Nur der Penibligkeit halber: wenn du die deutsche Staatsbürgerschaft hast, bist Du nicht mehr Ausländer. -- southpark 10:45, 30. Dez. 2014 (CET)
Sorbe zum Beispiel ist ja keine Staatsangehörigkeit, sondern nur (nur?) so eine Art Staatsangehörigkeit der Herzen. Die deutsche Staatsangehörigkeit ist hingegen eine praktische Frage.
Nun die gute Nachricht: Wenn du Ausländer bist, also im Besitz einer Staatsangehörigkeit von draußen, bekommst du heute zur deutschen Staatsangehörigkeit immer gleich noch eine dazu, nämlich die der EU - so gehört es sich...
Und ich denke mal, dass auch der Standesbeamte bei der Eindeutschung eines Sorben wenigstens ein Sorbet spendiert... Hummelhum (Diskussion) 11:03, 30. Dez. 2014 (CET)

Jetzt noch mal ernsthaft: Ich vermute doch, dass an die Zugehörigkeit zu einer anerkannten Minderheit gewisse Rechte geknüft sind. Als Sorbe, darf ich vielleicht die Natur anders nutzen, als Däne braucht meine Partei die 5%-Hürde nicht zu beachten, als Sinti, habe ich vielleicht andere Niederlassungsmöglichkeiten und ich weiß genau, dass ein Friese in Niedersachsen den Dolmetscher ins Deutsche vor Gericht bezahlt bekommt. Also?(nicht signierter Beitrag von Le Duc de Deux-Ponts (Diskussion | Beiträge) 12:22, 30. Dez. 2014 (CET))

Jetzt mal ernsthaft: Deine Frage ist unsinnig. Falls es solche Rechte gibt, und falls die nur Deutschen zustehen, stehen sie selbstverständlich auch dem zu, der nicht von Geburt an Deutscher war, sondern es erst geworden ist. --Eike (Diskussion) 12:31, 30. Dez. 2014 (CET)
Gerichtsdolmetscher werden ganz allgemein vom Staat bezahlt, ob für Friesisch oder Japanisch – einerlei.
Dem SSW kannst du beitreten ganz unabhängig von einer solchen Zugehörigkeit. Und wenn du eine neue Partei mit demselben Status gründen willst, reicht dein persönliches Dänentum sicherlich nicht aus.
Als Sinto hast du allenfalls Nachteile bei Niederlassungsmöglichkeiten, da zahlreiche Vermieter antiziganistische Ausschlusskriterien haben, kommt selbst bei Wohnungsgenossenschaften und Sozialwohnungen vor. Und bis vor einigen Jahren haben das sogar deutsche Gerichte bestätigt. --Chricho ¹ ² ³ 12:39, 30. Dez. 2014 (CET)
Ich hab dir ernsthaft geantwortet. Es gibt kein Register oder ähnliches, in das einzelne Angehörige einer anerkannten Nationalen Minderheit aufgenommen würden, somit gibt es staatlicherseits auch keine Kriterien für die Zugehörigkeit, geschweige denn Möglichkeiten zur Überprüfung. Wer sagen will, er sei Sorbe, der kann das. Und somit auch z. B. im sorbischen Siedlungsgebiet vor Gericht Sorbisch sprechen. Anders sieht es vielleicht aus, wer sich bei einem entspr. Heimat- o. Kulturverein zur Aufnahme bewerben will. --King Rk (Diskussion) 12:41, 30. Dez. 2014 (CET)
Wobei Du auch gerne als Nicht-Friese oder Nicht-Däne mit Behörden in Schleswig-Holstein in Friesisch oder Dänisch oder Plattdeutsch kommunizieren darfst. Es gibt bestimmte Rechte für die Gruppen (was zum Beispiel Kulturförderung oder Sprachförderung oder Parteien angeht) - die kannst du aber in Anspruch nehmen wenn Du an der Kultur oder an der Sprache interessiert bist, nicht wenn du einen imaginären persönlichen Status hast. -- southpark 12:58, 30. Dez. 2014 (CET) (und was das ganze dann mit der deutschen Staatsbürgerschaft zu tun hat? Rätsel über Rätsel).

Wobei ich mich frage, warum ein Standesbeamter sowas fragen muss. -- southpark 14:31, 30. Dez. 2014 (CET)

Ich halte Behauptungen, Angehörige einer Minderheit besäßen keine besonderen persönlichen Rechte, für falsch. Beispielsweise garantiert das schleswig-holsteinische Wahlgesetz sämtlichen Parteien der dänischen Minderheit, nicht nur dem SSW, eine Befreiung von der 5-Prozent-Hürde (§ 3 (1)). Es könnten sich also ausreichend viele Personen, die sich als Angehörige der dänischen Minderheit sehen, zu einer Partei zusammenschließen und den Minderheitenschutz geltend machen. Ich denke, dass in der Frage das Verwaltungsgericht des Bundeslandes die Zugehörigkeit zur Minderheit prüfen müsste. --BlackEyedLion (Diskussion) 14:42, 30. Dez. 2014 (CET)
Wobei das eben kein persönliches Recht ist - als Angehöriger der dänischen Minderheit in der CDU habe ich keinerlei Zusatzrechte ebenso wie ich als Nicht-Däne im SSW trotzdem unter deren Wahlgesetz falle. -- southpark 14:55, 30. Dez. 2014 (CET)
Nationale_Minderheit ("keine allseits akzeptierte Definition dieses Begriffs", und "Das Rahmenübereinkommen zum Schutz nationaler Minderheiten vom 1. November 1995 enthält hingegen keine Definition des Begriffs; dies bleibt den nationalen Übernahmen vorbehalten."); Dänische_Minderheit_in_Deutschland#Status. Es scheint "Zugehörigkeit anmelden" geht so nicht - man muß schon deutscher Däne, Friese, Sorbe, Sinti, Roma, sein. Wie im Einzelfall festgestellt, sich ausgewiesen wird, ob jemand dazugehört oder nicht? Was sind "nationale Übernahmen"?? --217.84.74.142 14:59, 30. Dez. 2014 (CET)
Wobei auch da, wie schon mehrfach geschrieben: die Schutzrechte wirken für die Gruppe, es gibt keine Rechte, die direkte Einzelpersonen zugeordnet sind. Weder muss ich Däne sein, um eine dänische Schule zu besuchen, noch muss ich Däne sein, um in den SSW einzutreten. -- southpark 15:25, 30. Dez. 2014 (CET)
Wie auch schon mindestens einmal geschrieben: Was passiert, wenn ich und meine Kumpels in Schleswig-Holstein eine Partei gründen und beantragen, als (weitere) Vertreter der dänischen Minderheit von der Fünf-Prozent-Hürde befreit zu werden? (Ich vermute, das ist gar nicht geregelt.) --Eike (Diskussion) 15:30, 30. Dez. 2014 (CET)
Das Landesverfassungsgericht Schleswig-Holstein sagt dazu: Der SSW ist eine Partei der dänischen Minderheit, denn er ist aus der Minderheit hervorgegangen, wird gegenwärtig personell von der Minderheit getragen und ist programmatisch von ihr geprägt....
aa) Das Hervorgegangensein des SSW aus der dänischen Minderheit wird durch die historischen Gegebenheiten belegt und spiegelt sich auch in der Historie des Landeswahlgesetzes wider.
bb) Die personelle Verknüpfung des SSW mit der dänischen Minderheit wird insbesondere deutlich aus der Doppelmitgliedschaft einer großen Anzahl von Personen, die sowohl im SSW als auch in weiteren Organisationen der Minderheit engagiert sind.
cc) Die programmatische Prägung durch die Minderheit ergibt sich aus der Satzung des SSW, seinen Programmen und seinem Zusammenwirken mit den örtlichen Vereinigungen in seinem Tätigkeitsgebiet Südschleswig und Helgoland, dem angestammten Siedlungsgebiet der dänischen Minderheit
Der eine Punkt, wo es um Personen geht bezieht sich auf "Doppelmitgliedschaft mit anderen dänischen Organisationen" -- southpark 16:17, 30. Dez. 2014 (CET)
Aus meiner Sicht bringt das nicht viel Neues. Es muss programmatisch passen - kriegen wir hin - und eben eine "personelle Verknüpfung [...] mit der dänischen Minderheit" geben. Datt wird schwierig. Das "Hervorgegangensein" (was für ein Wort) würde sich m. E. daraus ergeben. (Für mich hört sich das alles nach tönernen Füßen an, aber das ist nochmal eine andere Frage.)
Wollen wir den Abschnitt wegen Quatschigkeit der Frage schließen? Antwort: Es ist genauso wie bei einem, der schon immer deutsch war.
--Eike (Diskussion) 18:30, 30. Dez. 2014 (CET)
Ich glaube faktisch musst du vor allem zeigen, dass die dänische Minderheit sich durch dich vertreten fühlt. Ich glaube nicht, dass das für jemand außer dem SSW geht, außer der baut in den nächtsen Jahren echt viel Mist. Spannender könnte es bei den Sorben werden. Brandenburg hat im Wahlgesetz einen ähnlichen Passus wie Schleswig-Holstein und meines Wissens noch keinen sorbischen Platzhirsch. Und ja, ich denke wir haben das Thema jetzt gründlich durcherörtert und können schließen. -- southpark 18:33, 30. Dez. 2014 (CET)
Archivierung dieses Abschnittes wurde gewünscht von: --Eike (Diskussion) 18:39, 30. Dez. 2014 (CET)

Feldkaveln

In Medow (Goldberg) drüber gestolpert: Was bitte sind Feldkaveln? --Concord (Diskussion) 14:52, 30. Dez. 2014 (CET)

Teilstück eines Grundstücks. --Magnus (Diskussion) 14:55, 30. Dez. 2014 (CET)
Anscheinend noch im Niederländischen gebräuchlich. nl:Kavel (grond) verweist auf Flurstück. --Komischn (Diskussion) 15:28, 30. Dez. 2014 (CET)
Ah, Dankeschön an beide! Das klärt es. --Concord (Diskussion) 17:03, 30. Dez. 2014 (CET)
Archivierung dieses Abschnittes wurde gewünscht von: --Concord (Diskussion) 17:03, 30. Dez. 2014 (CET)
Auch wenn Du es schon auf "erledigt" gesetzt hast, gebe ich noch ein paar Hintergrundinformationen, nachzulesen im Meckl. Wörterbuch, Bd. 4, Sp. 202 f. (Zitate aus anderen Werke führe ich gesondert an):
"Kawel" (bzw. Kabel) bedeutete "eig. Los, ursprünglich in Form eines Holzstäbchens mit der darin eingeschnittenen Hausmarke". Lisch bemerkt dazu 1855 in den Meckl. Jahrbüchern (Bd. 20, S. 133 f.): "Homeyer forscht der Etymologie und der Bedeutung des Wortes Loos nach und beleuchtet die Stellen in den alten deutschen und nordischen Gesetzen. Er ist so glücklich gewesen, auf den Inseln Hiddensee und Föhr und zu Peenemünde noch Reste des alten Loosens aufzufinden, welches mit einem alten norddeutschen Ausdrucke Kaveln genannt wird. [...] Diese uralte deutsche Sitte des Verloosens oder Kavelns durch hölzerne Loose findet sich auch noch in Meklenburg. Es ist schon oben gesagt, daß man in dem Dorfe Retschow die Hausmarken auch zu den Loosen anwendet, die man Kaveln nennt. Dieser Gebrauch ist auch noch im Dorfe Börgerende an Rethwisch, bei Doberan, am Strande der Ostsee, im Gebrauche. Man nimmt runde Haselzweige von etwa 1/2 Zoll Durchmesser, schneidet aus diesen Stücke von 1 1/4 bis 1 1/2 Zoll Länge und schneidet in die Rinde die Hausmarken der Gehöfte; mit diesen Loosen wird dann "gekavelt", wenn z. B. die Wiesen verlooset werden." Im Band findest Du auch eine Abbildung eines solchen Holzstückchens.
Weiterhin hat Kawel dann die Bedeutung für "das Erloste, bes. ein Acker-, Wiesen- oder Waldstück" (Meckl. Wb.) angenommen, dazu folgender Beleg: "ein durch das Loos jemanden zugefallenes Stück, heißt eine Cawel". Weiterhin bezeichnete das Wort in Parchim dann sogar "einen Stadtteil insofern, als einst die Stadt in drei Herdschaften zu je acht Kaweln geteilt war, deren jede ihren Anteil an der Feldmark gemeinsam auslost". Es konnte ebenfalls die "Parzelle einer Gemeindewaldung" bezeichnen, schließlich ein "zu beliebigem Zweck abgemessenes und abgeteiltes Flurstück, sei es Acker, Wiese, Moor oder Wald". Mi (d. i. Friedrich Sibeth) gibt in seinem "Wörterbuch der Mecklenburgisch-Vorpommerschen Mundart" 1876 daher auch "Kaweln, (w-b) loosen" und allgemein "Kawel ein Ackertheil" an (S. 40). Generell konnten damit auch für Pflug oder Haken abgeteilte Ackerstücke gemeint sein: "twölf Schritt wir 'n Kawel (beim Pflügen), söß Schritt wir ümmer ne Rod' [Anm.: zu Rod' siehe Rute (Längenmaß) und Quadratrute]" (zit. n. Mecklb. Wb.).
Schließlich ist mit Kawel "überhaupt Anteil, Teil, Stück, Gruppe" gemeint, "bei der Erbteilung der Anteil, den jeder der Söhne aus der Erbmasse durch den ältesten Sohn zugewiesen erhielt, nachdem dieser das ihm Zustehende im voraus genommen hatte." (Meckl. Wb.)--IP-Los (Diskussion) 19:08, 30. Dez. 2014 (CET)

Da diese Straße wohl keine Verbindung zum schweizerischen Straßennetz hat, stellt sich die Frage, welche Verkehrsregeln auf ihr gelten – deutsche oder schweizerische? --79.228.217.204 22:02, 30. Dez. 2014 (CET)

Deutsche Gesetze gelten in der Schweiz nicht. Für diese Strasse hat man per Staatsvertrag eine Ausnahme vereinbart [16]. --Figugegl (Diskussion) 22:09, 30. Dez. 2014 (CET)
Und damit nicht jeder es selbst raussuchen muss: Art. 8 Abs. 1: „Auf der Verbindungsstrasse gilt das deutsche Strassenverkehrsrecht (…)“. --= (Diskussion) 22:18, 30. Dez. 2014 (CET)
Danke, hab's mal im Artikel erwähnt.--79.228.217.204 22:32, 30. Dez. 2014 (CET)
Es sind auch die deutschen Beamten und Gerichte dafür zuständig, die deutschen Gesetze durchzusetzen. Ausnahmen sind für Schweizer Bürger festgelegt.
"Das gilt auch für Gegenstände, die auf schweizerisches Gebiet neben die Verbindungsstrasse geraten, sofern sie unverzüglich wieder auf diese zurückgebracht werden." - großartig, alles geregelt. Falls ein Laster umkippt und Ware herausfällt, muss diese nicht verzollt werden wenn sie "unverzüglich" zurückwandert. --mfb (Diskussion) 02:14, 31. Dez. 2014 (CET)
"Dieser Vertrag gilt auch für das Land Berlin" (Art. 23) ist auch etwas... merkwürdig. --mfb (Diskussion) 02:14, 31. Dez. 2014 (CET)
Die Berlin-Klausel war damals (25. April 1977) üblich und notwendig. --Rôtkæppchen₆₈ 02:18, 31. Dez. 2014 (CET)
Nein, nicht für Schweizer Bürger, sondern für alle Angelegenheiten, bei denen jemand, der in der Schweiz gemeldet ist (kann auch ein Deutscher sein!) Beschuldigter ist. Wenn also ein in Riehen lebender Deutscher auf der Straße einen schweren Unfall verursacht, wird die Sache für alle Beteiligten vor einem Schweizer Gericht verhandelt. -- Janka (Diskussion) 04:15, 31. Dez. 2014 (CET)
Archivierung dieses Abschnittes wurde gewünscht von: Figugegl (Diskussion) 11:36, 31. Dez. 2014 (CET)

Jahresanfang

Wo fängt das neue Jahr zuerst an in Deutschland oder Neuseeland


--~~ 91.48.174.110 23:10, 31. Dez. 2014 (CET)--91.48.174.110 23:10, 31. Dez. 2014 (CET)--91.48.174.110 23:10, 31. Dez. 2014 (CET)

In Neuseeland natürlich, dort ist es doch viel später als hier. --Neuanmeldungsfetisch disk. 23:28, 31. Dez. 2014 (CET)
In der Infobox des Artikels Neuseeland steht etwas von Zeitzone UTC+12 im Südwinter und UTC+13 im Südsommer. Deutschland liegt im Nordwinter in der Zeitzone UTC+1 und im Nordsommer in UTC+2. Damit ist Neuseeland Deutschland zzt. 12 Stunden voraus und es stimmt, was die Radionachrichten heute den ganzen Tag gemeldet haben: In Neuseeland wurde das neue Jahr bereits begrüßt. --Rôtkæppchen₆₈ 23:50, 31. Dez. 2014 (CET)
Archivierung dieses Abschnittes wurde gewünscht von: Damit wurde die Frage beantwortet. --112.198.83.1 03:10, 1. Jan. 2015 (CET)

Zitat gesucht

Ich könnte schwören, dass es da ein Zitat aus der Romantik, der Aufklärung oder des Sturm und Drang gibt. Der genaue Wortlaut ist mir nicht mehr bekannt. Aber inhaltlich ist das Zitat quasi eine Aufforderung, dass jeder Mensch eine Autobiographie schreiben sollte, so banal sie auch scheinen mag. Denn nur so könne man die gesamte Vielfalt des Lebens, mit all seiner Freude und Tragik erfassen. Dachte, dass ich das in meiner Schulzeit als ein Zitat von Goethe aufgeschnappt habe. Jetzt, da ich selber gesucht, aber nichts gefunden habe, bin ich mir gar nicht mehr so sicher. Vielleicht spielt das Gedächtnis mir einen Streich (Erinnerungsverfälschung).

--93.128.185.152 17:01, 31. Dez. 2014 (CET)

Wilhelm Dilthey: Das Erleben und die Selbstbiographie (1905-11) weiter Googeln !--G-Michel-Hürth (Diskussion) 17:22, 31. Dez. 2014 (CET)

Gibt natürlich auch die gegenteilige Auffassung. Ein Schriftsteller, Thomas Mann?, meinte einmal bei der Besprechung eines Romans eines Kollegen, das Buch sei ja so furchtbar langweilig, man könne meinen, der Autor habe das alles selber erlebt ... --178.8.63.191 21:56, 31. Dez. 2014 (CET)

ich glaube, du meinst den Anfang von Thoreaus Walden: In most books, the I, or first person, is omitted; in this it will be retained; that, in respect to egotism, is the main difference. We commonly do not remember that it is, after all, always the first person that is speaking. I should not talk so much about myself if there were anybody else whom I knew as well. Unfortunately, I am confined to this theme by the narrowness of my experience. Moreover, I, on my side, require of every writer, first or last, a simple and sincere account of his own life, and not merely what he has heard of other men's lives; some such account as he would send to his kindred from a distant land; for if he has lived sincerely, it must have been in a distant land to me. --Edith Wahr (Diskussion) 16:43, 1. Jan. 2015 (CET)